100% found this document useful (1 vote)
3K views464 pages

05-0127 GRE Manual 8.0

Uploaded by

Dhanendra Singh
Copyright
© © All Rights Reserved
We take content rights seriously. If you suspect this is your content, claim it here.
Available Formats
Download as PDF, TXT or read online on Scribd
100% found this document useful (1 vote)
3K views464 pages

05-0127 GRE Manual 8.0

Uploaded by

Dhanendra Singh
Copyright
© © All Rights Reserved
We take content rights seriously. If you suspect this is your content, claim it here.
Available Formats
Download as PDF, TXT or read online on Scribd
You are on page 1/ 464

Manual for the

GRE Version 8.0


®

DO NOT DISTRIBUTE
Copyright © 2015 by TPR IP Education Holdings, LLC. All Rights Reserved.

Version 8.0

All rights reserved. No part of this manual may be reproduced for distribution to a third party in any
form or by any means, electronic or mechanical, including photocopy, recording, or any information
retrieval system, without the prior express written consent of the publisher, The Princeton Review.

This manual is for the exclusive use of The Princeton Review course students, and it is not legal for resale.

800-2Review

PrincetonReview.com

DO NOT DISTRIBUTE
Acknowledgments
Special thanks to the following for their many contributions to this manual:

Martin Cinke, Alan DeLoriea, Kyle Fox, John Fulmer, Mary Beth Garrick,
Jim Havens, Theodore Klein, Sara Kuperstein, Elissa Morel, Jason Morgan,
Craig Patches, Danny Poochigian, Tim Ricchuiti, and the staff and students
of The Princeton Review.

DO NOT DISTRIBUTE
DO NOT DISTRIBUTE
Table of Contents
Introduction..................................................................................... 1
Lesson 1 Math................................................................................. 7
Lesson 1 Verbal............................................................................... 33
Lesson 2 Math................................................................................. 45
Lesson 2 Verbal............................................................................... 65
Lesson 3 Math................................................................................. 85
Lesson 3 Verbal............................................................................... 111
Lesson 4 Math................................................................................. 129
Lesson 4 Verbal............................................................................... 169
Lesson 5 Math................................................................................. 189
Lesson 5 Verbal............................................................................... 215
Lesson 6 Math................................................................................. 233
Lesson 6 Verbal............................................................................... 265
Lesson 7 Math................................................................................. 281
Lesson 7 Verbal............................................................................... 307
Lesson 8 Math................................................................................. 327
Lesson 8 Verbal............................................................................... 379
Lesson 8 Essays............................................................................... 407
Math Practice.................................................................................. 419
Between Now and Test Day............................................................. 443

DO NOT DISTRIBUTE © TPR Education IP Holdings, LLC. |v


DO NOT DISTRIBUTE
Introduction

DO NOT DISTRIBUTE
MANUAL FOR THE GRE

Global Technique #1: Take the Easy Test First


Within a section, each question counts equally toward your score. However, some questions will be easy
and others will be difficult. The beauty of the GRE is that you can answer questions in any order you like.
A question you can nail in 25 seconds is worth just as much as a tough question. To maximize your score,
leave the questions you don’t like for last. If you are going to run out of time, make sure the questions you
end up answering with a letter of the day are the ones you didn’t want to work on anyway. Skip early and
skip often.

Global Technique #2: Scratch Paper


One of the genuinely few helpful tools ETS gives you on the GRE is scratch paper. Scratch paper can help
you to stay organized and efficient over the course of a four-hour test.

For each different question type, you will learn how to organize the information and answer all questions as
efficiently and accurately as possible.

Put your set-ups in the Reserve the right side for


upper left corner. scratch work.
1. A a b c d B 32
Clearly number each problem so that 65 x = 1 y = 1 34 2
you can find your work if you return 34 x = 2 y = 1 65 64
to the problem. 34 x = 2 y = 1 65

8
When you’re done with each
2. A 8 + 12 + 4
2 x
2
2 x=4 7
80 B 8 + 12 + 16 16 32
problem, draw a line underneath it, 10 y=6
y - 24 C 16 + 24 + 8 24 24
across the page, so that you have a 56 56
D 16 + 24 + 8
8 56
2 48
clean space upon which to work the EE
next problem.
3. A 1030 250 32
B
Note how every answer choice has
30 30
5 2 25 • 10 16 • 2
C
been checked and every problem has 5•5• 2• 5 4•4•2
D
2 • 2 • 2 • 2 • 2=2 5
its own distinct space. EE 3•2

4.

2 |
DO NOT DISTRIBUTE
© TPR Education IP Holdings, LLC.
INTRODUCTION

Questions 1–7 are Text Completions.

1 Blank 3 Blank 2 Blank


text completions. text completions. text completions.

latch
reflects false obvious wrong — — portray onto
Note that this student 1. A 2. A 3. A x x 4. A x 5. A x
has come up with her B B B x B x x B x x
own words for the C C C x C x C x
blank and has considered D D
and marked every E E
answer choice. fakes increased trend?
6. A 7. A
“Marked” questions B x x x B
are clearly marked on C x x x C
scratch paper. D
E
m
Questions 8–11/12 are Prob. 8. A 9. A 10. A 11 A
Reading Comprehension. 1. Sediment: Climate Hist. B B B B
Each answer choice has 2. Climate affects sediment C C C C
been considered. 3. “ D D D D
4. Cycle peaks at 1300 & 2400 E E E E

Questions 12–15 or care common taciturn lying


13–16 are Sentence 12. A 13. A 14. A 15. A
Equivalence. Note that B B B B
this student has come up C C C C
D D D D
with her own words for
E E E E
every blank and consid-
F F F F
ered every answer choice.

Every question is clearly numbered There is appropriate space


so that the student can easily return between each column of answer
to the question if needed. choices to avoid crowding and
to leave room for notes and marks.

DO NOT DISTRIBUTE © TPR Education IP Holdings, LLC. | 3


MANUAL FOR THE GRE

Global Technique #3: The Mark Button


Reading or calculation errors on a four-hour test are unavoidable. The problem is that a misread question
or a calculation error completely changes the way you see the problem, and once you see a question wrong,
it is almost impossible to see it correctly. As long as you stay with a misread question, you will continue to
see it wrong. Meanwhile, the clock is ticking and you’re not getting any closer to the answer. We call this
“Quicksand.” Once you’re in Quicksand, it is very difficult to get out.

On the flip side, once you’ve spotted the error, solving the problem correctly is often a straightforward
process. A question that bedeviled you for minutes on end may appear to be appallingly obvious later. The
trick is to change the way you see the question while you still have the opportunity to fix it.

Here are a few signs that you are in Quicksand:

• You’ve found an answer, but it is not one of the choices they’ve given you.
• You have half a page of calculations but are no closer to an answer.
• You’ve spent more than four minutes on a problem.
• You’re down to two answer choices and both seem correct.
• You’ve eliminated all of the answers.
• You’re beginning to wonder whether ETS made a mistake.

If you find yourself in any of these situations, you are in Quicksand. Stop what you’re doing and get out.

Step 1  Recognize you are in Quicksand.

Step 2  Mark and move.

Step 3  Distract your brain by doing two or three other questions.

Step 4  Return to the problem and take a second look.

Ways to see the problem with fresh eyes:

• Ask yourself if there are different ways to express the information.


• Can you use the answer choices to help?
• If the path to the right answer is not clear on a second viewing, guess and walk away again.
Why stick with a problem you don’t know how to solve?

4 |
DO NOT DISTRIBUTE
© TPR Education IP Holdings, LLC.
INTRODUCTION

Global Technique #4: Pacing


Speed kills on the GRE. The clock has a way of infecting your brain. Take a section untimed, and in
addition to answering more questions, you’ll make fewer mistakes. The questions don’t get any harder when
there is a clock, yet somehow most test-takers get more wrong. To make matters worse, the questions you
get wrong are likely to have taken you far more time than the ones you got right.

The trick is to take each section as if there is no clock. As long as you are skipping the hard ones and skip-
ping and coming back when you run into resistance on questions you’ve started, you should get very few
questions in a section wrong.

Remember that it is not the number of questions that you answer that determines your score, it is the
number of questions you answer correctly. Accuracy is everything. Ignore the clock. Slow down and work
for accuracy only. If you run into a brick wall, don’t continue to spend time on the problem; go do an easier
one and come back.

There is only one exception, and that is the last two minutes of a section. A skipped question and a wrong
answer count the same. In other words, there is no penalty for “guessing” on a question you don’t know.
When two minutes remain on your clock, stop what you’re doing and bubble in answers to any remaining
unanswered questions. A few lucky guesses will pay off. If you don’t get any of them right, no harm done.

DO NOT DISTRIBUTE © TPR Education IP Holdings, LLC. | 5


DO NOT DISTRIBUTE
Lesson 1
Math

DO NOT DISTRIBUTE
MANUAL FOR THE GRE

HABITS OF EFFECTIVE TEST TAKERS


Question 1
32 m
=
m 2

Quantity A Quantity B
m 8

Quantity A is greater.
Quantity B is greater.
The two quantities are equal.
The relationship cannot be determined from the
information given.

Question 2

At a certain store, shirts sell for $21 each. If a


customer buys a pair of shirts, the second shirt
costs only $9. How much money does the customer
save per shirt by buying a pair of shirts instead of
two shirts separately?

$ 6
$ 9
$12
$15
$21

8 |
DO NOT DISTRIBUTE
© TPR Education IP Holdings, LLC.
LESSON 1 MATH

SCRATCH PAPER 1

DO NOT DISTRIBUTE © TPR Education IP Holdings, LLC. | 9


MANUAL FOR THE GRE

1
Question 3

Four containers of flour are on the table:


1
The first contains of a pound, the second contains
1 3 1
of a pound, the third contains of a pound, and
6 1 9
the fourth contains of a pound. If each container
18
can hold one pound of flour, how many additional
pounds of flour are required to fill all four containers?

2

9
2

3
11

9
25

9
10

3

Question 4

2 1 4 2 5 5
n= + + + + + + x
7 3 9 3 9 7

For the equation above, if n is an integer, which of


the following could be a possible value of x ?

Indicate all such values.

 0
2

63
 1
65

63
 4

 6

10 |
DO NOT DISTRIBUTE
© TPR Education IP Holdings, LLC.
LESSON 1 MATH

SCRATCH PAPER 1

DO NOT DISTRIBUTE © TPR Education IP Holdings, LLC. | 11


MANUAL FOR THE GRE

MATH POE
Question 1
2
On Tuesday, Jasmine eats of a whole apple pie and on Thursday
5
2
she eats of what is left. How much of the pie has not been eaten?
5

4

25
1

5
6

25
9

25
3

5

Question 2

Paul drives from his apartment to his parents’ house and back along the
same route. On the trip to his parents’ house, he travels at an average
speed of 60 miles per hour. On the return trip, he drives at an average speed
of 80 miles per hour. Which of the following is the closest approximation of
Paul’s average speed, in miles per hour, for the round trip?

60.0
68.6
70.0
71.4
80.0

Question 3

1
n=
( 2 ) (5 )
6 13

Quantity A Quantity B
The number of nonzero 3
digits when n is expressed
as a terminating decimal

Quantity A is greater.
Quantity B is greater.
The two quantities are equal.

DO NOT DISTRIBUTE
The relationship cannot be determined from the
12 | © TPR Education IP Holdings, LLC. information given.
LESSON 1 MATH

SCRATCH PAPER 1

DO NOT DISTRIBUTE © TPR Education IP Holdings, LLC. | 13


MANUAL FOR THE GRE

PLUGGING IN
Question 1

Doug is 3 times as old as Miranda and half as old as Liz. If Doug


is d years old, what is the sum of their ages, in terms of d ?

⎛ d⎞
5⎜ ⎟
⎝ 3⎠

⎛ d⎞
7⎜ ⎟
⎝ 3⎠
⎛ d⎞
10 ⎜ ⎟
⎝ 3⎠
⎛ d⎞
7⎜ ⎟
⎝ 2⎠

⎛ d⎞
9⎜ ⎟
⎝ 2⎠
TRIGGER

Trigger: _______________________________________________________________

Question 2

3
If f = , where g ≠ 0 and f ≠ 1, then which of the
g
g−3
following is equal to ?
f −1

g
f
g–f
–f
–g

14 |
DO NOT DISTRIBUTE
© TPR Education IP Holdings, LLC.
LESSON 1 MATH

SCRATCH PAPER 1

1. Recognize the opportunity: PLUG IN!

RESPONSE
2. Set up your scratch paper.
3. Assign an easy number (e.g. 2, 5, 10, 100) to one or more variables.
4. Work through the problem.
5. Find the answer to the question. That’s your target number. Circle it.
6. Check all answer choices.

DO NOT DISTRIBUTE © TPR Education IP Holdings, LLC. | 15


MANUAL FOR THE GRE

1
Question 3

If a factory produces light bulbs at a constant rate of


1,500 light bulbs per minute, which of the following
represents the number of light bulbs it can produce
in t seconds?

1,500t

25t

25

t

750

t

90,000

t

Question 4

If y ≠ –1, y ≠ 0, and y ≠ 1, which of the following

1
represents the reciprocal of ?
1
y−
y

y2 − 1

y

y2 + 1

y

y

y +1

y
2
y −1

y
2
y +1

16 |
DO NOT DISTRIBUTE
© TPR Education IP Holdings, LLC.
LESSON 1 MATH

SCRATCH PAPER 1

DO NOT DISTRIBUTE © TPR Education IP Holdings, LLC. | 17


MANUAL FOR THE GRE

1
Question 5

A group of 10 roommates share the rent for an


apartment equally. If the apartment’s monthly rent is
r dollars and x of the roommates move out, which of
the following is an expression for the additional rent
paid by each remaining roommate?

rx

10(10 − x)

10r

x
r

10(10 − x)

r

10 − x

rx

10 − x

Check your work. For every problem, you should have terms
labeled, a target number circled, and all answer choices checked.

18 |
DO NOT DISTRIBUTE
© TPR Education IP Holdings, LLC.
LESSON 1 MATH

SCRATCH PAPER 1

DO NOT DISTRIBUTE © TPR Education IP Holdings, LLC. | 19


MANUAL FOR THE GRE

Quant Comp Plugging In


Question 1
y≠0

Quantity A Quantity B
–10y –y

Quantity A is greater.
Quantity B is greater.
The two quantities are equal.
The relationship cannot be determined from the
information given.
TRIGGER

Trigger: _______________________________________________________________

Need a weird number? Try FROZEN:

F – Fractions
R – Repeats
O – One
Z – Zero
E – Extremes
N – Negative

Question 2
x>y>0

Quantity A Quantity B
6x 7y

Quantity A is greater.
Quantity B is greater.
The two quantities are equal.
The relationship cannot be determined from the
information given.
20 |
DO NOT DISTRIBUTE
© TPR Education IP Holdings, LLC.
LESSON 1 MATH

SCRATCH PAPER 1

1. Recognize the Opportunity: PLUG IN!

RESPONSE
2. Set up your scratch paper.
3. Plug in an easy number (according to the problem’s restrictions).
4. Eliminate two of answers A, B, and C.
5. Repeat using FROZEN.

DO NOT DISTRIBUTE © TPR Education IP Holdings, LLC. | 21


MANUAL FOR THE GRE

1
Question 3

a, b, and c are consecutive even integers such that


a < b < c.

Quantity A Quantity B
a + c 2b + 2

Quantity A is greater.
Quantity B is greater.
The two quantities are equal.
The relationship cannot be determined from the
information given.

Question 4
a≠0

Quantity A Quantity B
|a – 1| |a| – 1

Quantity A is greater.
Quantity B is greater.
The two quantities are equal.
The relationship cannot be determined from the
information given.

22 |
DO NOT DISTRIBUTE
© TPR Education IP Holdings, LLC.
LESSON 1 MATH

SCRATCH PAPER 1

DO NOT DISTRIBUTE © TPR Education IP Holdings, LLC. | 23


MANUAL FOR THE GRE

Keep Plugging Away


Question 1

If a and b are distinct positive integers, and a is even,


then which of the following must also be even?

2(a + b) – 3
(a – b) + 2
a+b–1
a–b
ab − 2
TRIGGER

Trigger: _______________________________________________________________

Question 2

If p and q are integers, such that p < 0 < q, which of


the following statements must be true?

Indicate all such statements.

 2p < 2q
 p2 < q2
 p + q = 0

24 |
DO NOT DISTRIBUTE
© TPR Education IP Holdings, LLC.
LESSON 1 MATH

SCRATCH PAPER 1

1. Recognize the Opportunity: PLUG IN!

RESPONSE
2. Set up your scratch paper.
3. Plug in an easy number (according to the problem’s restrictions).
4. Eliminate incorrect answer choices.
5. Repeat using FROZEN.

DO NOT DISTRIBUTE © TPR Education IP Holdings, LLC. | 25


MANUAL FOR THE GRE

PLUGGING IN DRILL
Question 1 of 7

Bill is twice as old as Heidi and six years younger


than Mel. If Heidi is h years old, how old is Mel in
terms of h ?

h–4
h+4
2h – 4
2h
2h + 6

Question 2 of 7

Quantity A Quantity B
6 + x 6–x

Quantity A is greater.
Quantity B is greater.
The two quantities are equal.
The relationship cannot be determined from the
information given.

Question 3 of 7

0 < x < 10
0<y<1

Quantity A Quantity B
x – y 9

Quantity A is greater.
Quantity B is greater.
The two quantities are equal.
The relationship cannot be determined from the
information given.

26 |
DO NOT DISTRIBUTE
© TPR Education IP Holdings, LLC.
LESSON 1 MATH

SCRATCH PAPER 1

DO NOT DISTRIBUTE © TPR Education IP Holdings, LLC. | 27


MANUAL FOR THE GRE

1
Question 4 of 7

Eleven years ago, Lauren was half as old as Mike will be in 4


years. If Mike is currently m years old, then what is Lauren’s
current age, in terms of m ?

4m – 11

1
(m + 4) + 11
2
1
(m – 11)
2
11
4m +
2

2m – 7

Question 5 of 7
P<Q

Quantity A Quantity B
Q – P Q– P
3
Quantity A is greater.
Quantity B is greater.
The two quantities are equal.
The relationship cannot be determined from the
information given.

28 |
DO NOT DISTRIBUTE
© TPR Education IP Holdings, LLC.
LESSON 1 MATH

SCRATCH PAPER 1

DO NOT DISTRIBUTE © TPR Education IP Holdings, LLC. | 29


MANUAL FOR THE GRE

1
Question 6 of 7

If the sum of three consecutive odd integers is k, then,


in terms of k, what is the greatest of the three integers?

k−6

3

k−3

3
k

3

k+3

3

k+6

3

Question 7 of 7

Item F costs three times as much as item G, and item


H costs $4 more than one-third the price of item G.

Quantity A Quantity B
The cost of item F The cost of item H

Quantity A is greater.
Quantity B is greater.
The two quantities are equal.
The relationship cannot be determined from the
information given.

30 |
DO NOT DISTRIBUTE
© TPR Education IP Holdings, LLC.
LESSON 1 MATH

SCRATCH PAPER 1

DO NOT DISTRIBUTE © TPR Education IP Holdings, LLC. | 31


MANUAL FOR THE GRE

SUMMARY

1. List A,B,C,D,E on your scratch paper.

RESPONSE
2. Replace the variable(s) with number(s).
TRIGGER

Variables in the answer


3. Work the problem.
choices
4. Identify and circle the target number.
5. Check all answer choices.

1. Set up your scratch paper.

RESPONSE
TRIGGER

2. Plug in an easy number.


Quant Comp with variables
3. Eliminate two of answers A, B, and C.
4. Repeat using FROZEN.

1. Set up your scratch paper.

RESPONSE
TRIGGER

2. Plug in an easy number.


“Must be”
3. Eliminate answer choices.
4. Repeat using FROZEN.

32 |
DO NOT DISTRIBUTE
© TPR Education IP Holdings, LLC.
Lesson 1
Verbal

DO NOT DISTRIBUTE
MANUAL FOR THE GRE

1
Question 1
SCRATCH PAPER
Bob is well known for being _________ because he
frequently gives money to charities.

 critical
 stingy
 cunning
 munificent
 famous
 philanthropic

Ignore the answer choices. Don’t plug the words into


the sentence.

Steps for Sentence Equivalence Questions


1. Find the clues and transition words.
2. Come up with your own word or phrase for the blank. Write that word or phrase down on
your scratch paper.
3. Check each answer choice and use your scratch paper:
✓ an answer that sort of matches your word
× an answer that does not at all match your word
? any word you don’t know

34 |
DO NOT DISTRIBUTE
© TPR Education IP Holdings, LLC.
LESSON 1 VERBAL

GET A CLUE
Who or what is the blank describing?
What else in the sentence provides insight into that person or thing?

The clue is the words or phrases in the sentence that provides


insight into the word or phrase that goes in the blank.

His greatest talent was his _________ : his ability to lie to anyone.

Suddenly he _________ , and no one could tell where he had disappeared to.

Acclaimed by several important artists as a prodigy, Van Vliet was a sculpting _________ .

The _________ professor was so talkative that his rambling lectures would continue long after the students
had left the lecture hall.

Tamson was so gifted a singer that her colleagues were often dazzled by her _________ and failed to
appreciate her other talents.

Art critics have characterized Jackson’s latest work as a _________ of different ideas, all thrown together
with little thought of any unifying theme.

The _________ stories in Brown’s novels, which were written in the early years after the founding of the
United States of America and are frequently the subject of contextual analysis by historians, are noted for
their dark, forbidding tone.

DO NOT DISTRIBUTE © TPR Education IP Holdings, LLC. | 35


MANUAL FOR THE GRE

1
Question 2
SCRATCH PAPER
Filmmaking is a _________ effort, since the director,
cinematographer, writer, editor, and many others
must work together to produce a movie.

 creative
 lucrative
 collaborative
 joint
 concentrated
 glamorous

Question 3

As CEO, Mr. Bluvband distinguished himself


primarily because he was more _________ than his
peers; he remained calm and resolute even during
times of immense volatility in the market at large.

 prosperous
 stalwart
 dismayed
 prescient
 steadfast
 naïve

Question 4

Although most species of fish off the southwest


coast of Africa have died off due to the oxygen-poor
and jellyfish-rich waters, the bearded goby is
actually _________ .

 thriving
 destructive
 withering
 transitory
 flourishing
 picturesque

36 |
DO NOT DISTRIBUTE
© TPR Education IP Holdings, LLC.
LESSON 1 VERBAL

1
TRANSITION WORDS
I really like you, and _______________________________________________.

I really like you, but _______________________________________________.

Contrasts Agrees
although thus
though therefore
despite not only . . . but also
however insofar
nonetheless because
nevertheless and
instead ;
rather :
whereas since
while consequently
but
yet

TIME TRANSITIONS
She used to be poor; now she’s _______________________________________.

His early work was boring, so it’s refreshing to see that his latest piece is _______

_______________________________________________________________.

DO NOT DISTRIBUTE © TPR Education IP Holdings, LLC. | 37


MANUAL FOR THE GRE

1
Question 5
SCRATCH PAPER
Some pundits postulate that the end of the
Cold War _________ one of the major political
parties; many voters had previously viewed that
party as vital because of its role in the fight against
Communism.

 fragmented
 bolstered
 encouraged
 engendered
 debilitated
 blunted

Question 6

Brett’s on stage portrayal of his brash and


outspoken character is so natural that people are
surprised upon meeting him that his true demeanor
is so _________.

 retiring
 blunt
 reserved
 affected
 cocky
 melodramatic

38 |
DO NOT DISTRIBUTE
© TPR Education IP Holdings, LLC.
LESSON 1 VERBAL

SENTENCE EQUIVALENCE DRILL


Question 1 of 5
SCRATCH PAPER
The stunning array of colors present in our natural
world would suggest a seemingly _________ palette
for paint manufacturers to draw on for inspiration,
but only a small fraction of these colors are actually
available.

 fixed
 infinite
 inexhaustible
 realistic
 limited
 essential

Question 2 of 5

James’s _________ work associates were among the


many who trusted his financial savvy and testified
on his behalf in the hearing, only to find themselves
stunned and devastated by his confession on the
final day of the proceedings.

 cynical
 unwavering
 petty
 recent
 steadfast
 capricious

DO NOT DISTRIBUTE © TPR Education IP Holdings, LLC. | 39


MANUAL FOR THE GRE

1
Question 3 of 5
SCRATCH PAPER
Young adults are becoming _________ workers,
unlike their parents who remained at one employer
for life.

 stingy
 cunning
 itinerant
 famous
 peripatetic
 unmanageable

Question 4 of 5

Not even the most _________ traveler can resist the


rustic charms of the small fishing villages of western
Ireland.

 dogged
 crafty
 unstinting
 disgruntled
 jaded
 benevolent

Question 5 of 5

The studio head, naturally _________ , lets his


underlings speak for him during most meetings,
remaining silent until absolutely necessary.

 taciturn
 laconic
 solicitous
 impertinent
 loquacious
 blithe

40 |
DO NOT DISTRIBUTE
© TPR Education IP Holdings, LLC.
LESSON 1 VERBAL

CLUES & TRANSITIONS—NOT JUST FOR SENTENCE EQUIVALENCE!

Steps for Text Completions Questions


1 Find the clues and transition words.
2. Come up with your own word or phrase for the blank.
3. Check each answer choice and use your scratch paper.
• POE: Yes! = ✓ No! = Don’t Know Vocab = ?

Question 1 SCRATCH PAPER


The sudden drop in temperature, the darkening
clouds, and the increasing winds indicated that the
storm was _________.

pervasive
controversial
irrelevant
deliberate
imminent

Question 2

In the eighteenth and nineteenth centuries, the


American chestnut tree flourished along the Eastern
Seaboard, but it had been almost completely
_________ by the middle of the twentieth century.

cultivated
decimated
assuaged
pollinated
disseminated

Question 3

Having bestowed her patronage on many


writers and performers, not least among them
Shakespeare’s theater company, Queen Elizabeth I
was quite aptly labeled a grand _________ of the arts.

connoisseur
critic
ally
regent
judge

DO NOT DISTRIBUTE © TPR Education IP Holdings, LLC. | 41


MANUAL FOR THE GRE

1
Question 4 SCRATCH PAPER
Despite _______ cynicism about the
commercialization of holidays such as Christmas
and Easter, few can resist getting caught up in the
spirit of the season.

pervasive
naïve
disingenuous
commendable
festive

Question 5

Most often, the futurist’s greatest flaw is not that


she has the hubris to suppose she can imagine the
technologies and social structures of tomorrow, but
that she is so _______ that she cannot discern which
traits are so indelibly human that they must remain
unaltered.

immodest
prescient
clairvoyant
myopic
keen

42 |
DO NOT DISTRIBUTE
© TPR Education IP Holdings, LLC.
LESSON 1 VERBAL

TEXT COMPLETION DRILL

Question 1 of 3 SCRATCH PAPER


Though it is not as high profile as a light rail or
subway, increasingly, _________ cities are turning to
a concept called bus rapid transit as an affordable
transportation option.

sprawling
impoverished
resilient
provincial
ambitious

Question 2 of 3

The television critic attributed the enduring


popularity of police and medical “procedurals” such
as CSI and House to the _________ desire to see
mysteries solved and diseases cured.

ephemeral
perennial
nascent
hackneyed
intermittent

Question 3 of 3

The _________ of information about the recently


discovered species meant that ornithologists had
limited familiarity with the bird’s mating and nesting
practices.

bounty
succor
overabundance
paucity
rubric

DO NOT DISTRIBUTE © TPR Education IP Holdings, LLC. | 43


MANUAL FOR THE GRE

STEPS FOR SENTENCE EQUIVALENCE


QUESTIONS
1. Find the clues and transition words.

2. Come up with your own word or phrase for the


blank. Write it down on your scratch paper.

3. Check each answer choice:


Yes! = ✓
No! =
Don’t Know Vocab = ?

STEPS FOR TEXT COMPLETIONS


QUESTIONS
1. Find the clues and transition words.

2. Come up with your own word or phrase for the


blank. Write it down on your scratch paper.

3. Check each answer choice:


Yes! = ✓
No! =
Don’t Know Vocab = ?

44 |
DO NOT DISTRIBUTE
© TPR Education IP Holdings, LLC.
Lesson 2 Math

DO NOT DISTRIBUTE
MANUAL FOR THE GRE

2 EXPONENTS & ROOTS


Question 1

Quantity A Quantity B
12 11
4 −4
48
643
Quantity A is greater.
Quantity B is greater.
The two quantities are equal.
The relationship cannot be determined from the
information given.

Working with Exponents

MADSPM
Rewrite using common bases.
Look for ways to factor.

Question 2

If x > 0, which of the following statements must be true?

Indicate all such statements.

3 −1
 x = x
 x2 = (–x)2
 x–2 < x2

46 |
DO NOT DISTRIBUTE
© TPR Education IP Holdings, LLC.
LESSON 2 MATH

SCRATCH PAPER
2

DO NOT DISTRIBUTE © TPR Education IP Holdings, LLC. | 47


MANUAL FOR THE GRE

2 Question 3
mx
=1
my

If m > 1, which two of the following values for x and


y make the equation above true?

 x= 0
 x = –2
 x= 3
 y= 1
 y = –2
 y = –3

Question 4

If 126 = 2a3b, what is the value of a + b ?

a+b=

Question 5

x5 + x3
=
x 4 + x2

–4x2
–x2
2x
x
x2

Question 6

12 3 3
If = , then what is the value of x ?
20 x 5

5
10
25
36
100

48 |
DO NOT DISTRIBUTE
© TPR Education IP Holdings, LLC.
LESSON 2 MATH

SCRATCH PAPER
2

DO NOT DISTRIBUTE © TPR Education IP Holdings, LLC. | 49


MANUAL FOR THE GRE

2 EXPONENTS & ROOTS DRILL

Question 1 of 3

If 3n = 729 and ( 3n 1 ) ( x n 2 ) = 3 (18 4 ) , which of the


following could be the value of n + x ?

3
6
9
12
15

Question 2 of 3

3 2x + 2 y
Which of the following is equal to 2x 2y
for all
integers x and y ? 3

81−x
81−y
1
81x
81y

Question 3 of 3

Quantity A Quantity B
3 3 3
128 250 2

Quantity A is greater.
Quantity B is greater.
The two quantities are equal.
The relationship cannot be determined from the
information given.

50 |
DO NOT DISTRIBUTE
© TPR Education IP Holdings, LLC.
LESSON 2 MATH

SCRATCH PAPER
2

DO NOT DISTRIBUTE © TPR Education IP Holdings, LLC. | 51


MANUAL FOR THE GRE

2 HIDDEN PLUG INS


Question 1

1
Sarah pays of her monthly income for food,
8
1 1 4
for utilities, for student loans, and of the
8 8 5
remainder for rent. At the end of each month, if
1
Sarah puts of her remaining income into a CD
2
account, what portion of Sarah’s monthly income

does she put into the account?

1 1 7 1 1

8 10 80 16 20
TRIGGER

Trigger: _______________________________________________________________

Question 2

In a certain apartment building, 40 percent of the


units have one bedroom, and the remaining units
have two bedrooms. If 20 percent of the one-
bedroom units and 10 percent of the two-bedroom
units are vacant, what percent of the units in the
building are vacant?

10%
14%
15%
30%
40%

52 |
DO NOT DISTRIBUTE
© TPR Education IP Holdings, LLC.
LESSON 2 MATH

SCRATCH PAPER
2

RESPONSE
Plug In for the unknown value or amount.

DO NOT DISTRIBUTE © TPR Education IP Holdings, LLC. | 53


MANUAL FOR THE GRE

2 PLUGGING IN THE ANSWERS (PITA)


Question 1

Vicken, Roger, and Adam buy a radio. If Roger pays


twice as much as Adam, Vicken pays three times as
much as Roger, and the radio costs $90, how much
does Roger pay?

$10
$20
$30
$45
$65
TRIGGER

Trigger: _______________________________________________________________

Question 2

Mike buys a used car and has it repainted. If the


cost of the paint job is one-fifth of the purchase
price of the car, and if the cost of the car and the
paint job combined is $4,800, then what is the
purchase price of the car?

$800
$960
$3,840
$4,000
$4,250

54 |
DO NOT DISTRIBUTE
© TPR Education IP Holdings, LLC.
LESSON 2 MATH

SCRATCH PAPER
2

1. Recognize the Opportunity: Plug In the Answers (PITA)!

RESPONSE
2. List the answer choices on your scratch paper.
3. Label the answer choices.
4. Plug In (C).
5. Work the problem in bite-sized pieces, making a new column for each new step.
6. Use POE.

DO NOT DISTRIBUTE © TPR Education IP Holdings, LLC. | 55


MANUAL FOR THE GRE

2 Question 3

Gerald is three times as old as his cousin Lucy


and 14 years older than his parrot Polly. If Lucy will
be half as old as Polly in 4 years, what is Gerald’s
current age?

24
36
45
54
60

Question 4

In a high school pep band that consists of forty


students, the number of seniors is five fewer
than twice the number of juniors, and 30% of the
students in the band are neither juniors nor seniors.

Quantity A Quantity B
The number of juniors in 11
the band
Quantity A is greater.
Quantity B is greater.
The two quantities are equal.
The relationship cannot be determined from the
information given.

56 |
DO NOT DISTRIBUTE
© TPR Education IP Holdings, LLC.
LESSON 2 MATH

SCRATCH PAPER
2

DO NOT DISTRIBUTE © TPR Education IP Holdings, LLC. | 57


MANUAL FOR THE GRE

2 PLUGGING IN DRILL
Question 1 of 6

One-half the members of a team are juniors,


one-third are sophomores, and the remainder
are seniors.

Quantity A Quantity B
The number of juniors The number of seniors
on the team on the team
Quantity A is greater.
Quantity B is greater.
The two quantities are equal.
The relationship cannot be determined from the
information given.

Question 2 of 6

If –1 < x < 0, which of the following has the


greatest value?

x3
1

x
1

x
1+x

58 |
DO NOT DISTRIBUTE
© TPR Education IP Holdings, LLC.
LESSON 2 MATH

SCRATCH PAPER
2

DO NOT DISTRIBUTE © TPR Education IP Holdings, LLC. | 59


MANUAL FOR THE GRE

2 Question 3 of 6

1
If ( 3 2 x )
= 3 n , where n and x are integers, what is
9
the value of n in terms of x ?

2x
2x + 2
2x − 2
2x − 3
2x − 9

Question 4 of 6

S is a sequence s1, s2, s3…sn in which every term


after the first is one less than three times the
previous term. If s5 – s3 = 28, which of the following
is the first term in the sequence?

2
3
8

9

1
5
3

60 |
DO NOT DISTRIBUTE
© TPR Education IP Holdings, LLC.
LESSON 2 MATH

SCRATCH PAPER
2

DO NOT DISTRIBUTE © TPR Education IP Holdings, LLC. | 61


MANUAL FOR THE GRE

2 Question 5 of 6

If 20 percent of the trees in a certain park are


evergreens, 40 percent of the non-evergreens are
maple trees, and there are 75 percent as many oak
trees as maple trees in the park, what fraction of the
trees in the park are not maples, oaks, or evergreens?

1

10
3

25
1

5
6

25
1

4

Question 6 of 6

Colby has x dollars more than Mike has, but y dollars


less than Pat has. If Colby has z dollars, which of the
following is an expression for the number of dollars
that Mike and Pat have altogether?

z–y−x
z+y−x
2z + y – x
2z – y + x
2z – y − x

62 |
DO NOT DISTRIBUTE
© TPR Education IP Holdings, LLC.
LESSON 2 MATH

SCRATCH PAPER
2

DO NOT DISTRIBUTE © TPR Education IP Holdings, LLC. | 63


MANUAL FOR THE GRE

2 SUMMARY

The phrases “how much,” “how many,” 1. List the answers on your scratch paper.

RESPONSE
“what is the value of” 2. Label the answer choices.
TRIGGER

or 3. Plug In (C).
An urge to write your own algebraic 4. Work the steps of the problem.
equation 5. Use POE.

Plug In for the unknown value or amount:

RESPONSE
TRIGGER

Answer choices expressed as fractions or If fractions, plug in the common


percentages denominator.
If percentages, plug in 100 for total.

Working with Exponents

MADSPM
Rewrite using common bases.
Look for ways to factor.

Working with Roots

a b= a b

a a
b
= b

64 |
DO NOT DISTRIBUTE
© TPR Education IP Holdings, LLC.
Lesson 2
Verbal

DO NOT DISTRIBUTE
MANUAL FOR THE GRE

2 CLAIMS AND EVIDENCE


It is generally accepted that the color pink has
a calming effect on people who are exposed to
it. It is no surprise that Elizabeth is so calm. Her
entire apartment is painted pink.

Every time that Anahi tries to email a


document that is more than 3 megabytes,
the email is not delivered. When she emails
a document that is less than or equal to 3
megabytes, the email is always delivered.
Therefore, in order to email the weekly sales
summary to the corporate headquarters, Anahi
must email the document in two parts.

The condition of the infrastructure of a country


is a major indicator of the relative wealth of that
nation. If the highways connecting major cities
are in a state of disrepair, it is an indication that
the wealth of that country has fallen below an
acceptable level. So, it must be true that if a
country’s major highways are in good condition,
that country has adequate capital.

The technological conservatism of tennis


racket manufacturers is a reflection of the kinds
of demands they are trying to meet. The only
tennis players who are seriously interested
in innovation and willing to pay for it are
professional players. Therefore, innovation in
tennis racket technology is limited by what
authorities will accept as standard for purposes
of competition in professional tennis.

66 |
DO NOT DISTRIBUTE
© TPR Education IP Holdings, LLC.
LESSON 2 VERBAL

Claim: __________________________________________________________ 2
_______________________________________________________________
Evidence: ________________________________________________________
_______________________________________________________________

Claim: __________________________________________________________
_______________________________________________________________
Evidence: ________________________________________________________
_______________________________________________________________

Claim: __________________________________________________________
_______________________________________________________________
Evidence: ________________________________________________________
_______________________________________________________________

Claim: __________________________________________________________
_______________________________________________________________
Evidence: ________________________________________________________

_______________________________________________________________

DO NOT DISTRIBUTE © TPR Education IP Holdings, LLC. | 67


MANUAL FOR THE GRE

2 COMMON PATTERNS
Contrary to popular belief, high school
students overwhelmingly approve of the high
school administrative staff. We know this to
be true because the student council expressed
admiration for the high school principal and
her staff in the council’s editorial for the school
paper.

Motorboating accidents and ice cream sales


increase at the same time of year. Therefore,
increased consumption of ice cream causes
motorboating accidents.

New York City has a mass transit system


which relies upon buses, subways, and light rail.
Ogdenville, a town with a population of 30,000,
wishes to overhaul its mass transit system. Thus,
Ogdenville should build a mass transit system
which relies upon buses, subways, and light rail.

The sanitation department announced, “We


have solved the city’s pest problem. Poison bait
has been placed throughout the city’s parks.”

68 |
DO NOT DISTRIBUTE
© TPR Education IP Holdings, LLC.
LESSON 2 VERBAL

Pattern: _________________________________________________________
2
Assumption(s): ____________________________________________________
How to strengthen: ________________________________________________
_______________________________________________________________
How to weaken: __________________________________________________
_______________________________________________________________

Pattern: _________________________________________________________

Assumption(s): ____________________________________________________
How to strengthen: ________________________________________________
_______________________________________________________________
How to weaken: __________________________________________________
_______________________________________________________________

Pattern: _________________________________________________________

Assumption(s): ____________________________________________________
How to strengthen: ________________________________________________
_______________________________________________________________
How to weaken: __________________________________________________
_______________________________________________________________

Pattern: _________________________________________________________

Assumption(s): ____________________________________________________
How to strengthen: ________________________________________________
_______________________________________________________________
How to weaken: __________________________________________________

_______________________________________________________________

DO NOT DISTRIBUTE © TPR Education IP Holdings, LLC. | 69


MANUAL FOR THE GRE

2 ARGUMENTS BASIC APPROACH

Arguments Basic Approach


Step 1: Identify the Question Type
Step 2: Work the Argument
Step 3: Know what the Answer Needs to Do
Step 4: Use Process of Elimination

Types of Arguments Questions


• Main Conclusion
• Assumption
• Weaken
• Strengthen
• Inference
• ID the Reasoning
• Resolve/Explain

70 |
DO NOT DISTRIBUTE
© TPR Education IP Holdings, LLC.
LESSON 2 VERBAL

SCRATCH PAPER
2

DO NOT DISTRIBUTE © TPR Education IP Holdings, LLC. | 71


MANUAL FOR THE GRE

2 ASSUMPTION QUESTIONS

Arguments Basic Approach for Assumption Questions


Step 1: Identify the Question Type
• assumes
• presupposition
• relies on
Step 2: Work the Argument
• Conclusion
• Premise
• Gap / Pattern / Assumption
Step 3: Know what the Answer Needs to Do
• connects the premise and conclusion
Step 4: Use Process of Elimination
• out of scope
• extreme language

Question 1

Computers that can be identified and traced are of no


use to thieves. So Maguda Inc. has instituted a plan
to reduce theft by etching the serial number of each
computer both on the frame of the monitor and on
the face of the hard drive where it will be impossible
to overlook.

Which of the following is an assumption made by


Maguda Inc. in formulating its plan?

Thieves do not steal computers that are


of no value to them.
Maguda Inc.’s competitors also plan to
etch serial numbers on their computers.
Thieves are more inclined to steal
computers than any other type of office
equipment.
Etching the serial number on the
computer has no effect on the
performance of the computer.
Imposing stiffer jail sentences for thieves
would not decrease the number of
computers stolen.

72 |
DO NOT DISTRIBUTE
© TPR Education IP Holdings, LLC.
LESSON 2 VERBAL

SCRATCH PAPER
2

DO NOT DISTRIBUTE © TPR Education IP Holdings, LLC. | 73


MANUAL FOR THE GRE

2 WEAKEN QUESTIONS

Arguments Basic Approach for Weaken Questions


Step 1: Identify the Question Type
• weaken
• cast doubt
• undermine
Step 2: Work the Argument
• Conclusion
• Premise
• Gap / Pattern / Assumption
Step 3: Know what the Answer Needs to Do
• attacks the assumption
Step 4: Use Process of Elimination
• out of scope
• extreme language
• strengthens the argument

Question 1

A harmful algal bloom (HAB) occurred in the Gulf


of Mexico three months after unusually heavy farm
runoff was recorded along the Gulf Coast in July
of 2005. Thus, it is evident that farm runoff causes
harmful algal blooms.

Which of the following, if true, most seriously


weakens the argument?

The harmful algal bloom triggered by the


runoff along the Gulf Coast subsided
after a few months.
The growth of phytoplankton in the
ocean is generally limited by the amount
of nutrients available in ocean waters.
No harmful algal bloom occurred within
one year of the unusually heavy farm
runoff recorded off the coast of California
in June of 2004.
Farm runoff often contains key nutrients
on which algae feed.
Harmful algal blooms are often indicated
by red discolorations on the surface of
74 |
DO NOT DISTRIBUTE
© TPR Education IP Holdings, LLC.
the ocean.
LESSON 2 VERBAL

SCRATCH PAPER
2

DO NOT DISTRIBUTE © TPR Education IP Holdings, LLC. | 75


MANUAL FOR THE GRE

2 STRENGTHEN QUESTIONS

Arguments Basic Approach for Strengthen Questions


Step 1: Identify the Question Type
• strengthen
• justify
• defend
Step 2: Work the Argument
• Conclusion
• Premise
• Gap / Pattern / Assumption
Step 3: Know what the Answer Needs to Do
• supports the assumption
Step 4: Use Process of Elimination
• out of scope
• extreme language
• weakens the argument

Question 1

That newer planes can increase the number of paying


passengers for a given airline has been shown by the experience
of Beta Airlines which flies planes less than 5 years old on only
half its routes departing from City Airport. These newer planes
feature larger overhead bins which speed the boarding process.
Since Beta Airlines purchased the newer planes for these routes
three years ago, the number of paying passengers on the routes
with these newer planes has been substantially higher than on
the routes with older planes.

Which of the following, if true, most strengthens the argument?

When all seats have been sold for a flight on one of


the newer airplanes, some passengers may need to
check their carryon baggage at the gate.
When Beta Airlines recently purchased newer planes
for an additional route departing from City Airport that
route also saw a substantial increase in passengers.
Many passengers who depart from City Airport on one
of the newer planes have connecting flights that use
airplanes that are more than 5 years old.
Besides the larger overhead bins, there are several
other differences between the newer and older
airplanes.

DO NOT DISTRIBUTE
Flights offered by Beta Airline’s competitors on these
76 | © TPR Education IP Holdings, LLC. same routes also have higher numbers of passengers
than the competitors’ flights on other routes.
LESSON 2 VERBAL

SCRATCH PAPER
2

DO NOT DISTRIBUTE © TPR Education IP Holdings, LLC. | 77


MANUAL FOR THE GRE

2 INFERENCE QUESTIONS

Arguments Basic Approach for Inference Questions


Step 1: Identify the Question Type
• infer
• must be true
• support
Step 2: Work the Argument
• Read it!
Step 3: Know what the Answer Needs to Do
• makes a statement that must be true based on the
facts provided
Step 4: Use Process of Elimination
• out of scope
• extreme language

Question 1

According to a recent study, fifteen corporations


in the United States that follow a credo of social
responsibility are also very profitable. Because
of their credos, these fifteen corporations give
generously to charity, follow stringent environmental
protection policies, and have vigorous affirmative
action programs.

If the statements in the passage are true, then which


of the following must on the basis of them be true?

Following a credo of social responsibility


helps to make a corporation very
profitable.
It is possible for a corporation that
follows a credo of social responsibility to
be very profitable.
A corporation that gives generously to
charity must be doing so because of its
credo of social responsibility.
Corporations that are very profitable tend
to give generously to charity.
Corporations that have vigorous
affirmative action programs also tend to
follow stringent environmental protection
policies.

78 |
DO NOT DISTRIBUTE
© TPR Education IP Holdings, LLC.
LESSON 2 VERBAL

SCRATCH PAPER
2

DO NOT DISTRIBUTE © TPR Education IP Holdings, LLC. | 79


MANUAL FOR THE GRE

2 ARGUMENTS DRILL

Question 1 of 5 Question 3 of 5
If Intercontinental Airlines wants to attract more If tourists continue to visit Bridges National Park
customers, it should offer a gift to each passenger. at the current rate, the fragile desert environment
Computer Resources has been setting new sales that the park seeks to protect will be irreparably
records by including complimentary software with harmed. The park’s Director has put forth a proposal
every purchase of its latest computer model. to charge visitors to the park a fee based on the
number of hours spent in the park. This fee will
The argument depends upon which of the following induce tourists to limit their time in the park and
assumptions? thus protect the fragile desert environment.

Airline travelers are more likely than train Which of the following, if true, most seriously
travelers to be motivated by gifts. weakens the argument?
Airline travelers rarely consider an airline’s
safety record when making travel plans. Tourists will limit the amount of time that
Airline ticket buyers, like computer buyers, they spend in the park by hiking only the
make decisions based on complimentary shorter trails in the park.
offers. Most tourists will easily be able to pay the
Computer purchasers are more fee even if they do not reduce the number
discriminating than are airplane ticket of hours that they spend in the park.
purchasers. The fee will induce tourists to enter the
Computer purchasers, in general, have park early in the morning before the fee
discretionary income comparable to that of collection station has opened and leave
airplane ticket purchasers. only after the fee collection station has
closed for the night.
Protecting the fragile desert environment is
Question 2 of 5 an important issue for most visitors to the
A recent study revealed that participants who park.
overate also produced high concentrations of the Tourists who do not wish to pay the fee
hormone ghrelin, while patients who did not overeat can visit nearby Chasmlands National Park.
had normal or low concentrations of this hormone.
Clearly, ghrelin is at least partly responsible for
overeating.

The argument assumes which of the following?

Overeating does not affect the production


of ghrelin.
No other hormone affects overeating.
No other hormone is as responsible for
overeating as is ghrelin.
Ghrelin is a more important causative factor
for overeating than willpower.
People with low or normal concentrations
of ghrelin will never overeat.

80 |
DO NOT DISTRIBUTE
© TPR Education IP Holdings, LLC.
LESSON 2 VERBAL

SCRATCH PAPER
2

DO NOT DISTRIBUTE © TPR Education IP Holdings, LLC. | 81


MANUAL FOR THE GRE

Question 4 of 5 Question 5 of 5
2
Excessive government regulation, not the current A combination of anxiety and external pressure
high price of oil, is responsible for the poor industrial leads to nausea. All the auditioners for the new
production in Country A since its new government reality show Wanna Be’s suffer from external
came to power. Country B pays the same price for pressure. Some of the auditioners feel anxiety about
oil, but while industrial output in Country A has been performing well for the producers but others do not
falling, it has been rising in Country B. feel anxious. The producers of Wanna Be’s select
only those contestants who feel anxiety.
Which of the following, if true, would cast the most
doubt on the argument? If the statements in the passage are true, then which
of the following can be concluded?
Agricultural production is also declining in
Country A. The auditions for Wanna Be’s cause more
Whereas Country B must import most performers to feel external pressure than
minerals, Country A exports minerals. do auditions for other shows.
In both countries A and B, there has been Most of the people who audition become
little credit available through the credit contestants on Wanna Be’s.
markets. There is more nausea among auditioners
The industrial goods produced in Country A for Wanna Be’s than among producers of
are different from those that are produced Wanna Be’s.
in Country B. No auditioner who doesn’t feel anxiety has
Country A’s new government increased nausea.
regulation of its industry to promote a All of the auditioners for Wanna Be’s who
cleaner environment. become contestants have nausea.

82 |
DO NOT DISTRIBUTE
© TPR Education IP Holdings, LLC.
LESSON 2 VERBAL

SCRATCH PAPER
2

DO NOT DISTRIBUTE © TPR Education IP Holdings, LLC. | 83


MANUAL FOR THE GRE

SUMMARY
Arguments Basic Approach

Step 1: Identify the Question Type

Step 2: Work the Argument

Step 3: Know what the Answer Needs to Do

Step 4: Use Process of Elimination

Types of Arguments Questions

• Main Conclusion

• Assumption

• Weaken

• Strengthen

• Inference

• ID the Reasoning

• Resolve/Explain

84 |
DO NOT DISTRIBUTE
© TPR Education IP Holdings, LLC.
Lesson 3 Math

DO NOT DISTRIBUTE
MANUAL FOR THE GRE

PERCENTS AND PERCENT CHANGE


3 Question 1

A merchant makes a $10 profit on a jacket that the


merchant sells for $30. What is the merchant’s cost
as a percent of the jacket’s selling price ?

Give your answer to the nearest whole percent.

Question 2

A dress that originally sold for $120 now sells for


$96. The new price is what percent less than the
original price ?

8%

12%

20%

24%
1
33 %
3

Percent change = difference × 100


original

Question 3

Which of the following statements are equivalent to the


percent increase from 3 to 4 ?

Indicate all such statements.

 The percent decrease from 4 to 3.


 The percent decrease from 12 to 8.
 The percent increase from 6 to 8.

86 |
DO NOT DISTRIBUTE
© TPR Education IP Holdings, LLC.
LESSON 3 MATH

SCRATCH PAPER

DO NOT DISTRIBUTE © TPR Education IP Holdings, LLC. | 87


MANUAL FOR THE GRE

RATIOS AND PROPORTIONS


3 Question 1

A can of nuts contains almonds and cashews in a


ratio of 2 to 3. If there are 30 almonds in the can,
how many cashews are in the can?

10
15
20
30
45

Question 2

A can of nuts contains almonds and cashews in the


ratio 2:3. If there are 75 nuts in the can, how many
are cashews?

cashews
TRIGGER

Trigger: _______________________________________________________________

Question 3
The ratio of a to b is 3 to 4.

Quantity A Quantity B
a+1 4
b + 1 5

Quantity A is greater.
Quantity B is greater.
The two quantities are equal.
The relationship cannot be determined from the
information given.

88 |
DO NOT DISTRIBUTE
© TPR Education IP Holdings, LLC.
LESSON 3 MATH

SCRATCH PAPER

RESPONSE
Draw a ratio box on your scratch paper.

DO NOT DISTRIBUTE © TPR Education IP Holdings, LLC. | 89


MANUAL FOR THE GRE

AVERAGES, MEDIANS & MODES


3 Question 1

The average (arithmetic mean) salary of 12 employees


at a certain firm is $35,000. If the average salary of 8
of the employees is $40,000, what is the average
salary of the 4 remaining employees?

$25,000
$27,000
$27,500
$28,000
$30,000
TRIGGER

Trigger: _______________________________________________________________

Question 2

The average (arithmetic mean) score for half of the


students in a class on a certain test is 90. The average
score for another fourth of the students is 80. If the
average score for all of the students is 78, what is the
average score for the remaining students?

52
58
64
70
78

90 |
DO NOT DISTRIBUTE
© TPR Education IP Holdings, LLC.
LESSON 3 MATH

SCRATCH PAPER

RESPONSE
Total
÷ ÷
# of
things
× Average

DO NOT DISTRIBUTE © TPR Education IP Holdings, LLC. | 91


MANUAL FOR THE GRE

Question 3
Hours Spent Text Messaging
3 Hours per teenager Number of teenagers
6 1
8 2
10 3
12 3
14 4

The table above shows the number of hours spent


text messaging by a group of 13 teenagers during
a one-week period. What is the median number of
hours spent text messaging by these teenagers?

9
10
11
12
13

Question 4

List A: 5, 5, 4, 4, 1, 6, 3, x, y

The mode of List A shown above is 5 and the


median is 4. If x > y, and x and y are both integers,
what is the greatest possible value of y ?

92 |
DO NOT DISTRIBUTE
© TPR Education IP Holdings, LLC.
LESSON 3 MATH

SCRATCH PAPER

DO NOT DISTRIBUTE © TPR Education IP Holdings, LLC. | 93


MANUAL FOR THE GRE

AVERAGES, MEDIANS & MODES DRILL


3
Question 1 of 3

On Tuesday, a veterinary clinic sees both dogs and


cats, but no other animals, and there are fewer than 4
dogs seen for every cat seen. The average (arithmetic
mean) charge for a dog visit that day is $65 and $45 for
a cat visit. Which of the following amounts could be the
average charge for all of the patients seen by the clinic
on Tuesday?

Indicate all such amounts.

 $45.00
 $55.00
 $58.33
 $60.00
 $61.00
 $65.00
 $68.00

Question 2 of 3

Set X consists of 5 consecutive positive integers.

Quantity A Quantity B
The sum of the 5 integers 5 times the median of the 5
in set X integers in set X

Quantity A is greater.
Quantity B is greater.
The two quantities are equal.
The relationship cannot be determined from the
information given.

94 |
DO NOT DISTRIBUTE
© TPR Education IP Holdings, LLC.
LESSON 3 MATH

SCRATCH PAPER

DO NOT DISTRIBUTE © TPR Education IP Holdings, LLC. | 95


MANUAL FOR THE GRE

Question 3 of 3

Average Monthy Data Overage for Four Groups of


3 People

Group Number of Average Data


People in Group Overage (GB)
A 40 0.6
B 35 1.3
C 30 2.2
D 20 1.8

Each of the people in the four groups reported their


monthly data overage in gigabytes for a three-month
period. For each of these groups, the table above
shows the number of people in the group and the
average (arithmetic mean) of their data overages for
the three-month period. What is the average data
overage in gigabytes reported by the 125 people in
the four groups combined?

Give your answer to the nearest 0.1.

gigbytes

96 |
DO NOT DISTRIBUTE
© TPR Education IP Holdings, LLC.
LESSON 3 MATH

SCRATCH PAPER

DO NOT DISTRIBUTE © TPR Education IP Holdings, LLC. | 97


MANUAL FOR THE GRE

RATES
3 Question 1

Rob and David live 200 miles apart. Deciding to


have a picnic, they both start driving at 9:00 a.m.,
traveling in a straight line towards each other. Rob
and David drive at average speeds of 30 and 50
miles per hour, respectively. At what time do they
meet for their picnic?

11:30 a.m.
1:00 p.m.
1:30 p.m.
2:30 p.m.
3:40 p.m.
TRIGGER

Trigger: _______________________________________________________________

Question 2

Working at its constant rate, Machine A produces


x donuts in 12 hours. Working at its constant rate,
Machine B produces x donuts in 6 hours.

Quantity A Quantity B
The number of hours The number of hours
required for both required for Machine B
x
machines working to produce donuts
x 3
together to produce
2
donuts

Quantity A is greater.
Quantity B is greater.
The two quantities are equal.
The relationship cannot be determined from the
information given.

98 |
DO NOT DISTRIBUTE
© TPR Education IP Holdings, LLC.
LESSON 3 MATH

SCRATCH PAPER

RESPONSE
Work/Distance
÷ ÷
Time × Rate

DO NOT DISTRIBUTE © TPR Education IP Holdings, LLC. | 99


MANUAL FOR THE GRE

DRILL
3
Question 1 of 10

Joan can do an entire job in 12 hours. Ken can do an


entire job in 6 hours. If Joan and Ken work together
for 3 hours, how many hours will it take Ken to finish
the rest of the job by himself?

1
12

1
42

Question 2 of 10

At a certain factory, each worker either drives to


work or takes the bus. The ratio of workers who
take the bus to work to those who drive to work is
2 : 5. If 120 workers drive to work, how many
workers are there at the factory?

300
240
168
48
24

Question 3 of 10

Erik borrows a total of $6,000 in two separate loans


for a one-year period. At the end of one year, he pays
back the entire amount of each loan plus simple
annual interest at the rate of 4 percent for the first
loan and 6 percent for the second loan. If the interest
paid on each loan is the same amount, how much
total interest did Erik pay on these two loans?

$144
$240
$288
$360
$600

100 |
DO NOT DISTRIBUTE
© TPR Education IP Holdings, LLC.
LESSON 3 MATH

SCRATCH PAPER

DO NOT DISTRIBUTE © TPR Education IP Holdings, LLC. | 101


MANUAL FOR THE GRE

Question 4 of 10

The average (arithmetic mean) age of the members


3 of club K is 22 years, and the average age of the
members of club Q is 29 years. The average age of
the members of both clubs combined is 27 years.

Quantity A Quantity B
The number of The number of
members in club K members in club Q
Quantity A is greater.
Quantity B is greater.
The two quantities are equal.
The relationship cannot be determined from the
information given.

Question 5 of 10

If Dan had increased his average speed by 20 miles per


hour, he would have decreased the time it took him to
drive from his job to a certain restaurant by 25%. What
was Dan’s actual average speed, in miles per hour,
when he drove from his job to the restaurant?

30
40
45
50
60

102 |
DO NOT DISTRIBUTE
© TPR Education IP Holdings, LLC.
LESSON 3 MATH

SCRATCH PAPER

DO NOT DISTRIBUTE © TPR Education IP Holdings, LLC. | 103


MANUAL FOR THE GRE

Question 6 of 10
Depth of Smith Pond
3

Depth of Water (in inches)

May

June

July

August

September

October
The chart above shows the average depth of Smith Pond
for each of the months from May through October. If
each unit on the vertical axis represents one inch and the
average depth of the pond decreased by 20 percent from
July to August, what was the average depth of the pond
in August?

5 inches
20 inches
25 inches
30 inches
40 inches

Question 7 of 10

For a competition, 40 golfers each attempted the same


putt 20 consecutive times. The number of successful
putts was recorded for each golfer and the average
(arithmetic mean) of the 40 numbers of successful
putts per golfer was 10.3. Only one error was made in
recording the 40 numbers, “2” successful putts for one
golfer was incorrectly recorded as “20”.

Quantity A Quantity B
The actual average number 9.85
of successful putts per
golfer

Quantity A is greater.
Quantity B is greater.
The two quantities are equal.
The relationship cannot be determined from the
information given.

104 |
DO NOT DISTRIBUTE
© TPR Education IP Holdings, LLC.
LESSON 3 MATH

SCRATCH PAPER

DO NOT DISTRIBUTE © TPR Education IP Holdings, LLC. | 105


MANUAL FOR THE GRE

Question 8 of 10

A can contains almonds and cashews in the ratio of x to y.


3 If there are a total of z nuts in the can, which of the
following is an expression for the number of almonds?

x+y

z (x + y)

z
x+y

xz
x+y

yz
x+y

Question 9 of 10

If the average (arithmetic mean) of x and y is z,


which of the following is an expression for y in
terms of x and z ?

x+ z

2
x+ z

3

2z − x

z x

3 2
3z − 2x

106 |
DO NOT DISTRIBUTE
© TPR Education IP Holdings, LLC.
LESSON 3 MATH

SCRATCH PAPER

DO NOT DISTRIBUTE © TPR Education IP Holdings, LLC. | 107


MANUAL FOR THE GRE

Question 10 of 10
Average Annual
3 Earthquake Fault
Zones
Frequency of Earthquakes
1986-1993
Zone One x
Zone Two 8.7
Zone Three 5.3
Zone Four 5.7
Zone Five y

In the chart above, if the average (arithmetic mean)


frequency of earthquakes in Zones One, Two,
and Three is 8.0, and the average frequency of
earthquakes in Zones Four and Five is 5.5, then
how much less than the average of the five annual
earthquake frequencies is the mode of the five
annual earthquake frequencies?

1.7
5.3
5.5
7.0
8.0

108 |
DO NOT DISTRIBUTE
© TPR Education IP Holdings, LLC.
LESSON 3 MATH

SCRATCH PAPER

DO NOT DISTRIBUTE © TPR Education IP Holdings, LLC. | 109


MANUAL FOR THE GRE

SUMMARY
3

RESPONSE
TRIGGER

“…ratio…” Draw a ratio box.

RESPONSE
Total
TRIGGER

“…average…” ÷ ÷
# of
things
× Average

RESPONSE
Work/Distance
TRIGGER

“…speed…”
÷ ÷
“…rate…”
Time × Rate

Percent change
difference
Percent change = original × 100

Median
The middle number after putting the numbers in the list in
numerical order.
Mode

The number that appears most frequently in a list of numbers.

110 |
DO NOT DISTRIBUTE
© TPR Education IP Holdings, LLC.
Lesson 3
Verbal

DO NOT DISTRIBUTE
MANUAL FOR THE GRE

READING BASIC APPROACH


3
Reading Basic Approach
Step 1: Work the Passage
• Main Idea
• Structure
• Author’s side
Step 2: Understand the Question
• Subject
• Task
Step 3: Find the information in the passage that addresses the
task of the question.
Step 4: Use Process of Elimination
• Recycled language
• Extreme language
• No such comparison
• Reversals
• Outside knowledge
• Emotional appeals

112 |
DO NOT DISTRIBUTE
© TPR Education IP Holdings, LLC.
LESSON 3 VERBAL

DO NOT DISTRIBUTE
© TPR Education IP Holdings, LLC. | 113
MANUAL FOR THE GRE

MAIN IDEA & PRIMARY PURPOSE QUESTIONS


3 So remarkable and definite are the similarities .
among the Indo-European, or Aryan, languages
that linguists are convinced they all derived from
Line an earlier language spoken by some community in
5 the prehistoric past. While it is known that Latin
began as a rustic dialect in the province of Latium,
no one knows where Proto-Aryan was first spoken.
Some speculate that Proto-Aryan was first used
in Southern Russia, while still others point to the
10 Iranian plateau as a more likely potential cradle.
Though some philologists believe that the Old
Indic and Persian of the Avesta contain the most
archaic features of Aryan found to date, this does
not necessarily fix the habitat of these early Aryan-
15 speaking peoples closer to Asia than to Europe.
Consider Icelandic. Though this language has
strayed far from its birthplace, it preserves many
of the characteristics discarded by those who
remained behind.

Question 1

The primary purpose of the passage as a whole can


best be described as which of the following?

Offering an alternative theory for the most


likely birthplace of Proto-Aryan
Analyzing the role that Icelandic plays in
the evaluation of evidence from the Avesta
for the origin of Proto-Aryan
Countering evidence that the Aryan
languages are all derived from a common
earlier language known as Proto-Aryan
Offering support for the claim that the
birthplace of Proto-Aryan is unknown by
questioning some possible points of origin
Offering support for the contention that the
origins of Proto-Aryan predate writing

114 |
DO NOT DISTRIBUTE
© TPR Education IP Holdings, LLC.
LESSON 3 VERBAL

SCRATCH PAPER

DO NOT DISTRIBUTE © TPR Education IP Holdings, LLC. | 115


MANUAL FOR THE GRE

Theorists are divided about the cause of the


Permian mass extinction. Some hypothesize that
the impact of a massive asteroid caused the
3 Line sudden eradication of most species. However, a
5 look at the carbon-isotopic record suggests that
existing plant communities were decimated and
revived several times. To produce such a pattern
would require a succession of asteroid strikes
thousands of years apart. Other theorists have
10 proposed that volcanic explosions raised the
CO2 levels, leading to intense global warming.
One problem with this theory is that it cannot
explain the mass marine extinctions at the end
of the Permian period. A new theory proposes
15 that the combination of rising concentrations of
toxic hydrogen sulfide in the world’s oceans and
gradual oxygen depletion in the surface waters
caused the extinctions. Fortunately, this theory
is testable. If this theory is true, then oceanic
20 sediments from the Permian period will yield
chemical evidence of a rise in hydrogen sulfide-
consuming bacteria.

Question 1

The primary purpose of the passage is to

consider several hypotheses concerning


the cause of the Permian mass extinctions
discuss the strengths and weaknesses of
the asteroid hypothesis concerning the
cause of the Permian mass extinctions
propose that theories regarding the cause
of the Permian mass extinctions be tested
argue the Permian mass extinctions
could not have been caused by a volcanic
explosion
describe one reason that a rise in hydrogen
sulfide would cause massive marine
extinctions

116 |
DO NOT DISTRIBUTE
© TPR Education IP Holdings, LLC.
LESSON 3 VERBAL

SCRATCH PAPER

DO NOT DISTRIBUTE © TPR Education IP Holdings, LLC. | 117


MANUAL FOR THE GRE

It would be expected that a novel by a figure


as controversially brilliant and difficult as David
Foster Wallace would attract both the adoration
3 Line and ire of literary critics. Furthermore, when a
5 novel addresses the ceaseless consumption of
entertainment that constitutes modern American
culture, as does Wallace’s masterpiece Infinite
Jest (1996), it would be a surprise if a critic didn’t
take a jab at a piece of entertainment implicitly
10 purporting to satirize itself. However, other than
an occasional complaint about self-indulgence,
such slings and arrows were surprisingly absent
from scholarly commentary upon the novel. One
reason for this could be the fact that his novel
15 was simply so shockingly new, both in form and
content, that one couldn’t argue its artistic bona
fides, no matter how metacritical one would like
to be. But what if the truth were a little more
disturbing—that its very complexity prevented
20 anybody, readers and critics alike, from fully
understanding the text to begin with?

Question 1

Which of the following best describes the central


issue with which the passage is concerned?

The degree to which entertainment is the


most representative theme in modern
society
The reasons for the critical response to the
novel Infinite Jest
Whether entertainment deserves a central
place in the modern world’s viewpoint
The recursive nature of all literary art that is
relevant to modern society
The complex relationship between Wallace
and his literary critics

118 |
DO NOT DISTRIBUTE
© TPR Education IP Holdings, LLC.
LESSON 3 VERBAL

SCRATCH PAPER

DO NOT DISTRIBUTE © TPR Education IP Holdings, LLC. | 119


MANUAL FOR THE GRE

PRACTICE
3 Strikes by industrial workers became increasingly Question 2
common during the 1880s, a time when the
American labor movement was in its infancy. It can be inferred that the author of the passage
Line Despite the widespread admiration of the mentions “rising inequality” in the third sentence
5 famous titans of industry, such as Carnegie and primarily to suggest that
Rockefeller, the public had begun to sympathize
with the strikers. Anybody who wished to labor leaders were more in touch with
reconcile the two sides found themselves in the essential issues of the day than were
unenviable position of defending the positions Carnegie and Rockefeller
10 taken by the powerful capitalists, including Carnegie and Rockefeller were hopelessly
explaining rising inequality. out of touch with the common man
In contrast to the course of public opinion, the Carnegie and Rockefeller may have
Haymarket Riot in Chicago provided opponents of discovered that a hardline stance was
labor strikes with powerful evidence that bending the only way to combat those who would
15 to the will of the people would result in violence. potentially disrupt their businesses
At a peaceful rally organized by German immigrant the national discrepancy in income was
labor leaders on May 6, 1886 in Haymarket Square, difficult for anyone to justify
an event intended to protest the killing of several strikers were more likely to press for higher
strikers the day before at the McCormick Reaper wages when there was evidence of their
20 Works, an individual threw a bomb, causing seven exploitation
police officers to be slain in the ensuing mayhem.
While the bomber was never identified, eight
anarchist strikers were convicted of the crime, and Question 3
many historians point to this event as the catalyst
25 that set off a wave of xenophobia and set back It can be inferred that the author of the passage
labor for a generation. Although evidence of these would most likely agree with which of the following
negative effects can clearly be identified in the claims about the organizers of the Haymarket
historical record, a growing group of historians protest?
have correctly pointed to the popular identification
30 of the executed defendants as martyrs as the They were justified in organizing the
spark which, rather than slowing the labor protest, given that several strikers had
movement, actually carried it forward into the next already been killed the previous day.
century. They were mistaken in their belief that,
given the events of the day before, the
protest would proceed peacefully.
Question 1 They could have taken extra steps to
ensure the safety of everyone present,
The passage is primarily concerned with discussing
including police officers.
which of the following?
They subtly implied, through the decision
The relative merits of two points of view to hold the rally so soon after the deaths of
regarding a controversy the strikers, that the use of violence would
The proper interpretation of an event in the not be discouraged.
history of workers’ rights They held a rally because they expected
The misunderstandings that led to the end that it would lead to positive results for
of a historical trend in the 1880s their movement.
The role of protests in reconciling the
differences between capitalists and
workers in the 1880s
The difficulty that immigrants experienced
in organizing the labor movement in the
1880s

120 |
DO NOT DISTRIBUTE
© TPR Education IP Holdings, LLC.
LESSON 3 VERBAL

SCRATCH PAPER

DO NOT DISTRIBUTE © TPR Education IP Holdings, LLC. | 121


MANUAL FOR THE GRE

A handful of investigators assert that play serves Question 2


no function in the development of predatory
skills, such as stalking and killing prey. These According the author of the passage, which of the
3 Line investigators report that animals raised with a following notions about survival skills and play is
5 dearth of sibling wrestling nevertheless display valid?
normal levels of efficiency at catching their dinner,
and they reason that if play were essential to There is at least one predatory skill that
hunting and fighting, then these animals would isn’t directly developed through play.
exhibit diminished talents. However, these Play is more valuable to the development
10 scientists admit that the prefrontal cortex of of predatory skills than to the development
a “rough and tumble” deprived pup has never of other survival skills.
been monitored in the context of an extreme Sibling wrestling is more essential to the
survival environment; similarly, this particular development of predatory skills than are
group of neurons has never been the focus in other forms of play.
15 field experiments on non-threatening tasks that There is a wide range in the quantity of play
require creative problem solving. Unfortunately, animals need in order to develop predatory
these experiments would only elucidate the skills.
developmental effects of sibling wrestling rather
It is probable that the development of
than the effects of all forms of play.
survival skills occurs more during an
20 These investigators likewise declare that animal’s adulthood than during its youth.
enhanced survival skills are probably due to trial
and error, rather than the neural pathways formed
by play. But new research into positive social Question 3
interaction after a childhood of play—such as
25 the experiment that proved a causal connection The ground squirrel experiment mentioned in the
between ground squirrels that sparred as babies second paragraph is valuable because it
and their development into better mothers—
demonstrates the falsity of this declaration. elucidates the reasons why better mothers
Obviously, animals assimilate survival lessons develop from sparring babies
30 throughout their development, and not all of them recognizes a certain role that play assumes
are directly linked to mock fighting. Yet animals in the development of survival skills
exposed to early play exhibit slightly better examines the characteristics of better
coordination and less cognitive-impairing stress mothering that develop from babies
in adulthood; therefore, the advantages of play in sparring
35 evolved predatory skills should be recognized. implies that survival lessons come from a
myriad of sources
Question 1 describes how trial and error enhance the
development of survival skills.
The primary purpose of the passage is to

offer proof that backs a particular assertion


about sibling wrestling and predatory skills
articulate ways that multiple components
belie the correlation between play and
survival skills
argue a certain perspective concerning
play’s role in the development of survival
skills
recapitulate the conventional hypothesis
about play and survival skills
emphasize the value of developing
predatory skills

122 |
DO NOT DISTRIBUTE
© TPR Education IP Holdings, LLC.
LESSON 3 VERBAL

SCRATCH PAPER

DO NOT DISTRIBUTE © TPR Education IP Holdings, LLC. | 123


MANUAL FOR THE GRE

Art historians describe an underlying split between Question 2


Modernism—which emphasized the emotional
transcendence of the individual genius—and Which of the following does the author of the
3 Line Postmodernism—which valued conceptual art passage state can result from the “divergent
5 (and the dissipation of the self) over its physical classifications” mentioned in the passage?
representation. Theoretically, the Postmodern
conglomeration of highbrow and lowbrow subject Dependence on a scholarly paradigm that
matter coincided with the revolt against repressive is comparable to other historical realities
institutions in the late 1960s, merging popular The application of contemporary
10 culture with high culture. Accordingly, there is motivations to artists’ work from past
what some scholars call a historical divergence periods
between the two art periods, and so the very idea Inadequate grouping of art styles by
of a catalogue of Postmodernism before the mid- ignoring some of the inspiration behind
20th Century seems absurd. their creation
15 To discard the notion that the everyman’s Inadequate use of recent modifications to
pop culture positively influenced artists before the classification of high culture
Warhol’s era is not to ignore the Dadaists, artists Inadequate notation of the differences
who spliced commercial themes into their between the Modern and Postmodern eras
paintings, sculpture, and performances long
20 before the Factory. Indeed, some historians refer
to this aspect of their work, albeit as secondary Question 3
to those artists’ surrealist agendas. However,
the divergent classifications can be problematic, Which of the following best matches the structure of
especially for overlapping periods. Artists such as the passage?
25 Chuck Close employed Modernist methods on a
minute scale to produce figurative, non-Modernist Two schools of thought are examined, and
works as late as the 21st Century. Minimalism support for one of them is offered.
and Pop Art are often considered Modern Art A method of interpretation is displayed,
movements, despite the inspiration they drew and a boundary to that interpretation is
30 from what we now consider Postmodern ideals. articulated.
While Postmodernists often place social justice A theory is suggested, its purported
and irreverence before the elevation of the ego, benefits are proven to be true, and its
the origins of both movements can be attributed pitfalls are proven nonexistent.
to a discontent with the hierarchy of taste A theoretical discrepancy is revealed, two
35 and its effects on society at large. Divergent perspectives are defined to abolish it, and
categorization forces a great variety of works into both are shown to be unsatisfactory.
castes that, by definition, they resist.
An obstacle is observed, two routes around
it are offered, and a methodology unifying
Question 1 aspects of both is advised.

The primary purpose of the passage is to

disclaim the view that pop culture played a


sporadic role in Modernist art
demonstrate how a certain categorization
is more appropriate to the classification of
art
indicate the necessity of a revised, broader
definition of popular culture
outline the progression of a thesis
concerning the evolution of Postmodern art
highlight the foundation of, and the
limitations to, a method for separating two
art movements

124 |
DO NOT DISTRIBUTE
© TPR Education IP Holdings, LLC.
LESSON 3 VERBAL

SCRATCH PAPER

DO NOT DISTRIBUTE © TPR Education IP Holdings, LLC. | 125


MANUAL FOR THE GRE

The Big Bang Theory—which states that the Question 2


universe exploded into existence out of nothing
nearly 14 billion years ago—does not explain The Scale Symmetry Theory as described in the
3 Line why the universe contains so little homogeneity, passage answers all of the following questions
5 compared to what our computations indicate EXCEPT:
should occur during such an accelerated
expansion of gasses. According to the Scale What creates the lack of homogeneity in
Symmetry Theory, the universe—as we know it— the universe?
may lack a consistent, external scale to describe What characteristics can determine the
10 mass and length. Theoretical models of scale- size and shape of particles?
less particles interacting describe characteristics What generated the motion needed to
such as electric charge or antimatter that cause particles to interact?
determine sizes and shapes based on inter- Why don’t scientists find uniformity of
particle interactions. The lumps of space-less shape and distance across the visible
15 structures between celestial gravitational fields universe?
that we find today may not be quantifiable by
How quantifiable are the space-less
standard yardsticks. Homogeneity, or lack thereof,
structures between gravitational fields?
would be better imagined as a misinterpretation
of measurement itself.
Question 3
For the following question, consider each of the Which of the following best describes the structure
choices separately and select all that apply. of the passage?

The process by which one hypothesis is


Question 1 transformed into a wholly different one is
According to the passage, the Big Bang Theory described
posits that Two expositions are given, both of which
are shown to be deficient
 the universe was generated from a void A theory is articulated, then proof of its
 came from nothing a finite number of years inadequacy is reviewed
ago Parallels and divergences between two
 there is very little homogeneity throughout suppositions are examined
the universe A problematic point with a theory is
mentioned and a possible theoretical
solution described

126 |
DO NOT DISTRIBUTE
© TPR Education IP Holdings, LLC.
LESSON 3 VERBAL

SCRATCH PAPER

DO NOT DISTRIBUTE © TPR Education IP Holdings, LLC. | 127


MANUAL FOR THE GRE

SUMMARY
Reading Basic Approach for Main Idea & Primary Purpose
Questions

Step 1: Work the Passage

• Main Idea

• Structure

• Author’s side

Step 2: Understand the Question

• Subject

• Task

Step 3: Find the information in the passage that


addresses the task of the question.

Step 4: Use Process of Elimination

• Recycled language

• Extreme language

• No such comparison

• Reversals

• Outside knowledge

• Emotional appeals

128 |
DO NOT DISTRIBUTE
© TPR Education IP Holdings, LLC.
Lesson 4
Math

DO NOT DISTRIBUTE
MANUAL FOR THE GRE

BASIC APPROACH
Question 1

4 B

A C
D

In the figure above, the area of triangle ABC is 20.


If AD = BD, then what is the length of DC ?

2
2 2
4 2
6
10

Geometry Basic Approach


1. Draw the figure on your scratch paper.
2. Label any information from the problem on the figure.
3. Write down any formulas you need.

130 |
DO NOT DISTRIBUTE
© TPR Education IP Holdings, LLC.
LESSON 4 MATH

SCRATCH PAPER

DO NOT DISTRIBUTE © TPR


©Education
The Princeton
IP Holdings,
Review, LLC.
Inc. | 131
MANUAL FOR THE GRE

RULES REVIEW: LINES & ANGLES


Question 1

x° y°

In the figure above, if 3x = 2(x + y), what is the


value of y ?

y=

Question 2

B E

C

A D

In the figure above, C is the midpoint of both


segment AE and segment BD. If AE = BD and
1
∠ECD = ∠BCE, then what is the value of x ?
2

30
45
50
60
75

Question 3
k
x° y°

140° 110° m

In the figure above, if line k is parallel to line m, what


is the value of x + y ?

40
70
110
150

DO NOT DISTRIBUTE
250
132 | © TPR Education IP Holdings, LLC.
LESSON 4 MATH

SCRATCH PAPER

DO NOT DISTRIBUTE © TPR


©Education
The Princeton
IP Holdings,
Review, LLC.
Inc. | 133
MANUAL FOR THE GRE

RULES REVIEW: TRIANGLES


Question 1

4 If the three angles in a triangle have degree


x+y+z
measures x, y, and z, what is the value of ?
15
11
12
13
14
15

Question 2

10
6+y

8−y
Quantity A Quantity B
90 x

Quantity A is greater.
Quantity B is greater.
The two quantities are equal.
The relationship cannot be determined from the
information given.

134 |
DO NOT DISTRIBUTE
© TPR Education IP Holdings, LLC.
LESSON 4 MATH

SCRATCH PAPER

DO NOT DISTRIBUTE © TPR


©Education
The Princeton
IP Holdings,
Review, LLC.
Inc. | 135
MANUAL FOR THE GRE

Question 3
B
105°

4
30°
A C

If the length of BC is 6 2, then what is the area of


triangle ABC ?

18
18 3
36
18 + 18 3
36 + 36 3

Question 4

If the lengths of two sides of a triangle are 6 and 10,


respectively, which of the following could be the
perimeter of the triangle?

Indicate all such perimeters.

 20
 24
 27
 33

136 |
DO NOT DISTRIBUTE
© TPR Education IP Holdings, LLC.
LESSON 4 MATH

SCRATCH PAPER

DO NOT DISTRIBUTE © TPR


©Education
The Princeton
IP Holdings,
Review, LLC.
Inc. | 137
MANUAL FOR THE GRE

RULES REVIEW: CIRCLES


Question 1

4 1
The radius of circle A is r and the radius of circle B
2
1
is r. What is the ratio of the area of circle A to the
6
area of circle B ?

1 to 36
1 to 9
1 to 3
3 to 1
9 to 1

Question 2
A
B

In the circle shown above, chord AC has a length


of 6. If the length of arc ABC is equal to one-sixth
of the circumference of the circle, what is the area
of the circle?


12π
24π
36π
48π

138 |
DO NOT DISTRIBUTE
© TPR Education IP Holdings, LLC.
LESSON 4 MATH

SCRATCH PAPER

DO NOT DISTRIBUTE © TPR


©Education
The Princeton
IP Holdings,
Review, LLC.
Inc. | 139
MANUAL FOR THE GRE

Question 3
T

4 S

In the figure above, triangle STU is inscribed in a


circle. The measure of angle STU is greater than 85°
and less than 95°, and the area of the circle is 36π.

Quantity A Quantity B
The length of line 12
segment SU

Quantity A is greater.
Quantity B is greater.
The two quantities are equal.
The relationship cannot be determined from the
information given.

Question 4

If a circle with a radius of 6 is graphed in the


rectangular coordinate system such that the center
of the circle is at the origin, which of the following
points would lie outside the circle?

Indicate all such points.

 (–5, –1)
 (–3, –5)
 (–3, 4)
 (4, 3)
 (4, 5)
 (5, 2)

140 |
DO NOT DISTRIBUTE
© TPR Education IP Holdings, LLC.
LESSON 4 MATH

SCRATCH PAPER

DO NOT DISTRIBUTE © TPR Education IP Holdings, LLC. | 141


MANUAL FOR THE GRE

RULES REVIEW: QUADRILATERALS


Question 1

4 q°


In the figure above, if n = 60, what is the value of


p+q+r?

180
240
300
420
It cannot be determined from the information given.

Question 2

The diagonals of square T intersect at the point

(−3, −3) in the xy-plane. One of the vertices of


11 11
square T is at the point − , − .
2 2

Quantity A Quantity B
25 The area of T

Quantity A is greater.
Quantity B is greater.
The two quantities are equal.
The relationship cannot be determined from the
information given.

Question 3

The perimeter of a rectangular room is 64 feet. The


length of the room is 60 percent of its width. What
is the area of the room, in square feet?

32
60
240
480

DO NOT DISTRIBUTE
960
142 | © TPR Education IP Holdings, LLC.
LESSON 4 MATH

SCRATCH PAPER

DO NOT DISTRIBUTE © TPR Education IP Holdings, LLC. | 143


MANUAL FOR THE GRE

Question 4

What is the length of a diagonal of a rectangle that


has area 48 and perimeter 28 ?

Question 5
B E C

A D

In the parallelogram ABCD above, AD = 12. If the


3
area of ∆ABE is the area of parallelogram ABCD,
8
then EC =

3

2
2
8

3

144 |
DO NOT DISTRIBUTE
© TPR Education IP Holdings, LLC.
LESSON 4 MATH

SCRATCH PAPER

DO NOT DISTRIBUTE © TPR Education IP Holdings, LLC. | 145


MANUAL FOR THE GRE

RULES REVIEW: POLYGONS


Question 1

4 Quantity A Quantity B
The length of a side of The length of a side of
a regular hexagon with a regular octagon with
a perimeter of 19.5 a perimeter of 26

Quantity A is greater.
Quantity B is greater.
The two quantities are equal.
The relationship cannot be determined from the
information given.

Question 2



For the pentagon shown above, y = 80.

Quantity A Quantity B
x 108

Quantity A is greater.
Quantity B is greater.
The two quantities are equal.
The relationship cannot be determined from the
information given.

146 |
DO NOT DISTRIBUTE
© TPR Education IP Holdings, LLC.
LESSON 4 MATH

SCRATCH PAPER

DO NOT DISTRIBUTE © TPR Education IP Holdings, LLC. | 147


MANUAL FOR THE GRE

RULES REVIEW: SOLID GEOMETRY


Question 1

4 The volume, V, of a right circular cylinder is V =p r2h,


where r is the radius of the base and h is the height
of the cylinder. If the circumference of the base of a
right circular cylinder is 8p and its height is 4, what
is the volume of the cylinder?

4
8
16p
64p
256p

Question 2

Cube A has an edge of length 10. The length and


width of rectangular solid B are 20 percent greater
and 10 percent less, respectively, than the edge
of cube A. The height of rectangular solid B is
30 percent greater than the edge of cube A. The
volume of rectangular solid B is what percent
greater than the volume of cube A ?

40.4%
50.4%
60.0%
71.6%
95.0%

Question 3

Rectangular solid C has dimensions 6 inches by 8


inches by 10 inches.

Quantity A Quantity B
The greatest distance 10 2
between any two vertices
of rectangular solid C

Quantity A is greater.
Quantity B is greater.
The two quantities are equal.
The relationship cannot be determined from the
information given.

148 |
DO NOT DISTRIBUTE
© TPR Education IP Holdings, LLC.
LESSON 4 MATH

SCRATCH PAPER

DO NOT DISTRIBUTE © TPR Education IP Holdings, LLC. | 149


MANUAL FOR THE GRE

GEOMETRY PLUGGING IN
Question 1

4 The area of circle A is twice the area of circle B, and


the diameter of circle A is x. In terms of x, what is
the area of circle B ?

π x2

2

π x2

4

π x2

8

πx

2

πx

4

Question 2

Quantity A Quantity B
The area of a square The area of a rectangular
region with perimeter region with perimeter 20
20

Quantity A is greater.
Quantity B is greater.
The two quantities are equal.
The relationship cannot be determined from the
information given.

150 |
DO NOT DISTRIBUTE
© TPR Education IP Holdings, LLC.
LESSON 4 MATH

SCRATCH PAPER

DO NOT DISTRIBUTE © TPR Education IP Holdings, LLC. | 151


MANUAL FOR THE GRE

OVERLAPPING FIGURES
Question 1

4
B C

A D

In the figure above, rectangle ABCD has length 6


and width 8 and is inscribed in the circle as shown.
What is the area of the circle?

10π
20π
25π
64π
100π

Question 2

B C

A D

AD || BC

Quantity A Quantity B

The area of triangle The area of triangle


ABC BCD

Quantity A is greater.
Quantity B is greater.
The two quantities are equal.
The relationship cannot be determined from the
information given.

152 |
DO NOT DISTRIBUTE
© TPR Education IP Holdings, LLC.
LESSON 4 MATH

SCRATCH PAPER

DO NOT DISTRIBUTE © TPR Education IP Holdings, LLC. | 153


MANUAL FOR THE GRE

SHADED REGIONS AND STRANGE-LOOKING FIGURES


Question 1
X Y
4

W Z

In the figure shown above, side YZ of square WXYZ


has a length of 2. If W is the center of the circle,
then what is the area of the shaded region?

4-π
8 - 2π
16 - 4π
4 + 2π
8+π

Question 2
G
10
F

10
8

E 6 H

What is the area of quadrilateral EFGH shown above?

24 + 25 3
24 + 50 3
48 + 50 3
72
It cannot be determined from the
information given.

154 |
DO NOT DISTRIBUTE
© TPR Education IP Holdings, LLC.
LESSON 4 MATH

SCRATCH PAPER

DO NOT DISTRIBUTE © TPR Education IP Holdings, LLC. | 155


MANUAL FOR THE GRE

GEOMETRY DRILL
Question 1 of 10

4
Q

x° 55°
P R

PR = QR

Quantity A Quantity B
120 x

Quantity A is greater.
Quantity B is greater.
The two quantities are equal.
The relationship cannot be determined from the
information given.

Question 2 of 10

A company produces flags in the shape of a right


triangle with legs of length 5 feet and 12 feet. The
three sides of each flag are finished with reflective
trim. If the company has a total of 8,250 feet of
reflective trim and there is no waste of material in
the application of the reflective trim, what is the
greatest possible number of flags that the company
can make?

137
275
284
375
485

156 |
DO NOT DISTRIBUTE
© TPR Education IP Holdings, LLC.
LESSON 4 MATH

SCRATCH PAPER

DO NOT DISTRIBUTE © TPR Education IP Holdings, LLC. | 157


MANUAL FOR THE GRE

Question 3 of 10

A flat, rectangular horse corral has an area of


1,500 square feet and is bordered on three sides
by a fence and on the fourth side by a barn. If the
4 entire length of the fence is 110 feet, which of the
following could be the length, in feet, of one side of
the corral?

Indicate all such lengths.

 25
 30
 35
 40
 45
 50
 55
 60

Question 4 of 10

A C
B

In the figure above, three circles with their centers


on line segment AC are tangent at points A, B, and
C. If point B lies on line segment AC, what is the
ratio of the circumference of the largest circle to
the sum of the circumferences of the two smaller
circles?

1 to 3
1 to 2
1 to 1
2 to 1
3 to 1

158 |
DO NOT DISTRIBUTE
© TPR Education IP Holdings, LLC.
LESSON 4 MATH

SCRATCH PAPER

DO NOT DISTRIBUTE © TPR Education IP Holdings, LLC. | 159


MANUAL FOR THE GRE

Question 5 of 10
S T

4 8
40°
R 10 U

RSTU is a parallelogram.

Quantity A Quantity B
80 The area of RSTU

Quantity A is greater.
Quantity B is greater.
The two quantities are equal.
The relationship cannot be determined from the
information given.

Question 6 of 10
6

5
11 2
10

22

What is the area of the region above?

160 |
DO NOT DISTRIBUTE
© TPR Education IP Holdings, LLC.
LESSON 4 MATH

SCRATCH PAPER

DO NOT DISTRIBUTE © TPR Education IP Holdings, LLC. | 161


MANUAL FOR THE GRE

Question 7 of 10
C

4 B D

A E

In the figure above, AC = CE = AE = 8. If B and D


are the midpoints of AC and CE, respectively, what
is the area of the shaded region?

12 3

16 3

24 3

64

Question 8 of 10

G
H

K
I J

In the rectangular solid depicted above, HI = 5,


IJ = 12, and JK = 3.

Quantity A Quantity B
The area of rectangle HGKJ 39

Quantity A is greater.
Quantity B is greater.
The two quantities are equal.
The relationship cannot be determined from the
information given.

162 |
DO NOT DISTRIBUTE
© TPR Education IP Holdings, LLC.
LESSON 4 MATH

SCRATCH PAPER

DO NOT DISTRIBUTE © TPR Education IP Holdings, LLC. | 163


MANUAL FOR THE GRE

Question 9 of 10

N M
x l1

4
40
65 l2

In the figure above, l1 || l2. What is the value of x ?

60
75
105
115
140

Question 10 of 10
B
x

A C

In the figure above, if A and C are the centers of the


two circles, then what is the value of x ?

30
40
45
60
90

164 |
DO NOT DISTRIBUTE
© TPR Education IP Holdings, LLC.
LESSON 4 MATH

SCRATCH PAPER

DO NOT DISTRIBUTE © TPR Education IP Holdings, LLC. | 165


MANUAL FOR THE GRE

SUMMARY
Lines and Angles

• The sum of the degree measures of two adjacent


angles on a line is 180°.

• Opposite (or vertical) angles are equal.

• When two parallel lines are intersected by a third


line, big and small angles are created. The big angles
are equal and the small angles are equal.

Triangles

• The sum of the degree measures of the 3 angles in a


triangle is 180°.
1
• The formula for the area of a triangle is A = bh.
2
• The side lengths of a right triangle satisfy the
Pythagorean theorem, a2 + b2 = c 2.

• Commonly tested Pythagorean triples include


3-4-5, 6-8-10, and 5-12-13.

• The ratio of the sides in a 45°-45°-90° triangle is


x: x: x 2 .

• The ratio of the sides in a 30°-60°-90° triangle is


x: x 3 : 2x.

• The length of the third side of a triangle is less


than the sum and greater than the difference of the
lengths of the other two sides.

166 |
DO NOT DISTRIBUTE
© TPR Education IP Holdings, LLC.
LESSON 4 MATH

Circles

• The formula for the circumference of a circle is


C = 2πr = πd.

• The formula for the area of a circle is A = πr 2.


central angle sector arc
• = =
360 area circumference

• The diameter is the longest chord in a circle.

Quadrilaterals

• The sum of the degree measures of the 4 angles in a


quadrilateral is 360°.

• The formula for the area of a square is A = s2 and the


formula for the perimeter of a square is P = 4s.

• The formula for the area of a rectangle is A = lw


and the formula for the perimeter of a rectangle is
P = 2l + 2w.

• The formula for the area of a parallelogram is A = bh.

Polygons

• Regular polygons have equal sides and equal angles.

• The sum of the degree measures of the angles in an


n-sided polygon is given by the formula 180(n − 2).

Solid Geometry

• The formula for the volume of a cube is V = s3 and the


formula for the surface area of a cube is SA = 6s2.

• The formula for the volume of a rectangular solid is


V = lwh.

• To find the greatest distance between any two vertices


of a rectangular solid, use the Super Pythagorean
theorem, d 2 = a2 + b2 + c 2.

• The formula of the volume of a right circular cylinder


is V = πr 2h.

DO NOT DISTRIBUTE © TPR Education IP Holdings, LLC. | 167


DO NOT DISTRIBUTE
Lesson 4
Verbal

DO NOT DISTRIBUTE
MANUAL FOR THE GRE

READING BASIC APPROACH

4 Reading Basic Approach


Step 1: Work the Passage
• Main Idea
• Structure
• Author’s side
Step 2: Understand the Question
• Subject
• Task
Step 3: Find the information in the passage that addresses the
task of the question.
Step 4: Use Process of Elimination
• Recycled language
• Extreme language
• No such comparison
• Reversals
• Outside knowledge
• Emotional appeals

170
DO NOT DISTRIBUTE
| © TPR Education IP Holdings, LLC.
LESSON 4 VERBAL

SCRATCH PAPER

DO NOT DISTRIBUTE © TPR Education IP Holdings, LLC. | 171


MANUAL FOR THE GRE

VOCAB IN CONTEXT QUESTIONS


So remarkable and definite are the similarities
among the Indo-European, or Aryan, languages
that linguists are convinced they all derived from
4 Line an earlier language spoken by some community in
5 the prehistoric past. While it is known that Latin
began as a rustic dialect in the province of Latium,
no one knows where Proto-Aryan was first spoken.
Some speculate that Proto-Aryan was first used
in Southern Russia, while still others point to the
10 Iranian plateau as a more likely potential cradle.
Though some philologists believe that the Old
Indic and Persian of the Avesta contain the most
archaic features of Aryan found to date, this does
not necessarily fix the habitat of these early Aryan-
15 speaking peoples closer to Asia than to Europe.
Consider Icelandic. Though this language has
strayed far from its birthplace, it preserves many
of the characteristics discarded by those who
remained behind.

Question 1

In the context in which it appears, “cradle”


(highlighted) most nearly means

crib
rocker
pedigree
birthplace
summit

172
DO NOT DISTRIBUTE
| © TPR Education IP Holdings, LLC.
LESSON 4 VERBAL

SCRATCH PAPER

DO NOT DISTRIBUTE © TPR Education IP Holdings, LLC. | 173


MANUAL FOR THE GRE

RETRIEVAL, INFERENCE & SPECIFIC PURPOSE QUESTIONS


So remarkable and definite are the similarities Question 2
among the Indo-European, or Aryan, languages
that linguists are convinced they all derived from The author mentions Icelandic in order to
4 Line an earlier language spoken by some community in
provide support for the contention that
5 the prehistoric past. While it is known that Latin
early Aryan-speaking peoples did not
began as a rustic dialect in the province of Latium,
necessarily live in Asia
no one knows where Proto-Aryan was first spoken.
Some speculate that Proto-Aryan was first used provide another example of the archaic
in Southern Russia, while still others point to the features found in the Avesta
10 Iranian plateau as a more likely potential cradle. provide evidence for the inception of
Though some philologists believe that the Old Proto-Aryan
Indic and Persian of the Avesta contain the most counter the speculation that
archaic features of Aryan found to date, this does Proto-Aryan was first spoken in Russia
not necessarily fix the habitat of these early Aryan- provide an alternative explanation for the
15 speaking peoples closer to Asia than to Europe. similarities found in early languages
Consider Icelandic. Though this language has
strayed far from its birthplace, it preserves many
of the characteristics discarded by those who
remained behind. Common Wrong Answer Types
1. Recycled Language
• Words and phrases verbatim from
Question 1 the passage
• Words and phrases that come from
Based on the passage, which of the following can the wrong place in the passage
be inferred about languages? • Answers that make you say “I
remember reading that!”
The location in which a language is spoken
may shift over time.
2. Extreme Language
Every modern language derives from • Answers that use strong words
Proto-Aryan. such as must, always, never
The characteristics of a language are • Answers that make claims stronger
invariable. than those in the passage
The Avesta was written to preserve the 3. No Such Comparison
most archaic features of Proto-Aryan. • Answers that use comparison
Latin and Old Indic share definite words such as better, more than,
similarities. less than
• The items compared are discussed
in the passage but not compared.
4. Reversal
• Answers that state the opposite of
the main idea of the passage
• Answers that state the opposite of
a fact in the passage
5. Outside Knowledge/Emotional Appeal
• Answers that are true but not
discussed in the passage (outside
knowledge)
• Answers that state positions
that you believe but that are
not discussed in the passage
(emotional appeal)
174
DO NOT DISTRIBUTE
| © TPR Education IP Holdings, LLC.
LESSON 4 VERBAL

SCRATCH PAPER

DO NOT DISTRIBUTE © TPR Education IP Holdings, LLC. | 175


MANUAL FOR THE GRE

Theorists are divided about the cause of the Question 2


Permian mass extinction. Some hypothesize that
the impact of a massive asteroid caused the Which of the following best describes the function
Line sudden eradication of most species. However, a of the fourth sentence (“To produce…apart”)?
5 look at the carbon-isotopic record suggests that
It offers support for the theory that volcanic
4 existing plant communities were decimated and
eruptions caused the Permian mass
revived several times. To produce such a pattern
would require a succession of asteroid strikes extinction.
thousands of years apart. Other theorists have It elaborates on the claim that rising levels
10 proposed that volcanic explosions raised the of hydrogen sulfide were responsible for
CO2 levels, leading to intense global warming. the extinctions.
One problem with this theory is that it cannot It provides a reason to doubt that an
explain the mass marine extinctions at the end asteroid strike led to the extinctions during
of the Permian period. A new theory proposes the Permian period.
15 that the combination of rising concentrations of It describes a key feature of the volcanic
toxic hydrogen sulfide in the world’s oceans and eruptions theory.
gradual oxygen depletion in the surface waters
It details a method whereby the asteroid
caused the extinctions. Fortunately, this theory
strike theory can be tested.
is testable. If this theory is true, then oceanic
20 sediments from the Permian period will yield
chemical evidence of a rise in hydrogen sulfide-
consuming bacteria.

Question 1 Down to 2?
According to the passage, which of the following is
a potential obstacle to the acceptance of the theory 1. Ask “Which answer uses more
that volcanic eruptions caused the extinctions during words or phrases from the passage?”
the Permian period? Eliminate that answer and choose the
other!
There was a large scale die-off of sea- 2. Ask “Which answer uses the stronger
dwelling species before the Permian period
language or makes the stronger
drew to a close.
claim?” Eliminate that answer and
Many plant species experienced cycles of choose the other!
near extinction and recovery.
3. Ask “Which answer makes a
Increased CO2 levels led to global comparison?” Eliminate that answer
increases in temperature.
and choose the other!
The hydrogen sulfide theory can be tested
by examining the fossil record.
Still struggling? Just mark it, move on, and
Earth experienced a series of meteor come back to it after two questions.
strikes that destroyed plant life.

176
DO NOT DISTRIBUTE
| © TPR Education IP Holdings, LLC.
LESSON 4 VERBAL

SCRATCH PAPER

DO NOT DISTRIBUTE © TPR Education IP Holdings, LLC. | 177


MANUAL FOR THE GRE

It would be expected that a novel by a figure Question 2


as controversially brilliant and difficult as David
Foster Wallace would attract both the adoration In the context of the passage, what is the function of
Line and ire of literary critics. Furthermore, when a the last sentence?
5 novel addresses the ceaseless consumption of
It suggests the author’s agreement with
4 entertainment that constitutes modern American
the idea that the newness of Infinte Jest
culture, as does Wallace’s masterpiece Infinite
Jest (1996), it would be a surprise if a critic didn’t was the reason for the subdued tone of the
take a jab at a piece of entertainment implicitly academic commentary.
10 purporting to satirize itself. However, other than It presents an alternative explanation for an
an occasional complaint about self-indulgence, observation made earlier in the passage.
such slings and arrows were surprisingly absent It poses a question that the academic
from scholarly commentary upon the novel. One commentators of Infinite Jest failed to ask
reason for this could be the fact that his novel when they analyzed the novel.
15 was simply so shockingly new, both in form and It criticizes reader and critics for their
content, that one couldn’t argue its artistic bona failure to understand Infinite Jest.
fides, no matter how metacritical one would like
It provides a reason to doubt the view that
to be. But what if the truth were a little more
Wallace was a brilliant novelist.
disturbing—that its very complexity prevented
20 anybody, readers and critics alike, from fully
understanding the text to begin with?

Question 1 If you can’t put your finger on a single


word, phrase, or sentence that proves
Which of the following can be inferred from the
passage regarding Wallace’s Infinite Jest?
your answer choice, you cannot pick it.
It’s wrong.
Among Wallace’s novels, Infinite Jest
is unusual in that it received so much
attention from critics.
In the period when Infinite Jest was
published, there was still a strong
distinction between literary critics and
academic scholars.
The enormous attention that was paid to
Infinite Jest can be attributed to the fact
that it was Wallace’s most entertaining
novel.
Infinite Jest was better researched and
more expertly written than Wallace’s other
novels.
Despite its subject matter, Infinite Jest
is written in a manner that defies easy
comprehension.

178
DO NOT DISTRIBUTE
| © TPR Education IP Holdings, LLC.
LESSON 4 VERBAL

SCRATCH PAPER

DO NOT DISTRIBUTE © TPR Education IP Holdings, LLC. | 179


MANUAL FOR THE GRE

PRACTICE
In the early twentieth century, the idea that the Question 2
architecture of private residences should consist
of a series of divided rooms and natural darkness Which of the following can be inferred from the
4 Line was replaced by the notion that a private home passage about the Prairie School?
5 could consist of a more relaxed floor plan and be
It was founded by Frank Lloyd Wright.
flooded with natural light. One important figure
It was an outgrowth of the English Arts and
to emerge during this period, though he trained
Crafts movement.
as an engineer rather than as an architect, was
Frank Lloyd Wright (1867-1959). It describes a more or less unified design
aesthetic that a group of architects
10 His designs, which are now viewed as part of followed at the time.
the Prairie School of architecture, were inspired It was the first original American movement
by long walks that he took as a boy in the of architectural design.
Wisconsin countryside and reflect an intense
It was largely a regional movement with
interest in organic integration of place with
little effect outside of the Upper Midwest.
15 structure. His Robie House, in particular, was
cited as an influence in Europe; it also shared
many qualities with the designs of John Ruskin Question 3
and with the English Arts and Crafts movement
of that period, including an aversion to mass The passage suggests that Wright would most likely
20 production and the embrace of handcrafting. have strongly objected to which of the following?
Due to Wright’s innovations, the neoclassical
A modern design that failed to follow the
Greek Revival that had so stultified American
principles of simplicity.
architecture during the nineteenth century was
A sheet of particleboard that had been
at last stopped in its tracks. Ironically, Wright’s
customized to customer specifications.
25 influence was itself dampened by the arrival
A pair of hotels constructed in the form of
of the International Style, and his final massive
a swan and a dolphin.
project, the Solomon B. Guggenheim Museum
in New York City, is today regarded as Wright’s A second home at the beach that had
take on modernism’s rigid preoccupation with originally been designed for a city.
30 geometry. A casino that featured an open floor plan
but was shrouded in darkness.

Question 1

The passage suggests that Wright’s designs


embodied a rejection of which of the following?

The rebirth of well-known traditional


European designs
Excessively nationalistic forms that depend
upon shared blood
The idea that the method of creation is
more important than the result
The preoccupation with natural, organic
forms
His own newfound obsession with
modernist geometry

180
DO NOT DISTRIBUTE
| © TPR Education IP Holdings, LLC.
LESSON 4 VERBAL

SCRATCH PAPER

DO NOT DISTRIBUTE © TPR Education IP Holdings, LLC. | 181


MANUAL FOR THE GRE

In a believable yet theoretical model, geologist Question 2


Paul Stevenson argues that the recent earthquake
activity in the Andes Mountains might result in It can be inferred from the passage that Stevenson
Line even more earthquakes in the area, where the might conclude which of the following about
5 Nazca tectonic plate meets the South American earthquakes along the Nazca-South American fault
4 plate. It is true that earthquakes release pent-up on the basis of his theory?
energy under the Earth’s surface, reducing the
They would occur less frequently if there
likelihood of future earthquakes at least in the
were no volcanoes near the fault line.
short term. However, the energies released by
10 earthquakes can also trigger other events that They are powered by energies released
cause seismic activity. Stevenson contends that from other earthquakes.
volcanic activity along the fault will be triggered They occur with greater strength as faults
by the recent earthquakes. These volcanoes will are increasingly disturbed by seismic
shake up the interior of the Earth when they erupt, events.
15 shifting rocks around and destabilizing the crust They are more regular in the absence of
along the fault line. With the fault disturbed anew, disturbed faults.
the risk of another earthquake will also increase. They are likely to shake up rocks in the
Thus, even though the initial earthquakes might Earth’s interior and destabilize the crust.
release stored energy in the Earth’s crust, hence
20 immediately relieving the stresses on the Earth’s
Question 3
crust, this released energy will trigger other
events, which could themselves increase those According to the passage, which of the following is
very stresses once more. true concerning the Nazca-South American tectonic
fault line?

The fault has been dangerously active as of


Question 1 late.
The fault’s stability is ensured through
The passage suggests which of the following is seismic activity.
true regarding the volcanoes along the Nazca-South
The likelihood of an earthquake along this
American fault?
fault is greatly increased by local volcanic
They would erupt less frequently if more activity.
earthquakes occurred along the fault. Earthquakes preserve the pent-up energy in
They can relieve the stresses of distant the Earth’s crust along the fault line.
faults by erupting. Earthquakes along the fault line could
They may trigger other tectonic events trigger nearby volcanic activity.
along the same fault line that set off their
eruptions.
They are more likely to cause earthquakes
by erupting than by destabilizing the
Earth’s interior.
They are more likely to be caused by
tectonic energy than other seismic events.

182
DO NOT DISTRIBUTE
| © TPR Education IP Holdings, LLC.
LESSON 4 VERBAL

SCRATCH PAPER

DO NOT DISTRIBUTE © TPR Education IP Holdings, LLC. | 183


MANUAL FOR THE GRE

The criticism of electronic books, or ebooks, by For the following question, consider each of the
literary scholars reveals as much about their choices separately and select all that apply.
confusion as it does about their elitism. In recent
Line years, many screeds have been penned by such
5 well-known professional authors as Ursula K. Question 2
4 LeGuin promoting the pleasures of the literary
Which of the following statements about “cheap
experience in printed form—the tactile feel of the
thrills” is supported by the passage?
paper book, the smell of its binding, and the ease
of reading a printed book while in the bathtub—
 Readers of genre fiction have a difficult
10 while simultaneously decrying the arrival of
time understanding the subtleties of
cheap, limitless digital stories, many of which are
literary fiction because they lack the
genre tales of romance, adventure, and mystery.
necessary sophistication.
Many of these same critics have on occasion
shown their true bias—that reading remain a  Genre stories such as romance, adventure,
15 pastime of those sufficiently sophisticated to and mystery are likely to contain more so-
linger over a felicitous turn of phrase, and that called cheap thrills than do other types of
published works avoid pandering to the masses books.
eager for nothing more than cheap thrills.  In addition to providing intellectual
stimulation, books may also provide
visceral entertainment.

Question 1
Question 3
The author of the passage cites LeGuin most likely in
order to In the context in which it appears, “pandering”
(highlighted) most nearly means
suggest that criticisms of ebooks leveled
by women are more likely to be taken disappointing
seriously than those leveled by men catering
offer an explanation for the fact that not all challenging
critics can command the same amount of joining
attention with their essays lowering
demonstrate that the group of people
opposed to electronic books includes but
is not limited to literary scholars
suggest that employing a strictly utilitarian
approach to reading may be unproductive
provide support for the theory that a
certain level of literary fame causes undue
reverence for traditional publishing

184
DO NOT DISTRIBUTE
| © TPR Education IP Holdings, LLC.
LESSON 4 VERBAL

SCRATCH PAPER

DO NOT DISTRIBUTE © TPR Education IP Holdings, LLC. | 185


MANUAL FOR THE GRE

Was Andrew Jackson (1767-1845) a great Question 2


president? On its face, the answer to this
question appears obvious, as suggested by his
The passage suggests that anyone wishing to
Line status on the twenty-dollar bill. His reputation
evaluate Jackson’s presidency must confront which
5 in popular history was solidified by his refusal,
of the following contradictions?
4 as the first president to hail from outside the
hallowed enclaves of Virginia and Massachusetts, The contrast between Jackson’s awareness
to recharter the National Bank for a third time. of his place of birth and the place of birth
But his popularity has been severely undercut of his predecessors
10 by twentieth-century historians’ re-evaluation of The gap between Jackson’s achievements
other portions of his presidency, particularly his and those of Lincoln and Roosevelt
treatment of Native Americans and his imposition The irony in placing the man who closed
of the Specie Circular. As Arroyo has stated, the National Bank onto the national
“There is a reluctance to maintain his stature as currency
15 one of the top-tier presidents, such as Lincoln or
The discrepancy between his actions
Roosevelt, owing to the decidedly mixed nature
regarding the National Bank and other
of his accomplishments.”
actions during his presidency
The tension between nineteenth-century
popular opinion and twentieth-century
popular opinion
Question 1

Select the sentence in the passage whose function


Question 3
is to indicate the range of reasons for the change in
perspective on Jackson’s presidency. It can be inferred that the presidents who resided in
the “hallowed enclaves” mentioned in the passage
were generally

lionized by contemporary historians


supporters of a national bank
in favor of expanded rights for Native
Americans
featured on American currency
participants in the American Revolution

186
DO NOT DISTRIBUTE
| © TPR Education IP Holdings, LLC.
LESSON 4 VERBAL

SCRATCH PAPER

DO NOT DISTRIBUTE © TPR Education IP Holdings, LLC. | 187


MANUAL FOR THE GRE

SUMMARY
Reading Basic Approach for Retrieval, Inference & Specific
Purpose Questions

Step 1: Work the Passage

• Main Idea

• Structure

• Author’s side

Step 2: Understand the Question

• Subject

• Task

Step 3: Find the information in the passage that


addresses the task of the question.

Step 4: Use Process of Elimination

• Recycled language

• Extreme language

• No such comparison

• Reversals

• Outside knowledge

• Emotional appeals

188
DO NOT DISTRIBUTE
| © TPR Education IP Holdings, LLC.
Lesson 5
Math

DO NOT DISTRIBUTE
MANUAL FOR THE GRE

THE CHART
Questions 1 to 3 are based on the following data.

5
CARS SOLD BY DEALER X, 2009-2013
Number of Cars Sold Mean and Median Prices of Cars Sold
$60,000 Mean
Year Number (arithmetic
$50,000 mean)

2009 650
$40,000 Median

2010 461
$30,000
2011 595
$20,000
2012 320
$10,000
2013 610
$0
2009 2010 2011 2012 2013

190
DO NOT DISTRIBUTE
| © TPR Education IP Holdings, LLC.
LESSON 5 MATH

GET YOUR BEARINGS


Before you start the questions, spend a few seconds looking over the charts.

• Scroll down to find any data or notes that aren’t visible.


• Read titles and legends.
• Check units.
5

WORKING THE QUESTIONS


1. Read the question and determine
• which chart the question deals with
• what information you need to find
• what calculations you need to perform
2. Work the chart(s)
• Get the information you need from the chart(s).
• Write down what you found on your scratch paper.
3. Approximate or calculate
• Are the numbers in the answers far apart? If so, you can probably estimate the answer.
• Use the calculator!

DO NOT DISTRIBUTE © TPR Education IP Holdings, LLC. | 191


MANUAL FOR THE GRE

Question 1

Which of the following is the closest to the mean


of the prices of the 930 cars sold in 2012 and 2013
combined?

$30,200
$34,800
5
$48,500

$51,500
$54,800

Question 2

Based on the information given, which of the following


statements about the sum of the prices of all cars sold
in a given year must be true?

Indicate all such statements.

 The sum of the prices for 2011 was greater than


the sum for 2010.
 The sum of the prices for 2012 was greater than
the sum for 2013.
 The sum of the prices for 2009 was greater than
the sum for 2012.

Question 3
The county government where Dealer X is located
collected a tax equal to 4% of the price of each car
sold in the county in 2013. Approximately how much
did the county collect in taxes from all cars sold at
Dealer X in 2013 ?

$576,000
$976,000
$1,342,000
$9,760,000
$13,420,000

192
DO NOT DISTRIBUTE
| © TPR Education IP Holdings, LLC.
LESSON 5 MATH

SCRATCH PAPER

DO NOT DISTRIBUTE © TPR Education IP Holdings, LLC. | 193


MANUAL FOR THE GRE

Questions 4 to 6 are based on the following data.

DISTRIBUTION OF THE 50 STATES OF THE UNITED STATES BY 2016 ELECTORAL VOTES

Category Electoral Votes Number of States


5 A > 50
B 29 to 49
C 19 to 28
D 16 to 18
E 12 to 15
F 8 to 11
G 5 to 7
H 1 to 4
0 2 4 6 8 10 12 14

Question 4

How many of the 50 states were in the five categories


from D to H ?

44
41
37
24
13

Question 5

In 2016, New Hampshire had 4 electoral votes. If the


ratio of the electoral votes of Maryland to those of New
Hampshire was 5 to 2, in which electoral category was
Maryland?

D
E
F
G
H

Question 6

The number of states in the three electoral categories


A, B, and C is approximately what percent less than the
number in the two electoral categories G and H ?

25%
33%
45%
75%
90%

194
DO NOT DISTRIBUTE
| © TPR Education IP Holdings, LLC.
LESSON 5 MATH

SCRATCH PAPER

DO NOT DISTRIBUTE © TPR Education IP Holdings, LLC. | 195


MANUAL FOR THE GRE

CHARTS DRILL
Questions 1 to 3 are based on the following data.

5 TOTAL EXPENDITURES AND SUMMARY OF


COMMUNITY SERVICE EXPENDITURES FOR CITY T
Total Expenditures in City T, 1998

Community Services
Public Transit 44%
14%
Urban Planning
and Development
1%

Corporate Services
Other 4%
12%
Culture and Tourism
6%
Public Works Police
11% 8%

100% = $5.6 billion


Summary of Community Services Expenditures by Category in City T

1997 1998
Children’s Services $211,000,000 $228,000,000
Retirement Homes $181,000,000 $176,000,000
Hostels $48,000,000 $62,000,000
Housing $264,000,000 $624,000,000
Libraries $121,000,000 $117,000,000
Public Health $89,000,000 $96,000,000
Social Development $9,000,000 $8,000,000
Social Services $906,000,000 $1,153,000,000

Total $1,829,000,000 $2,464,000,000

196
DO NOT DISTRIBUTE
| © TPR Education IP Holdings, LLC.
LESSON 5 MATH

SCRATCH PAPER

DO NOT DISTRIBUTE © TPR Education IP Holdings, LLC. | 197


MANUAL FOR THE GRE

Question 1

Which one of the following Community Service


programs had the greatest percent increase in
expenditure from the 1997 budget to the 1998
budget in City T ?

Children’s Services
5
Hostels

Public Health
Social Development
Social Services

Question 2
The amount by which City T’s Community Service
expenditures in 1998 exceeded Public Works
expenditures in 1998 was what percent of the
expenditures for Public Works in 1998 ?

Question 3

Which of the following statements can be inferred from


the graph or table?

Indicate all such statements.

 In 1998, more was spent on Social Services than


on any other area.
 In 1998, Community Service expenditures in City T
exceeded expenditures for the next four most
highly funded areas combined.
 In both 1997 and 1998, Library expenditures were
greater than those for Public Health.

198
DO NOT DISTRIBUTE
| © TPR Education IP Holdings, LLC.
LESSON 5 MATH

SCRATCH PAPER

DO NOT DISTRIBUTE © TPR Education IP Holdings, LLC. | 199


MANUAL FOR THE GRE

Questions 4 to 6 are based on the following data.


UNITED STATES UNEMPLOYMENT RATES AS A PERCENT OF
THE POPULATION 16 YEARS AND OLDER BY REGION
1998

5
1999

6 5.7
5.4
Percent 5.2
5 4.9
4.7
4.4 4.4
4.1
4
3.7
3.6

Northeast Midwest South West Pacific


Region
Question 4

Which region had the least decrease in its


unemployment rate from 1998 to 1999 ?

Northeast
Midwest
South
West
Pacific

Question 5

In 1999, if a total of 1.2 million people were


unemployed in the Northeast region, then what
was the overall population, in millions, of people 16
years or older in the Northeast that year?

Give your answer to the nearest million.

million

Question 6

Which of the following regions had the greatest


percent decrease in unemployment rate from 1998
to 1999 ?

Northeast
Midwest
South
West
Pacific
200
DO NOT DISTRIBUTE
| © TPR Education IP Holdings, LLC.
LESSON 5 MATH

SCRATCH PAPER

DO NOT DISTRIBUTE © TPR Education IP Holdings, LLC. | 201


MANUAL FOR THE GRE

RANGE AND STANDARD DEVIATION

Question 1

5 MONTHS OF EXPERIENCE DISTRIBUTION


OF EMPLOYEES OF COMPANY T
Months of Number of
Experience Employees
0−12 15
13−24 22
25−36 36
37−48 18
49−60 13
61−72 10
Total 114

Quantity A Quantity B
The range of the months
of experience of the
24 months
30 least experienced
employees of Company T

Quantity A is greater.
Quantity B is greater.
The two quantities are equal.
The relationship cannot be determined from the
information given.

Question 2

List A consists of 10 integers, each of which is


between 0 and 100.

List B consists of 10 integers, each of which is


between 100 and 200.

Quantity A Quantity B

The standard The standard


deviation of List A deviation of List B

Quantity A is greater.
Quantity B is greater.
The two quantities are equal.
The relationship cannot be determined from the
information given.

202
DO NOT DISTRIBUTE
| © TPR Education IP Holdings, LLC.
LESSON 5 MATH

SCRATCH PAPER

DO NOT DISTRIBUTE © TPR Education IP Holdings, LLC. | 203


MANUAL FOR THE GRE

Question 3

The heights of 900 fourth grade children were


measured and recorded. In this distribution, x inches
is at the 80th percentile. If there are 225 children
with heights that are greater than y inches but
less than x inches, then y is at approximately what
5 percentile in the distribution?

35th
40th
45th
50th
55th

204
DO NOT DISTRIBUTE
| © TPR Education IP Holdings, LLC.
LESSON 5 MATH

SCRATCH PAPER

DO NOT DISTRIBUTE © TPR Education IP Holdings, LLC. | 205


MANUAL FOR THE GRE

RANGE AND STANDARD DEVIATION DRILL

Question 1 of 3

For a certain distribution, the measurement 14.2


5 is 0.75 standard deviations above the mean and
the measurement 17.2 is 3.25 standard deviations
above the mean. What is the mean of the
distribution?

13.0
13.1
13.2
13.3
13.4

Question 2 of 3

Each of the following linear equations defines y as


a function of x for all integers x from 51 to 150. For
which of the following equations is the standard
deviation of the y-values corresponding to all the
x-values the greatest?

2
y = − x − 10
3
2
y = − 3 x + 10

2
y = x
3

y = 3x + 30

y = 6x + 2

Question 3 of 3

List L consists of the numbers −15, −8, 0, 8, and 15.


Which of the following lists of numbers have the
same range as the numbers in list L ?

Indicate all such lists.

 8, 9, 10, 38
 2, 3, 5, 7, 11, 32
 0, 1, 3, 7, 9, 15
 −6, −2, −1, 1, 24
 −20, −2, 0, 2, 20

206
DO NOT DISTRIBUTE
| © TPR Education IP Holdings, LLC.
LESSON 5 MATH

SCRATCH PAPER

DO NOT DISTRIBUTE © TPR Education IP Holdings, LLC. | 207


MANUAL FOR THE GRE

COMPREHENSIVE DRILL
Question 1 of 10

Quantity A Quantity B
512 – 510 (23)510
5 Quantity A is greater.
Quantity B is greater.
The two quantities are equal.
The relationship cannot be determined from the
information given.

Question 2 of 10

x>0

Quantity A Quantity B
2
1 1
2+
x

Quantity A is greater.
Quantity B is greater.
The two quantities are equal.
The relationship cannot be determined from the
information given.

Question 3 of 10

On a geography quiz, Emily and Katherine scored an


average (arithmetic mean) of 80 points. If Katherine
and Julia scored an average of 85 points, how many
more points did Julia score than Emily ?

15
10
7.5
5
2.5

Question 4 of 10
1
In a mixture of alcohol and water there is more
5
alcohol than water. What is the ratio of alcohol to

water in the mixture?

5:1
4:1
12 : 5
6:5
5:6

208
DO NOT DISTRIBUTE
| © TPR Education IP Holdings, LLC.
LESSON 5 MATH

SCRATCH PAPER

DO NOT DISTRIBUTE © TPR Education IP Holdings, LLC. | 209


MANUAL FOR THE GRE

Question 5 of 10

Of 110 customers at a restaurant, 63 order


hamburgers and 71 order french fries. If 22
customers order neither hamburgers nor french
fries, how many customers order both hamburgers
and french fries?
5

Question 6 of 10
M

O N

In the figure above, if MN = 2 , then the area of


the circle with the center O is

π

4
π

2
π

Question 7 of 10
B C
50°

F
E

D G
A

In the figure above, if BC  EF  DG and


ED = EA, what is the degree measure of ∠DEA ?

40°
50°
60°
65°
80°

210
DO NOT DISTRIBUTE
| © TPR Education IP Holdings, LLC.
LESSON 5 MATH

SCRATCH PAPER

DO NOT DISTRIBUTE © TPR Education IP Holdings, LLC. | 211


MANUAL FOR THE GRE

Question 8 of 10

If a machine can make x tires in y minutes, how many


minutes does it take the machine to make 1,272 tires
more than x when working at the same rate?

1,272y
5 x
1,272y
y+
x
1,272x
y+ y
1,272 x
x+ y
x 2 + 1,272 x
y

Question 9 of 10

Fred and Bobbie are book collectors, and Fred has


twice as many books as Bobbie does. One-fourth of
Fred’s books are signed by the authors, and three-
fifths of Bobbie’s books are signed by the authors.
If Fred and Bobbie combine their collections, what
fraction of the books are signed by the authors?

Question 10 of 10

The area of circle R is twice the area of circle S. If


the diameter of circle R is 20 centimeters, what is
the length, in centimeters, of the radius of circle S ?

5 2

2
5

5 2

10

10 2

212
DO NOT DISTRIBUTE
| © TPR Education IP Holdings, LLC.
LESSON 5 MATH

SCRATCH PAPER

DO NOT DISTRIBUTE © TPR Education IP Holdings, LLC. | 213


MANUAL FOR THE GRE

SUMMARY
Charts

• Get Your Bearings

1) Scroll down to find any data or notes that aren’t


visible.

2) Read titles and legends.

3) Check units.

• Working the Question

1) Read the question and determine

• which chart the question deals with

• what information you need to find

• what calculations you need to perform

2) Work the chart(s)

• Get the information you need from the chart(s).

• Write down what you found on your scratch


paper.

3) Approximate or calculate

• Are the numbers in the answers far apart?


If so, you can probably estimate the answer.

• Use the calculator!

Range and Standard Deviation

• Range is the difference between the greatest and the least


numbers in a data set.

• Standard deviation describes the spread of data from the


mean.

214
DO NOT DISTRIBUTE
| © TPR Education IP Holdings, LLC.
Lesson 5
Verbal

DO NOT DISTRIBUTE
MANUAL FOR THE GRE

TEXT COMPLETION: MULTIPLE BLANKS

Steps for Multiple Blank Text Completions


1 Start with the easiest blank.
2. Find the clues and transition words.
5 3. Come up with your own word or phrase for the blank.
4. Check each answer choice and use your scratch paper.
• POE: Yes! = ✓ No! = Don’t Know Vocab = ?
5. Repeat for the remaining blank(s).

Question 1
SCRATCH PAPER
The reviewer gave the new bistro a (i)________
review, claiming that while the food was innovative
and well executed, the service and decor were
(ii)________.

Blank (i) Blank (ii)


glowing inappropriate
scathing stunning
guarded lackluster

Question 2

Moby Dick, now regarded as a (i)________ work of


American literature, was virtually ignored when it
was first published, and it was not until many years
later that Melville’s achievements were (ii)________.

Blank (i) Blank (ii)


protracted deconstructed
tedious touted
seminal merited

Question 3

Persevering despite hundreds of (i)__________


attempts to produce a working light bulb, Edison
eventually triumphed, tenacity contributing to his
(ii)________ success.

Blank (i) Blank (ii)


constructive consequent
fruitless haphazard
auspicious celebrated

216
DO NOT DISTRIBUTE
| © TPR Education IP Holdings, LLC.
LESSON 5 VERBAL

Question 4 SCRATCH PAPER


Proponents of the International Style in architecture
called for reducing buildings to purely functional
form and found beauty in highlighting (i)_________
features. They rejected references to (ii)________
styles, offering designs indifferent to location, a
quality subsequently (iii)_________ by those who
viewed the style as bland or unappealing. 5
Blank (i) Blank (ii) Blank (iii)
structural oracular disparaged
aesthetic provincial embraced
hackneyed secular reclaimed

DO NOT DISTRIBUTE © TPR Education IP Holdings, LLC. | 217


MANUAL FOR THE GRE

Sentence Equivalence: Advanced Clues

Question 1 SCRATCH PAPER


Pundits dismissed the candidate’s remarks as
5 purely ________ : They had not been based on any
documented evidence.

 euphemism
 speculation
 disregard
 conjecture
 contingency
 rhetoric

Question 2

The CEO developed a newfound appreciation for


his underlings after they volunteered to work on
weekends without pay, but his _________ returned
upon receiving their written demand for health
insurance and paid vacation.

 disregard
 stature
 disdain
 credibility
 gratitude
 aloofness

Question 3

The modern availability of 24-hour news channels


has resulted in a loss of _______ by some media
organizations because stories are broadcast before
the details can be completely verified.

 vilification
 apathy
 reputability
 vacillation
 credibility
 instability

218
DO NOT DISTRIBUTE
| © TPR Education IP Holdings, LLC.
LESSON 5 VERBAL

Question 4 SCRATCH PAPER


Modern writers ________ to drape reality with pretty
phrases to such a degree that they show us every
aspect of life, both the putrid and the pure, with a
candor that can, at times, be altogether grim.

 disdain 5
 endeavor
 aspire
 loathe
 purport
 avow

Question 5

While the term “Dark Ages” is commonly and


disparagingly applied to the Early Middle Ages
to indicate a period of intellectual stagnation, the
development of the Carolingian minuscule, the first
modern handwriting, _______ this notion.

 belies
 chronicles
 buttresses
 repudiates
 reinforces
 transcribes

Question 6

The disparity between a fiction writer’s and a police


officer’s narrative of an event is stark: The former
is skilled at creating intrigue in even the most droll
incident, whereas the latter has been trained to
merely report the facts without _____ any details.

 disclosing
 debunking
 embellishing
 distorting
 squandering
 exacerbating

DO NOT DISTRIBUTE © TPR Education IP Holdings, LLC. | 219


MANUAL FOR THE GRE

Text Completion: Advanced Clues

Question 1 SCRATCH PAPER


The professor argues that the rise of the sound
bite, with its often instant appeal to the broad
5 swath of voters who favor reaction over reasoned
response, has led to the intellectual ________ of the
political party which mastered it and the increasing
irrelevance of the party which has not.

debilitation
orthodoxy
despondency
desperation
invigoration

Question 2

Due to the emergent nature of the situation, the


discussion on flood prevention and response
necessarily ________ the other topics on the
meeting’s agenda.

abrogated
stratified
preempted
inundated
predisposed

Question 3

The (i) _________ of the writer for the editor is well-


deserved, for the editor (ii) ________ the author’s
cherished text, in which each carefully chosen word
is the product of long, lonely hours of toil, with
seeming disregard.

Blank (i) Blank (ii)


esteem prunes
antipathy stifles
gratitude augments

220
DO NOT DISTRIBUTE
| © TPR Education IP Holdings, LLC.
LESSON 5 VERBAL

Question 4 SCRATCH PAPER


Despite its (i) ______ among beginners, the work
is known by expert musicians as one that requires
great (ii) ______ in terms of musicality, dynamics,
and tempo: While a novice may be able to achieve
technical proficiency, the less quantifiable aspects of
performing the piece can only be mastered through 5
years of practice.

Blank (i) Blank (ii)


disfavor celerity
ignorance ineptitude
popularity finesse

Question 5

One of the most salient (i) ______ of travel is the


ability to understand one’s own culture from
a different perspective; it is often not until we
encounter a (ii) ______ way of life that we realize
that what is normal to us is merely the result of
enculturation, not an innate correctness as it may
have previously seemed. It is through this (iii) _____
that alternative customs from other regions can be
seen not as inferior or bizarre but as equally valid
traditions that are no less natural than ours.

Blank (i) Blank (ii) Blank (iii)


benefits baffling realization
relaxations dissimilar bewilderment
drawbacks complementary zeal

Question 6

The advent of mobile computing has significantly


increased the (i) _________ of information. However,
an overexposure to media of any form limits
one’s ability to (ii) _________ information. As a
result, the speed at which one can (iii) _________
relevant information has actually decreased as
communication technology has evolved.

Blank (i) Blank (ii) Blank (iii)


inaccuracy assimilate exploit
accessibility revise replicate
redundancy disburse circumvent

DO NOT DISTRIBUTE © TPR Education IP Holdings, LLC. | 221


MANUAL FOR THE GRE

Text Completion: Relationships Between the Blanks

Question 1 SCRATCH PAPER


5 Elizabeth was (i)________ worker and therefore
expected (ii)________ of her work.

Blank (i) Blank (ii)


a diligent ignorance
a listless bewilderment
an insolent approbation

Question 2

Since the producer was (i)________ the staggering


costs typically associated with post-production
editing, she was surprised when she saw the
(ii)________ amount allocated to that line item in her
editor’s budget.

Blank (i) Blank (ii)


unaware of scant
cognizant of unintelligible
indifferent to apposite

222
DO NOT DISTRIBUTE
| © TPR Education IP Holdings, LLC.
LESSON 5 VERBAL

SCRATCH PAPER

DO NOT DISTRIBUTE © TPR Education IP Holdings, LLC. | 223


MANUAL FOR THE GRE

SENTENCE EQUIVALENCE DRILL

Question 1 of 10 SCRATCH PAPER


Unlike the bioluminescent plankton that lights up the
5 ocean in an organic display of chemical engineering,
the ink of the cephalopod ________ localized
darkness.

 precludes
 forestalls
 engenders
 obviates
 induces
 negates

Question 2 of 10

An island’s _______ other land masses can lead to


the evolution of species not found anywhere else
in the world due to the relatively small populations
of these species and their inability to migrate
elsewhere.

 sequestration from
 adjacency to
 contiguity with
 association with
 derivation from
 isolation from

Question 3 of 10

_________ by the question of which variable was


causing the reaction, the scientists ran successive
tests for weeks but ultimately came no closer to
achieving a resolution.

 Appeased
 Vexed
 Disquieted
 Exonerated
 Insinuated
 Mollified

224
DO NOT DISTRIBUTE
| © TPR Education IP Holdings, LLC.
LESSON 5 VERBAL

Question 4 of 10 SCRATCH PAPER


Despite the prevalent application of phrases
such as popular belief and it is well known, there
seldom tends to be even essential ________ among
individuals who proclaim a shared dogma.

 perturbation 5
 consensus
 assignation
 precedence
 accord
 dissension

Question 5 of 10

While the new police uniforms drew rave reviews on


social media, long-form journalists wrote multiple
editorial and investigative pieces _________
the police department’s extravagant albeit stylish
squandering of taxpayers’ dollars.

 extolling
 lauding
 admonishing
 advocating
 endorsing
 castigating

Question 6 of 10

The professor was known for being a _______


because every assignment was returned with
numerous corrections to the citation formatting
and grammar but few comments on the actual
substance of the paper.

 pedant
 maverick
 sycophant
 pundit
 redactor
 savant

DO NOT DISTRIBUTE © TPR Education IP Holdings, LLC. | 225


MANUAL FOR THE GRE

Question 7 of 10 SCRATCH PAPER


Proponents of so-called “free-range parenting”
value the benefits of greater independence for
children, while skeptics contend that the potential
risks to children’s safety ______ any psychological
advantages resulting from unsupervised time.
5
 censure
 balance
 outweigh
 substantiate
 eclipse
 reciprocate

Question 8 of 10

As the cane toad has no natural predators in


Australia, the species has been able to _______ in a
way that was unforeseen by those who had hoped
its introduction would mitigate the nation’s beetle
problem.

 proliferate
 dominate
 abate
 insinuate
 eradicate
 ravage

Question 9 of 10

Spontaneous, uncensored expression, a hallmark


of writers such as Woolf and Kerouac, has been
_________ in contemporary fiction by polished
prose, edited and re-edited to quotable perfection.

 mirrored
 supplanted
 distinguished
 displaced
 reflected
 memorialized

226
DO NOT DISTRIBUTE
| © TPR Education IP Holdings, LLC.
LESSON 5 VERBAL

Question 10 of 10 SCRATCH PAPER


If the team is to forestall certain failure, immediate
corrective action is paramount; no subsequent tasks
possess a whit of worth should the team ________
at this critical juncture.

 prevail 5
 founder
 unite
 triumph
 orchestrate
 falter

DO NOT DISTRIBUTE © TPR Education IP Holdings, LLC. | 227


MANUAL FOR THE GRE

TEXT COMPLETIONS DRILL

Question 1 of 10 SCRATCH PAPER


The creation of photorealistic objects onscreen,
5 though not necessarily one of the most onerous of
tasks in the visual effects industry, is still one most
bound to impress audiences given that the precision
with which it is practiced is immediately discernible
to even the most unsophisticated _________.

artist
lackey
virtuoso
layperson
critic

Question 2 of 10

While the professor often loudly criticized the greed


that attended commercial enterprise, his private
negotiations with the university that employed him
betrayed the ______ of his public position for he
demanded a large salary increase and perquisites
that might rival those of the prerevolutionary French
nobility.

avarice
erudition
hypocrisy
forthrightness
rectitude

Question 3 of 10

A renowned _________, Pablo Picasso broke from


conventional painting styles and forged his own
avant-garde idiom.

iconoclast
patriarch
conformist
aesthete
curmudgeon

228
DO NOT DISTRIBUTE
| © TPR Education IP Holdings, LLC.
LESSON 5 VERBAL

Question 4 of 10 SCRATCH PAPER


Ignoring the lesson that factionalism is a perennial
adjunct to political organization, political scientist
John Mueller argues that war is an institution that
can fade from the historical stage—a conclusion that
is perhaps more rooted in _______ than scholarship.
5
dissension
pragmatism
vanity
belligerence
utopianism

Question 5 of 10

The lawyer was known as much for her (i)_______ as


for her (ii)_______ : that is, she argued her point with
all due vigor but knew when to back down.

Blank (i) Blank (ii)


lethargy conciliation
pugnacity repudiation
prescience cacophony

Question 6 of 10

The study was (i)______ by many in the field as


flawed because, they argued, it clearly lacked the
meticulous analysis of possible (ii)_______ that could
have unexpectedly contributed to the published
results.

Blank (i) Blank (ii)


appraised regularities
debated anachronisms
entertained anomalies

DO NOT DISTRIBUTE © TPR Education IP Holdings, LLC. | 229


MANUAL FOR THE GRE

Question 7 of 10 SCRATCH PAPER


The drafters of the U.S. Constitution considered
a system of checks and balances a necessary
(i)______ against the plebiscitary (ii)_______ to which
a fickle electorate can subject a republican form of
government.
5
Blank (i) Blank (ii)
bulwark enumeration
legality demagoguery
compromise spasms

Question 8 of 10

Few (i)_______ dared to challenge the skilled debater;


though they disagreed with his arguments, they
were unable to articulate their opinions as effectively
as he could and invariably (ii)________.

Blank (i) Blank (ii)


advocates triumphed
proponents were vanquished
dissenters were amplified

Question 9 of 10

An unfortunate (i)_______ occurs within communities:


There are myriad organizations that need assistance
and many more residents who would like to engage
in volunteer work but there exists a (ii)_______ of
effective mobilization. Lacking the resources of a
company, non-profits are often unable to successfully
articulate their needs and use the available help as
productively as possible. Those community
agencies with full-time volunteer coordinators,
training programs, and clear delegation of tasks
are best able to adequately (iii)_______ their local
humanitarian resources.

Blank (i) Blank (ii) Blank (iii)


boon plethora inundate
disjunction paucity impede
serendipity synergy utilize

230
DO NOT DISTRIBUTE
| © TPR Education IP Holdings, LLC.
LESSON 5 VERBAL

Question 10 of 10 SCRATCH PAPER


The image of the architect as the lonely artist
drawing three-dimensional forms is (i)________ the
public’s understanding of the architect’s role. As a
result, buildings are viewed as the singular creations
of an artistic vision (ii)________ the architect.
Certainly architects should take much of the credit 5
for the form of a unique building, but the final
product is hardly a (iii)________. The architect relies
heavily upon façade consultants, engineers, and
skilled builders, while the form of the building may
depend, in addition, upon zoning regulations, cost,
and market demands.

Blank (i) Blank (ii) Blank (iii)


at odds with tangentially virtuoso
related to performance
central to but an collaborative
afterthought effort
to
irrelevant to justifiably physical
embodied by triumph

DO NOT DISTRIBUTE © TPR Education IP Holdings, LLC. | 231


MANUAL FOR THE GRE

SENTENCE EQUIVALENCE &


TEXT COMPLETIONS SUMMARY

Steps for Sentence Equivalence Questions


1. Find the clues and transition words.

2. Come up with your own word or phrase for the


blank. Write it down on your scratch paper.

3. Check each answer choice:


✓ an answer that sort of matches your word
× an answer that does not at all match your word
? any word you don’t know

Steps for Text Completions Questions


1. Find the clues and transition words.

2. Come up with your own word or phrase for the


blank. Write it down on your scratch paper.

3. Check each answer choice and use your scratch paper.


✓ an answer that sort of matches your word
× an answer that does not at all match your word
? any word you don’t know

232
DO NOT DISTRIBUTE
| © TPR Education IP Holdings, LLC.
Lesson 6
Math

DO NOT DISTRIBUTE
MANUAL FOR THE GRE

PROBABILITY
Question 1

A piggy bank contains exactly 6 dimes and 10


nickels. If four coins are selected at random from
the bank, without replacement, and the first 3
selected are nickels, what is the probability that
fourth coin selected is also a nickel?
6
1

16
1

13
3

13
1

4
7

13

Working with Probability


want
0≤ ≤1
total
Write out the sequence of events and find the probability of
each event.

234
DO NOT DISTRIBUTE
| © TPR Education IP Holdings, LLC.
LESSON 6 MATH

SCRATCH PAPER

DO NOT DISTRIBUTE © TPR Education IP Holdings, LLC. | 235


MANUAL FOR THE GRE

Question 2

Of the 40 employees at a certain company, 20 are


available to meet on Monday, 17 are available to
meet on Wednesday, and 8 are available neither
Monday nor Wednesday. What is the probability that
a randomly selected employee is available to meet
on both Monday and Wednesday?

Question 3

Alejandro flips a coin four times. What is the


probability that he gets heads on the first 2 throws
and tails on the last 2 throws?

1

16
1

8
1

4
1

2
3

4

To find the probability of consecutive events,


multiply the probability of each event.
Probability of A and B = Probability of A × Probability of B

236
DO NOT DISTRIBUTE
| © TPR Education IP Holdings, LLC.
LESSON 6 MATH

SCRATCH PAPER

DO NOT DISTRIBUTE © TPR Education IP Holdings, LLC. | 237


MANUAL FOR THE GRE

Question 4

Griffin has a bag of marbles that contains only six


black marbles and four red marbles. If he removes
three marbles at random without replacing any of
the marbles, what is the probability that all three
marbles selected will be red?

6
2
5
1

6
3

25
8

125
1

30

Question 5

Ann has a deck of 52 cards, made up of four suits,


each with cards numbered 1 through 13. If she
selects a card at random, what is the probability that
she selects a 2 or a 7 or a 9 ?

1
3,825

1
64

3
52

3
13

3
4

To find the probability that either of two mutually exclusive


events happens, add the probabilities of each event.
Probability of A or B = Probability of A + Probability of B

238
DO NOT DISTRIBUTE
| © TPR Education IP Holdings, LLC.
LESSON 6 MATH

SCRATCH PAPER

DO NOT DISTRIBUTE © TPR Education IP Holdings, LLC. | 239


MANUAL FOR THE GRE

Question 6
1
The probability of rain on each of five days is ,
2 6
except on the first day, when it is and on the last
4 5
day when it is . What is the probability that rain
5
will occur on at least one of the five days?

6 1

675
5

72
5

27
22

27
67

72

To find the probability for “at least” questions, find the


probability that the event does not happen, and subtract it from 1.
Probability of A = 1 – Probability of not A

Question 7

0.04 0.05
0.18 0.27 0.24 0.22

0 1 2 3 4 5 6
Y

The figure above shows the probability distribution


of a continuous random variable Y. For each of the
six intervals shown, the figure gives the probability
that the value of Y is in that interval. What is the
probability that 2 < Y < 5 ?

240
DO NOT DISTRIBUTE
| © TPR Education IP Holdings, LLC.
LESSON 6 MATH

SCRATCH PAPER

DO NOT DISTRIBUTE © TPR Education IP Holdings, LLC. | 241


MANUAL FOR THE GRE

PROBABILITY DRILL
Question 1 of 3

One third of the seventy five colored tiles in a bag


are blue, 40% of the remaining tiles are green, and
all the other tiles are white. If one tile is selected at
random, what is the probability that the selected tile
will be white?
6

Question 2 of 3

Four people are playing a game in which each person


rolls a six-sided die at the same time.

Quantity A Quantity B
The probability that
2
at least two of the
3 people roll the same
number

Question 3 of 3

A bowl contains 4 red marbles and 12 blue marbles. If


two marbles are selected at random one after the other
without replacement, what is the probability that both a
red and a blue marble will be selected?

1

5
1

4
2

5
1

2
3

4

242
DO NOT DISTRIBUTE
| © TPR Education IP Holdings, LLC.
LESSON 6 MATH

SCRATCH PAPER

DO NOT DISTRIBUTE © TPR Education IP Holdings, LLC. | 243


MANUAL FOR THE GRE

QUADRATICS
Question 1
x2 + x – 20 = 0

Quantity A Quantity B
x 3

Quantity A is greater.
6 Quantity B is greater.
The two quantities are equal.
The relationship cannot be determined from the
information given.

Working with Quadratics

• Put the equation in the form x2 + bx + c = 0.


• Underneath the equation, write down (x )(x ) = 0.
• Remember A.M.
Add Multiply
x2 + bx +c = 0
• Find two numbers that multiply to c and add to b. If
necessary, write out every factor of c.

Question 2
x2 + kx + 12 = 0

In the equation shown above, k is a constant. Which


of the following is a possible value of k ?

Indicate all such values.

 –13
 –7
 –1
 0
 1
 7
 13

Question 3
x2 + 15x + 56 = 0

What is the difference between the greatest and


least roots of the equation shown above?

244
DO NOT DISTRIBUTE
| © TPR Education IP Holdings, LLC.
LESSON 6 MATH

SCRATCH PAPER

DO NOT DISTRIBUTE © TPR Education IP Holdings, LLC. | 245


MANUAL FOR THE GRE

Question 4

If x 2 + 3x 1 = 18 , which of the following could be the


value of x ?

–6

6 _1
6

_1
3

Question 5

1
x=y+
2
Quantity A Quantity B
x2 – 2xy + y2 x−y

Quantity A is greater.
Quantity B is greater.
The two quantities are equal.
The relationship cannot be determined from the
information given.

Question 6
a2 + b2 = 16
(a + b)2 = 30

For the equations shown above, what is the value of ab ?

ab =

246
DO NOT DISTRIBUTE
| © TPR Education IP Holdings, LLC.
LESSON 6 MATH

SCRATCH PAPER

DO NOT DISTRIBUTE © TPR Education IP Holdings, LLC. | 247


MANUAL FOR THE GRE

QUADRATICS DRILL
Question 1 of 3

If x2 + 19x + 48 = 0, then which of the following is a


possible value of x ?

Indicate all such values.


6
 –2
 –3
 –6
 –8
 –12
 –16
 –24
 –48

Question 2 of 3

In the equation x2 + px – 18 = 0, one of the roots is 6


and p is a constant. What is the value of p ?

–3
–1
3
6
18

Question 3 of 3
7+2 5 −1 5 +1 7−2 =

248
DO NOT DISTRIBUTE
| © TPR Education IP Holdings, LLC.
LESSON 6 MATH

SCRATCH PAPER

DO NOT DISTRIBUTE © TPR Education IP Holdings, LLC. | 249


MANUAL FOR THE GRE

FUNCTIONS
Question 1

For all positive integers greater than 1, let x  y be


defined as the remainder when x is divided by y.

Quantity A Quantity B
5  6 65
6
Quantity A is greater.
Quantity B is greater.
The two quantities are equal.
The relationship cannot be determined from the
information given.

Question 2

If k is a positive integer and f(k) = 3k, for how many


values of k is f(k) < 729 ?

3
4
5
6
7

Question 3

For all values of x, the function f is defined as


follows.

x2 +1 if x is an integer
f ( x) = 1
if x is not an integer
x

For which of the following values of x is f(x) greater


1
than f _ ?
3

Indicate all such values.

1

4

 0
1

2

 2
250
DO NOT DISTRIBUTE
| © TPR Education IP Holdings, LLC.
LESSON 6 MATH

SCRATCH PAPER

DO NOT DISTRIBUTE © TPR Education IP Holdings, LLC. | 251


MANUAL FOR THE GRE

PATTERNS AND SEQUENCES

Question 1

Which of the following is the units digit of 3333 ?

0
6 1
3
7
9

Question 2

48, 20, 6, −1

In the sequence of numbers above, each term after


the first is found by dividing the previous term by a
and subtracting b from that result. What is the value
of b – a ?

b–a=

Question 3

a ,a ,a ,…, an ,…
1 2 3
In the sequence shown above, each term after the
first is equal to the preceding term plus the constant
k. If the average (arithmetic mean) of a2, a3, and a4 is
10, what is the average of a1 and a5 ?

2
4
6
10
30

252
DO NOT DISTRIBUTE
| © TPR Education IP Holdings, LLC.
LESSON 6 MATH

SCRATCH PAPER

DO NOT DISTRIBUTE © TPR Education IP Holdings, LLC. | 253


MANUAL FOR THE GRE

FUNCTIONS AND PATTERNS AND SEQUENCES DRILL


Question 1 of 3

For all positive integers a and b,


1
a∇ b =
1 1
+
6 a b

Quantity A Quantity B
3
3 ∇ 4
4

Quantity A is greater.
Quantity B is greater.
The two quantities are equal.
The relationship cannot be determined from the
information given.

Question 2 of 3
x +14
The function f is defined by f = 5 x 2 2x +10
4
for all numbers x. What is the value of f(5) ?

6
20
119
125
178

Question 3 of 3

The sequence of numbers a1,a2 ,a3 ,…,an ,… is defined



1 1
as an = for each integer n ≥ 1.
n +1 n + 2
What is the sum of the first 15 terms of this sequence?

1
272

1
6
7

16

1
2

15
254
DO NOT DISTRIBUTE
| © TPR Education IP Holdings, LLC.
34
LESSON 6 MATH

SCRATCH PAPER

DO NOT DISTRIBUTE © TPR Education IP Holdings, LLC. | 255


MANUAL FOR THE GRE

COMPREHENSIVE DRILL
Question 1 of 10
0≤ g ≤5
0≤ h ≤1

Which of the following could be the value of g − h ?


6 Indicate all such values.

 –1
 0
 1
 2
 3
 4
 5

Question 2 of 10

There are four cookies in a jar: chocolate chip,


peanut butter, sugar, and oatmeal. If two are chosen
at random, what is the probability that one of the
two cookies will be chocolate chip?

1

12
1

6
1

2
2

3
3

4

256
DO NOT DISTRIBUTE
| © TPR Education IP Holdings, LLC.
LESSON 6 MATH

SCRATCH PAPER

DO NOT DISTRIBUTE © TPR Education IP Holdings, LLC. | 257


MANUAL FOR THE GRE

Question 3 of 10

Jordan is j years old. Five years ago, if Marshall was


three times older than Jordan will be in two years,
then, in terms of j, how old is Marshall now?

j
– 11
3
6
j
–5
3
3j + 5

3j + 11

j+7

Question 4 of 10

m and n are positive integers such that 81m = 3n.

Quantity A Quantity B

1 m

4 n

Quantity A is greater.
Quantity B is greater.
The two quantities are equal.
The relationship cannot be determined from the
information given.

Question 5 of 10

According to surveys at a supermarket chain, 12.5%


of its customers bought organic produce in 2012
and 50% of its customers bought organic produce
in 2014. From 2012 to 2014, what was the percent
increase in the fraction of customers who bought
organic produce?

3%
20%
30%
200%
300%

258
DO NOT DISTRIBUTE
| © TPR Education IP Holdings, LLC.
LESSON 6 MATH

SCRATCH PAPER

DO NOT DISTRIBUTE © TPR Education IP Holdings, LLC. | 259


MANUAL FOR THE GRE

Question 6 of 10

Eight points are equally spaced on a circle. If 3 of


the 8 points are to be selected at random, what is
the probability that a triangle having the 3 points
chosen as vertices will be a right triangle?

1
14
6
1
7
3
14
3
7
6
7

Question 7 of 10

x2 ≥ 36 and y2 ≤ 9

Quantity A Quantity B
x
–2
y

Quantity A is greater.
Quantity B is greater.
The two quantities are equal.
The relationship cannot be determined from the
information given.

260
DO NOT DISTRIBUTE
| © TPR Education IP Holdings, LLC.
LESSON 6 MATH

SCRATCH PAPER

DO NOT DISTRIBUTE © TPR Education IP Holdings, LLC. | 261


MANUAL FOR THE GRE

Question 8 of 10

Points A, B, and C, lie on the circumference of a

circle with center O. If the circle has a diameter of


p
2 and arc ABC has a length of , what is the least
30
possible measure, in degrees, of angle AOC ?

6 degrees

Question 9 of 10

2d + e + 2f = 0
3d – e – 5f = 0

For the system of equations shown above, if d ≠ 0,


the ratio of f to d is

3

2
5

3
2

3
3

5
5

3

Question 10 of 10

1
For all numbers , where n > 0, which of the
n
following operations carried out on both the

numerator and denominator will always produce an

equivalent number?

Indicate all such operations.

 Multiplying by n
 Dividing by 9
 Subtracting 3

262
DO NOT DISTRIBUTE
| © TPR Education IP Holdings, LLC.
LESSON 6 MATH

SCRATCH PAPER

DO NOT DISTRIBUTE © TPR Education IP Holdings, LLC. | 263


MANUAL FOR THE GRE

SUMMARY
Working with Probability

want
0 ≤     ≤ 1
total
Write out the sequence of events and find the
probability of each event.

Working with Quadratics

• Put the equation in the form x 2 + bx + c = 0.

• Underneath the equation, write down (x )(x ) = 0.

• Remember A.M.

Add Multiply

x 2 + bx +c = 0

• Find two numbers that multiply to c and add to b. If


necessary, write out every factor of c.

264
DO NOT DISTRIBUTE
| © TPR Education IP Holdings, LLC.
Lesson 6
Verbal

DO NOT DISTRIBUTE
MANUAL FOR THE GRE

READING BASIC APPROACH

Reading Basic Approach for Complex Questions


Step 1: Work the Passage
• Main Idea
6 • Structure
• Author’s side
Step 2: Understand the Question
• Subject
• Task
Step 3: Find the information in the passage that addresses the
task of the question.
Step 4: Use Process of Elimination
• Recycled language
• Extreme language
• Reversals (weaken answers on strengthen
questions / strengthen answers on weaken
questions)
• Out of scope

266
DO NOT DISTRIBUTE
| © TPR Education IP Holdings, LLC.
LESSON 6 VERBAL

SCRATCH PAPER

DO NOT DISTRIBUTE © TPR Education IP Holdings, LLC. | 267


MANUAL FOR THE GRE

COMPLEX QUESTIONS
So remarkable and definite are the similarities .
among the Indo-European, or Aryan, languages
that linguists are convinced they all derived from
an earlier language spoken by some community in
5 the prehistoric past. While it is known that Latin
began as a rustic dialect in the province of Latium,
no one knows where Proto-Aryan was first spoken.
6 Some speculate that Proto-Aryan was first used
in Southern Russia, while still others point to the
10 Iranian plateau as a more likely potential cradle.
Though some philologists believe that the Old
Indic and Persian of the Avesta contain the most
archaic features of Aryan found to date, this does
not necessarily fix the habitat of these early Aryan-
15 speaking peoples closer to Asia than to Europe.
Consider Icelandic. Though this language has
strayed far from its birthplace, it preserves many
of the characteristics discarded by those who
remained behind.

Question 1

Which of the following, if true, would most weaken


the author’s contention that the habitat of early
Aryan-speaking peoples is not necessarily closer to
Asia than to Europe ?

Old Indic evolved before Persian.


Linguists consider the way Icelandic
changed over time unique.
Iceland is thousands of miles from Asia.
The oldest known version of the Avesta is a
copy of a lost text.
Literature exists which predates the Avesta.

268
DO NOT DISTRIBUTE
| © TPR Education IP Holdings, LLC.
LESSON 6 VERBAL

SCRATCH PAPER

DO NOT DISTRIBUTE © TPR Education IP Holdings, LLC. | 269


MANUAL FOR THE GRE

Theorists are divided about the cause of the


Permian mass extinction. Some hypothesize that
the impact of a massive asteroid caused the
Line sudden eradication of most species. However, a
5 look at the carbon-isotopic record suggests that
existing plant communities were decimated and
revived several times. To produce such a pattern
would require a succession of asteroid strikes
thousands of years apart. Other theorists have
10 proposed that volcanic explosions raised the
6 CO2 levels, leading to intense global warming.
One problem with this theory is that it cannot
explain the mass marine extinctions at the end
of the Permian period. A new theory proposes
15 that the combination of rising concentrations of
toxic hydrogen sulfide in the world’s oceans and
gradual oxygen depletion in the surface waters
caused the extinctions. Fortunately, this theory
is testable. If this theory is true, then oceanic
20 sediments from the Permian period will yield
chemical evidence of a rise in hydrogen sulfide-
consuming bacteria.

Question 1

Which of the following, if true, would most


strengthen the theory that increased levels of
toxic hydrogen sulfide caused the Permian mass
extinction?

The discovery of a new crater indicating an


a previously unknown asteroid strike
A new study which concludes that
volcanoes were less active than normal
prior to the Permian mass extinction
The discovery that hydrogen sulfide-
consuming bacteria had not evolved until
well after the Permian mass extinction
New evidence which suggests that intense
global warming can cause mass marine
extinctions
Evidence in the fossil record that several
species of clams that prey upon hydrogen
sulfide-consuming bacteria evolved during
the Permian period

270
DO NOT DISTRIBUTE
| © TPR Education IP Holdings, LLC.
LESSON 6 VERBAL

SCRATCH PAPER

DO NOT DISTRIBUTE © TPR Education IP Holdings, LLC. | 271


MANUAL FOR THE GRE

It would be expected that a novel by a figure


as controversially brilliant and difficult as David
Foster Wallace would attract both the adoration
Line and ire of literary critics. Furthermore, when a
5 novel addresses the ceaseless consumption of
entertainment that constitutes modern American
culture, as does Wallace’s masterpiece Infinite
Jest (1996), it would be a surprise if a critic didn’t
take a jab at a piece of entertainment implicitly
10 purporting to satirize itself. However, other than
6 an occasional complaint about self-indulgence,
such slings and arrows were surprisingly absent
from scholarly commentary upon the novel. One
reason for this could be the fact that his novel
15 was simply so shockingly new, both in form and
content, that one couldn’t argue its artistic bona
fides, no matter how metacritical one would like
to be. But what if the truth were a little more
disturbing—that its very complexity prevented
20 anybody, readers and critics alike, from fully
understanding the text to begin with?

Question 1

Which of the following, if true, would cast the most


doubt on the author’s conjecture about the reason
for the critical response to Infinite Jest?

Other novels such as James Joyce’s


Ulysses used new forms when first
published.
The author of Infinite Jest had commented
publicly that interpretations offered
by several critics convey the intended
meaning of his work.
Literary critics are seldom correct in their
interpretations of more recently published
works.
Several critics overlooked the satirical
elements of Infinite Jest.
Wallace’s other works are considered more
accessible.

272
DO NOT DISTRIBUTE
| © TPR Education IP Holdings, LLC.
LESSON 6 VERBAL

SCRATCH PAPER

DO NOT DISTRIBUTE © TPR Education IP Holdings, LLC. | 273


MANUAL FOR THE GRE

PRACTICE
Although the ancients had more or less a Question 2
solid understanding of Aristophanes’ political
leanings since they were his contemporaries, we The information in the passage suggests that which
Line moderns have only his extant plays and what of the following would have been an important
5 little reliable biographical information about the factor to an ancient rhetorician who selected one of
comic playwright ancient rhetoricians preserved Aristophanes’ plays from which to teach?
to make any sort of conclusion. The plays that
The play contains political jokes, which
6 have survived to the modern day were selected
mock opponents of conservatives in
by the ancients because they were considered
10 Aristophanes’ best; when selecting examples Athenian politics.
from which to teach, ancient rhetoricians sifted The play’s dialogue was considered by the
through his plays to find the most persuasive ancients to be relatively weak.
dialogue. However, these rhetoricians also had The play advocates egalitarianism in its
their own personal biases, and these biases plot.
15 affected their appreciation of the plays, so the The play was immensely popular among its
impression we get about Aristophanes from his contemporary viewers.
plays is that he is a staunch conservative and
aristocrat, but this might simply be because his
The play is more about characters’
persuasive dialogue than about political
audience was aristocratic and conservative—after
views.
20 all, comedies are written to entertain. Therefore,
I believe that modern scholars cannot identify
what Aristophanes’ political leanings actually Question 3
were.
Which of the following, if true, would most seriously
weaken the argument made in the passage about
Question 1 whether moderns can deduce Aristophanes’ political
affiliations?
The author of this passage is primarily focused on
discussing
A survey of all of Aristophanes’ plays
indicates an overwhelming presence of
the reasons ancient rhetoricians selected
conservative political comments.
certain of Aristophanes’ plays
Aristophanes’ aristocratic leanings Certain scholars have identified which of
Aristophanes’ plays are no longer extant.
the educational merits of Aristophanes’
plays
Lists of Aristophanes’ political actions are
available in the archaeological record.
the difficulty of identifying a political
viewpoint for Aristophanes
Modern scholars have translated ancient
pedagogical texts that indicate the reasons
the prevalence of political bias in comedy ancient rhetoricians taught from certain
writing in ancient times when compared to plays of Aristophanes.
comedy writing in the modern day
Modern scholars have found Aristophanes’
political humor to be written chiefly to
entertain his audiences.

274
DO NOT DISTRIBUTE
| © TPR Education IP Holdings, LLC.
LESSON 6 VERBAL

SCRATCH PAPER

DO NOT DISTRIBUTE © TPR Education IP Holdings, LLC. | 275


MANUAL FOR THE GRE

What makes a captive dolphin so difficult to For the following question, consider each of the
reintegrate into the wild? From the 1980s to the choices separately and select all that apply.
1990s, researchers focused on several aspects
Line of its physiology, including the native intelligence
5 of the animal; it has a brain that is notably large Question 2
and complex for its body size, which makes
It can be inferred from the passage that Davenport
its behavior less predictable than that of other
and earlier researchers would agree with which of
species. Other factors believed to have major
the following statements about dolphins?
effects upon this task included the dolphin’s age—
10 all living creatures change their behavior patterns
6 as they get older—and its genetics. However, new
 Genetics predominates over other factors
in determining a dolphin’s success in
research has unearthed different obstacles in
reintegrating in the wild.
achieving this goal. James Davenport, a marine
mammal expert from Seattle, discovered through  Trainers standing at the surface of a pool
15 close observation that wild dolphins spend eighty can ease a dolphin’s reentry into the
percent of their time underwater, while captive oceans.
dolphins spend eighty percent of their time at  Dolphins are adaptable creatures whose
the surface. By engaging the dolphins below the behavior can be changed.
surface and eliminating the practice of trainers
20 standing at the surface of the pool, he was able
to facilitate the reintroduction of captive dolphins For the following question, consider each of the
into the oceans. choices separately and select all that apply.

For the following question, consider each of the Question 3


choices separately and select all that apply.
The last sentence of the passage serves which of
the following roles?
Question 1
 It compares the discoveries of Davenport
According to the passage, which of the following to the discoveries made by earlier
factors were considered from the 1980s to the researchers.
1990s to be most important in the reintegration of  It provides further explanation of how
dolphins into the wild? Davenport’s research supports his position.
 It justifies the decision to impede the
 The age of the dolphins reintegration of dolphins into the world’s
 Dolphins’ time spent underwater while in oceans.
captivity .
 The dolphins’ mental capacity

276
DO NOT DISTRIBUTE
| © TPR Education IP Holdings, LLC.
LESSON 6 VERBAL

SCRATCH PAPER

DO NOT DISTRIBUTE © TPR Education IP Holdings, LLC. | 277


MANUAL FOR THE GRE

In a recent study, Ernest Sendry announces that Question 2


he will examine the central question regarding
the Dutch tulip mania of 1637: Why did it Select the sentence that provides a projection about
Line happen? Using evidence such as investment the eventual outcome of Sendry’s study.
5 records, personal journals, and anti-speculative
pamphlets, Sendry notes that the 1621 Prussian
phenomenon of debasing metal coins in order to Question 3
fund warfare bore many similarities to the later
tulip mania. More conventionally, and with a nod In the passage, the author is primarily concerned
to Charles Mackay’s landmark study on the topic, with
6 10
Sendry repeats the well-known anecdote of the
Dutchman who was willing to trade 12 acres of summarizing a well-known set of opinions
land for a single tulip bulb. analyzing a counterargument
Sendry doesn’t deny that the tulip mania evaluating another author’s purpose for
15 existed, but his study does illustrate that when writing a text
put into its proper context, the paucity of synthesizing several opinions into a
economic data nullifies previous economists’ consensus
characterization of the event as the world’s first
hypothesizing about the reasons behind a
economic bubble. Such a contrarian view will
historical event
20 naturally lead many to wonder whether he is
questioning the reasons for the event, or the very
nature of the event itself.

For the following question, consider each of the


choices separately and select all that apply.

Question 1

Based on the passage, the author would agree with


which of the following statements about Charles
Mackay’s study of tulip mania?

 Mackay was very familiar with most of the


primary sources that Sendry used in his
study.
 Sendry’s analysis is superior in that
it examines two potential views of a
historical event.
 It is regarded as a foundational study of the
tulip mania.

278
DO NOT DISTRIBUTE
| © TPR Education IP Holdings, LLC.
LESSON 6 VERBAL

SCRATCH PAPER

DO NOT DISTRIBUTE © TPR Education IP Holdings, LLC. | 279


MANUAL FOR THE GRE

SUMMARY
Reading Basic Approach for Complex Questions

Step 1: Work the Passage

• Main Idea

• Structure

• Author’s side

Step 2: Understand the Question

• Subject

• Task

Step 3: Find the information in the passage that


addresses the task of the question.

Step 4: Use Process of Elimination

• Recycled language

• Extreme language

• Reversals (weaken answers on strengthen


questions / strengthen answers on weaken
questions)

• Out of scope

280
DO NOT DISTRIBUTE
| © TPR Education IP Holdings, LLC.
Lesson 7
Math

DO NOT DISTRIBUTE
MANUAL FOR THE GRE

COORDINATE GEOMETRY
Question 1
y

Q (4, 5)

P (1, 1)
7 x

In the figure of the xy-plane shown above, what is


the length of PQ ?

4 3

5 2

Question 2

Coworkers Max and Selina are both driving to their


respective homes from work. Max drives west at
30 miles per hour for 20 minutes and then north at
60 miles per hour for 3 minutes. Selina drives east
at 30 miles per hour for 4 minutes and then south at
40 miles per hour for 3 minutes. What is the straight-
line distance, in miles, between Max’s home and
Selina’s home?

miles

282 |
DO NOT DISTRIBUTE
© TPR Education IP Holdings, LLC.
LESSON 7 MATH

SCRATCH PAPER

DO NOT DISTRIBUTE © TPR Education IP Holdings, LLC. | 283


MANUAL FOR THE GRE

Question 3
y

A B

G D C x

F E

7 In the figure of the xy-plane shown above, G, D, and


C are points on the x-axis, A, D, and E are points on
the y-axis, and squares ABCD and DEFG have areas
of 16 and 25, respectively. What is the length of line
BF (not shown)?

9
9 2
10 3
20 2
41

284 |
DO NOT DISTRIBUTE
© TPR Education IP Holdings, LLC.
LESSON 7 MATH

SCRATCH PAPER

DO NOT DISTRIBUTE © TPR Education IP Holdings, LLC. | 285


MANUAL FOR THE GRE

SLOPE
Question 1

In the xy-plane, line k passes through points A and


B, the coordinates of which are (2, 3) and (5, 10),
respectively. What is the slope of line k ?

Give your answer as a fraction.

Working with Slopes


Draw right triangles and look for Pythagorean triples.

Question 2

In the xy-plane, points C and D are located at (–3, 4)


and (5, –12), respectively. Line m passes through
points C and D. What is the slope of a line that is
perpendicular to line m ?

–4

–2
1

2
1

2

286 |
DO NOT DISTRIBUTE
© TPR Education IP Holdings, LLC.
LESSON 7 MATH

SCRATCH PAPER

DO NOT DISTRIBUTE © TPR Education IP Holdings, LLC. | 287


MANUAL FOR THE GRE

EQUATION OF A LINE
Question 1

What is the x-intercept of the line defined by the


equation y = 2x + 3 ?

⎛ 2 2⎞
⎜ , ⎟
⎝ 3 3⎠
7 ⎛ 3⎞
⎜ 0, − ⎟
⎝ 2⎠
⎛ 3 ⎞
⎜− , 0⎟
⎝ 2 ⎠

(–2, 0)

(0, –3)

Equation of a Line
y = mx + b
If you know two points, you can find the slope.
If you know a point and the slope, you can find any other point on
the line.

288 |
DO NOT DISTRIBUTE
© TPR Education IP Holdings, LLC.
LESSON 7 MATH

SCRATCH PAPER

DO NOT DISTRIBUTE © TPR Education IP Holdings, LLC. | 289


MANUAL FOR THE GRE

Question 2
y

•H

• x
G O

The equation of the line graphed in the


rectangular coordinate system above is

10 x
y= +2
11
Quantity A Quantity B
GO HO

Quantity A is greater.
Quantity B is greater.
The two quantities are equal.
The relationship cannot be determined from the
information given.

Question 3
y

y = g(x)

1
x
1

The figure above shows the graph of the function g


defined by g(x) = |3x| + 2 for all numbers x. For which
of the following functions h, defined for all numbers x,
does the graph of h intersect the graph of g ?

h(x) = x + 1
h(x) = x – 1
h(x) = 2x + 1
h(x) = 3x – 3

DO NOT DISTRIBUTE h(x) = 4x – 1


290 | © TPR Education IP Holdings, LLC.
LESSON 7 MATH

SCRATCH PAPER

DO NOT DISTRIBUTE © TPR Education IP Holdings, LLC. | 291


MANUAL FOR THE GRE

ARRANGEMENTS & COMBINATIONS


Question 1

A space shuttle crew consists of one Commander,


one Medical Officer, and one Chief Scientist. If there
are 7 candidates for Commander, 4 for Medical
Officer, and 2 for Chief Scientist, and no candidate
applies for more than one position, then how many
different shuttle crews are possible?

Arrangements & Combinations


1. Write dashes for each spot.
2. Label any restrictions.
3. How many options can go in each spot?
4. Does order matter?

Question 2

There are five runners in a race.

Quantity A Quantity B
10 The total number of
possible arrangements
for the runners from first
place to third place
Quantity A is greater.
Quantity B is greater.
The two quantities are equal.
The relationship cannot be determined from the
information given.

292 |
DO NOT DISTRIBUTE
© TPR Education IP Holdings, LLC.
LESSON 7 MATH

SCRATCH PAPER

DO NOT DISTRIBUTE © TPR Education IP Holdings, LLC. | 293


MANUAL FOR THE GRE

Question 3

A congressional committee on legislative


procedures is to be made up of four members.
If 10 representatives are available, how many
different groups are available to make up the four-
member ­committee?

60
120
210
400
7 720

Question 4

Six children, one boy and five girls, must stand in a


line. If the boy cannot stand first or last in line, in how
many different ways could the children be arranged?

720
480
360
240
120

Question 5

The co-op board of a certain residential building


must consist of two men and three women. If there
are six men and seven women available for the
committee, how many different groups could be
chosen for the committee?

65
525
1,050
1,287
100,800

294 |
DO NOT DISTRIBUTE
© TPR Education IP Holdings, LLC.
LESSON 7 MATH

SCRATCH PAPER

DO NOT DISTRIBUTE © TPR Education IP Holdings, LLC. | 295


MANUAL FOR THE GRE

ARRANGEMENTS & COMBINATIONS DRILL


Question 1 of 4

Beth has five trophies which she wants to display


from left to right on her mantle. How many different
arrangements of the trophies are possible?

20
60
120
7 176
216

Question 2 of 4

On Random Omelet Monday, a chef creates omelets by


randomly choosing three out of a possible six fillings.
How many different omelets can the chef make?

9
18
20
120
720

296 |
DO NOT DISTRIBUTE
© TPR Education IP Holdings, LLC.
LESSON 7 MATH

SCRATCH PAPER

DO NOT DISTRIBUTE © TPR Education IP Holdings, LLC. | 297


MANUAL FOR THE GRE

Question 3 of 4

Griffith Bank & Trust assigns each of its depositors


a 4-digit personal identification number (PIN). What
is the probability that a randomly generated PIN will
have no repeated digits?

Give your answer as a fraction.

Question 4 of 4

A sundae can be ordered with any of seven


possible toppings.

Quantity A Quantity B
The number of The number of
different ways to order different ways to order
a sundae with two a sundae with five
toppings ­toppings
Quantity A is greater.
Quantity B is greater.
The two quantities are equal.
The relationship cannot be determined from the
information given.

298 |
DO NOT DISTRIBUTE
© TPR Education IP Holdings, LLC.
LESSON 7 MATH

SCRATCH PAPER

DO NOT DISTRIBUTE © TPR Education IP Holdings, LLC. | 299


MANUAL FOR THE GRE

NUMBER THEORY
Question 1

Which of the following numbers are both multiples


of 3 and factors of 81 ?

Indicate all such numbers.

 3
 6
7  9
 18
 27
 81

Question 2

If the remainder when integer x is divided by 4 is 3


and the remainder when integer y is divided by 4 is 2,
what is the remainder when x + y is divided by 4 ?

1
2
3
4
5

Question 3

2x + 5y = z
x and y are positive integers and x is a multiple of 5.

Quantity A Quantity B
The remainder when z The remainder when z
is divided by 5 is divided by 10

Quantity A is greater.
Quantity B is greater.
The two quantities are equal.
The relationship cannot be determined from the
information given.

300 |
DO NOT DISTRIBUTE
© TPR Education IP Holdings, LLC.
LESSON 7 MATH

SCRATCH PAPER

DO NOT DISTRIBUTE © TPR Education IP Holdings, LLC. | 301


MANUAL FOR THE GRE

Question 4

If x and y are positive integers and 15x = 735y ,


what is the least possible value of x + y ?

7
Question 5

m and n are positive integers such that m > n.

Which of the following statements individually


provide(s) sufficient additional information to
determine whether m is divisible by n ?

Indicate all such statements.

 m = an, where a is a positive integer.


 m is a multiple of 6 and n is a
multiple of 3.
 Every prime factor of n is also a
prime factor of m.

302 |
DO NOT DISTRIBUTE
© TPR Education IP Holdings, LLC.
LESSON 7 MATH

SCRATCH PAPER

DO NOT DISTRIBUTE © TPR Education IP Holdings, LLC. | 303


MANUAL FOR THE GRE

NUMBER THEORY DRILL

Question 1 of 3

(2) (3) (5) (7) (8) (13) (17)


If m and n are integers and m = ,
7 91n
which of the following could be the value of n ?

14
21
38
68
96

Question 2 of 3

15!
If is an integer, what is the greatest possible
3m
value of m ?

4
5
6
7
8

Question 3 of 3

For a positive integer n, if the remainder is 1 when


2n is divided by 3, then which of the following
statements must be true?

Indicate all such statements.

 n2 + 1 is a multiple of 3.
 3n = (-3)n
 2n is an integer.

304 |
DO NOT DISTRIBUTE
© TPR Education IP Holdings, LLC.
LESSON 7 MATH

SCRATCH PAPER

DO NOT DISTRIBUTE © TPR Education IP Holdings, LLC. | 305


MANUAL FOR THE GRE

SUMMARY
Slope

vertical change Δy Rise y −y



= = = 2 1
horizontal change Δx Run x −x
2 1

Equation of a line

y = mx + b

If you know two points, you can find the slope.

If you know a point and the slope, you can find any
other point on the line.

Arrangements & Combinations

1. Write dashes for each spot.

2. Label any restrictions.

3. How many options can go in each spot?

4. Does order matter?

306 |
DO NOT DISTRIBUTE
© TPR Education IP Holdings, LLC.
Lesson 7 Verbal

DO NOT DISTRIBUTE
MANUAL FOR THE GRE

LONG PASSAGES

Reading Basic Approach


Step 1: Work the Passage
• Main Idea
• Structure
• Author’s side
7 Step 2: Understand the Question
• Subject
• Task
Step 3: Find the information in the passage that addresses the
task of the question.
Step 4: Use Process of Elimination
• Recycled language
• Extreme language
• No such comparison
• Reversals
• Outside knowledge
• Emotional appeals

308
DO NOT DISTRIBUTE
| © TPR Education IP Holdings, LLC.
LESSON 7 VERBAL

SCRATCH PAPER

DO NOT DISTRIBUTE © TPR Education IP Holdings, LLC. | 309


MANUAL FOR THE GRE

Though application of evolutionary theory to the Question 1


psychology of animals is generally done without
controversy, evolutionary psychology as it is The primary purpose of the passage is to
Line applied to human psychology is quite contentious.
explain the origins of evolutionary
5 Proponents of evolutionary psychology believe
psychology
that psychology must be rooted in biology. Just
as the body’s circulatory, digestive, endocrine, resolve a dispute regarding acceptable
immune, lymphatic, muscular, nervous, forms of evidence
reproductive, respiratory, skeletal, and urinary reconcile the differences between two
10 systems are evolved adaptations resulting from methods seeking to explain the same
natural selection or sexual selection, so too must phenomenon
the seemingly inherent psychological mechanisms describe a field of research and caution
7 be the result of evolution. These psychologists against its dismissal
believe that natural selection has engendered argue the importance of a debate between
15 many cognitive modules in the brain, ranging from scientists regarding the application of one
language-acquisition modules to cheater-detection methodology to another
modules. Survivability and sexual selection
determine which modules are passed on.
Question 2
Some critics offer several objections to
20 extending this application to humans. For It can be inferred from the passage that the author
instance, humans evolved during a period—the believes which of the following?
Pleistocene—about which very little essential
demographic information on humans is Evolutionary psychology has some merit
known. Additionally, some accuse evolutionary but has failed to bridge the divide between
25 psychologists of proffering “just-so stories”— the social sciences and the natural
internally consistent hypotheses that, nevertheless, sciences.
have no other supporting evidence. This, skeptics Evolutionary psychologists should be
argue, can lead to contradictory conclusions. admonished for failing to justify their
For example, such behaviors as monogamy can hypotheses with more than “just-so
30 perpetuate genes, but so too can infidelity. Lastly, stories.”
some critics accuse evolutionary psychology of Evolutionary psychology should be
ethnocentrism since many traits once considered restricted to non-human animals.
universal have turned out to be culturally Evolutionary psychology should not be
dependent. uniformly rejected.
35 Though evolutionary psychology remains The debate between proponents of
controversial, many detractors confess their evolutionary psychology and its detractors
inexperience and lack of firsthand knowledge is intractable.
with the discipline. Admittedly, the quality of work
in this field has been uneven, but, as Edouard
40 Machery stated, “the heuristics and the strategies
of confirmation used by evolutionary psychologists
are on a firm grounding.“

310
DO NOT DISTRIBUTE
| © TPR Education IP Holdings, LLC.
LESSON 7 VERBAL

SCRATCH PAPER

DO NOT DISTRIBUTE © TPR Education IP Holdings, LLC. | 311


MANUAL FOR THE GRE

One of several frames of reference utilized by Question 1


living creatures in the effort at orientation is
the direction of gravity. In human beings, this According to the passage, motion sickness is
Line orientation is provided by the vestibular system of induced by
5 the inner ear, a labyrinth of chambers comprising
delayed compensation of the vestibular
the semicircular canals, the utricle, and the
­system
saccule. The three semicircular canals are oriented
perpendicularly to one another and provide the asynchronous pairing of information from
locus of balance. In essence, these canals form a two systems
10 system of Cartesian coordinate axes along which the non-perpendicular orientation of the
the brain maps the body’s horizontal, vertical, head with respect to the body
and lateral displacements. A mass of particulate a failure to register linear motion and the
7 material presses down on membranes, further direction of gravity
allowing the mind to register linear motion and disorientation caused by pressure in the
15 the direction of gravity. By utilizing this system of inner ear
mental signals, we manage to maintain a sound
sense of orientation despite the fact that our
For the following question, consider each of the
heads are seldom perfectly perpendicular to the
choices separately and select all that apply.
plane of gravitational pull.
20 Yet this complex system has its imperfections.
As anyone will attest, watching television Question 2
while lying sideways is taxing on the brain,
while reading at the same inclination is all but According to the passage, the vestibular system of
impossible unless one tilts the book to match the the inner ear allows humans to accomplish which of
25 angle of sight. Another instance of the system’s the following?
failure is motion sickness.
 Track forward movement
When signals from the retina’s frame and
signals from the inner ear’s frame fail to coincide,  Pinpoint the origin of a sound
the result is motion sickness. The brain is flooded  Maintain spatial orientation
30 with contradictory signals: While the eye indicates
that the body is at rest, the inner ear signals
just the opposite. The simple cure for this is to Question 3
restore both systems of signals to synchrony. It can be inferred from the passage that the utricle
This is accomplished by redirecting the gaze to and the saccule
35 the exterior of the moving vehicle, allowing the
eye to match the information contributed by the are solely responsible for the maintenance
vestibular system. of balance
correct for gravitational orientation
provide axes of spatial orientation
are integral with the semicircular
canals
function only in tandem with visual signals

312
DO NOT DISTRIBUTE
| © TPR Education IP Holdings, LLC.
LESSON 7 VERBAL

SCRATCH PAPER

DO NOT DISTRIBUTE © TPR Education IP Holdings, LLC. | 313


MANUAL FOR THE GRE

Question 4

The main function of the passage as a whole is to

account for the failure of the vestibular


system of the inner ear
reconcile discordant theories about spatial
orientation
investigate the impact of stresses on the
vestibular system of the inner ear
consider the limitations of one bodily
system
7 describe the various frames of reference
used to orient the body

Question 5

Which of the following best describes the


organization of the passage?

A system is described, details are provided,


and possible alternative explanations are
­considered.
A system is described, and its
imperfections are explained and contrasted
with another system’s.
A system is described, its components
are enumerated, and an explanation for its
purpose is introduced.
A system is described, and examples
suggesting that the system is inadequate
are considered.
A system is described and weaknesses of
that system are then exposed.

314
DO NOT DISTRIBUTE
| © TPR Education IP Holdings, LLC.
LESSON 7 VERBAL

SCRATCH PAPER

DO NOT DISTRIBUTE © TPR Education IP Holdings, LLC. | 315


MANUAL FOR THE GRE

The vermiform appendix is an unctuous dead-end physicians who attempt to breach the domain
sac that hangs between the small and large 55 of the evolutionary biologist and engage in
intestines. Some have concluded that the phylogenetic analysis of the appendix do so with a
Line appendix is the evolutionary remains of a larger misguided and long since abandoned assumption
5 structure, the cecum, which was once used for that evolutionary “progress” accompanies
digesting food by ancestral organisms long-since evolutionary change.
extinct. Some scientists, such as Charles Darwin
in On the Origin of Species (1859), classify the
appendix as a vestigial organ. However, such a Question 1
10 classification does not enjoy universal support. In According to the author, which of the following is
fact throughout medical history, several possible true regarding the vermiform appendix?
purposes for the appendix have been offered,
7 evaluated, and refuted, including neuromuscular, It aids the body in immunization.
endocrine, and exocrine functions. A consensus Its immunizing functions are rendered
15 among medical specialists has begun to emerge, useless by modern sanitation, giving it the
speculating that the appendix serves some sort appearance of a vestigial organ.
of gastrointestinal immune function, citing the
organ’s concentrations of lymphoids and its highly
It is as large as primate ceca.
vascular structure as apparently homologous It contributes significantly to cellulose
20 features between it and primate ceca. That the fermentation.
appendix may help facilitate proper development It is a vestigial organ.
of the immune system in young mammals has
been intimated by some studies on animals,
though not conclusively proven. Question 2
25 Utilizing cladistics, a new approach to The author suggests that the reason medical
evolutionary biology, which uses genetic researchers believe that the vermiform appendix is
information in combination with a variety of other not vestigial is ultimately explained by
data to evaluate biological relationships that
emerge over the ages, Parker and his colleagues recent studies that depict the immunizing
30 found supporting evidence that further suggests function of the appendix in humans
that the appendix may confer some evolutionary incontrovertible evidence that
benefit. That the appendix has existed for at demonstrates immune function
least 80 million years and has evolved at least their correct application of cladistics
twice, once among Australian marsupials and later the fact that doctors receive no training in
35 among rats, certain other rodents, and particular evolutionary biology and are limited solely
primates is, by itself, not compelling evidence that to expertise regarding human anatomy and
the appendix must serve a function. physiology
Admittedly the appendix contains a partial ignorance on the part of medical
concentration of lymphoid tissue, but such studies researchers regarding the field of
40 as those conducted by Andersson and Buergal evolutionary biology and its precise
provide evidence that the appendix is actually taxonomy
maladaptive; this lymphoid tissue seems prone to
pathological inflammatory states. Moreover, if the Question 3
human appendix were not vestigial, it would be In the context, the “breach” mentioned on line 54
45 as large and developed as the cecum is in other most clearly refers to
primates and would contribute significantly to
cellulose fermentation. the penetration of foreign bodies in an
However, evolutionary biology allows for a organism
vestige to be a complex or specialized structure— the perforation of a ruptured appendix
50 as, for example, in the wings of an ostrich
the misguided research conducted by
or the eyes of the blind cavefish—yet still be
Parker and his colleagues
rudimentary and degenerate relative to the same
homologous structures in other organisms. Those the lack of communication between
evolutionary biologists and physicians
the interdisciplinary research conducted by

DO NOT DISTRIBUTE
doctors
316 | © TPR Education IP Holdings, LLC.
LESSON 7 VERBAL

SCRATCH PAPER

DO NOT DISTRIBUTE © TPR Education IP Holdings, LLC. | 317


MANUAL FOR THE GRE

ID THE REASONING QUESTIONS

Arguments Basic Approach for ID the Reasoning Questions


Step 1: Identify the Question Type
• reasoning
• role played
• proceeds by
7
Step 2: Work the Argument
• Conclusion
• Premise
Step 3: Know what the Answer Needs to Do
• Describes the reasoning in the argument
Step 4: Use Process of Elimination
• partial matches

Question 1
Many people feel that hard work deserves a
reward. This, however, is not true. Hard work
should be its own reward, which means that work
intrinsically gives the worker a reward when it is
performed satisfactorily.

Which of the following best describes the


reasoning strategy used in the argument?

Claiming that what is commonly believed


is false for that very reason
Pointing out the inconsistencies in a
counterargument
Challenging the definition of the word
reward as something extrinsic to a task
Repeating a premise which was
previously stated
Showing that a popular belief is not as
popular as presumed

318
DO NOT DISTRIBUTE
| © TPR Education IP Holdings, LLC.
LESSON 7 VERBAL

SCRATCH PAPER

DO NOT DISTRIBUTE © TPR Education IP Holdings, LLC. | 319


MANUAL FOR THE GRE

RESOLVE/EXPLAIN QUESTIONS

Arguments Basic Approach for Resolve / Explain Questions


Step 1: Identify the Question Type
• resolve
• explain
• paradox
7 Step 2: Work the Argument
• state the conflict
Step 3: Know what the Answer Needs to Do
• shows how both statements can be true at the
same time
Step 4: Use Process of Elimination
• out of scope
• makes the conflict worse
• one sided

Question 1
Many people predicted that business at Jack’s
Bistro, previously the only restaurant in Santa
Monica, would suffer once its only competitor,
Cindy’s Cafe, opened across the street last
year. Yet, the average number of lunches
served at Jack’s Bistro has actually increased
significantly in the past 12 months.

Which of the following, if true, most helps to


explain the increase described in the argument?

Unlike Jack’s Bistro, Cindy’s Café serves


considerably more meals on weekends
than it does on weekdays. 
Jack’s Bistro has hired away a
significant proportion of the staff who
formerly worked at Cindy’s Cafe. 
The profit per meal is higher, on average,
for meals served at Jack’s Bistro than
for those served at Cindy’s Cafe.
The number of restaurant patrons in
Santa Monica has doubled in the past
18 months. 
Most of Jack’s Bistro’s patrons had
never dined in Santa Monica before this

DO NOT DISTRIBUTE
restaurant opened.
320 | © TPR Education IP Holdings, LLC.
LESSON 7 VERBAL

SCRATCH PAPER

DO NOT DISTRIBUTE © TPR Education IP Holdings, LLC. | 321


MANUAL FOR THE GRE

ARGUMENTS DRILL

Question 1 of 3 Question 3 of 3

The ancient Hrugians used water power in their In an effort to save energy, and thus money, many
border towns for various purposes. Inexplicably, people keep their electric freezers half-empty at all
however, they continued to use manual labor for the times, using them to store pre-frozen foods bought
same functions in their capital city of Avallone. at a store and nothing else. Yet, freezers that are
half-empty usually consume more energy than they
would if they were kept fully stocked.
Which of the following, if true, explains the
7 situation?
Which of the following, if true, contributes most
The king of Hrugia officially endorsed water to an explanation of the apparent discrepancy
power. described?
The Hrugians learned water power
technology from their neighbors the A person who normally maintains a half-
Pastelites. empty freezer would save a considerable
Water power was cheaper than manual amount of money by using a freezer that is
labor. half as large.
The border towns of any country must An electric freezer can operate efficiently
have stronger security than the towns in only if chilled air is free to circulate within
the interior of the country. the freezer compartment.
There was no major source of water in the A given volume of air in a freezer requires
city of Avallone. much more energy to maintain at a
temperature below freezing than does an
identical volume of frozen food.
Question 2 of 3 The door of a full freezer is likely to be
opened as often, and for longer periods
In 2013, the Highway Bus Company’s annual report of time, than is the door of a half-empty
noted an increase in A-One Miles, a figure that is freezer.
computed by multiplying the number of paying On average, it takes less energy to keep
passengers by the total number of miles traveled by food frozen than it does to get it from room
all Highway buses. However, Highway experienced temperature to a frozen state.
a decrease in the number of paying passengers per
bus trip and in the number of bus trips made during
2013.

Which of the following, if true, would resolve the


discrepancy?

There was an increase in the number of


canceled trips in 2013.
Highway employees, who ride for free,
comprised a greater percentage of all
Highway bus riders in 2013 than in
previous years.
The average passenger capacity of
Highway’s buses decreased in 2013 due to
changes in design.
The average length of a Highway bus trip
increased in 2013.
Highway raised fares between large cities
by twenty percent in 2013.

322
DO NOT DISTRIBUTE
| © TPR Education IP Holdings, LLC.
LESSON 7 VERBAL

SCRATCH PAPER

DO NOT DISTRIBUTE © TPR Education IP Holdings, LLC. | 323


MANUAL FOR THE GRE

SUMMARY
Reading Basic Approach

Step 1: Work the Passage

Step 2: Understand the Question

Step 3: Find the information in the passage that


addresses the task of the question.

Step 4: Use Process of Elimination

Arguments Basic Approach for ID the Reasoning Questions

Step 1: Identify the Question Type

• reasoning

• role played

• proceeds by

Step 2: Work the Argument

• Conclusion

• Premise

Step 3: Know what the Answer Needs to Do

• describes the reasoning in the argument

Step 4: Use Process of Elimination

• partial matches

324
DO NOT DISTRIBUTE
| © TPR Education IP Holdings, LLC.
LESSON 7 VERBAL

Arguments Basic Approach for Resolve / Explain Questions

Step 1: Identify the Question Type

• resolve

• explain

• paradox

Step 2: Work the Argument

• state the conflict

Step 3: Know what the Answer Needs to Do

• shows how both statements can be true at the


same time
Step 4: Use Process of Elimination

• out of scope

• makes the conflict worse

• one sided

DO NOT DISTRIBUTE © TPR Education IP Holdings, LLC. | 325


DO NOT DISTRIBUTE
Lesson 8
Math

DO NOT DISTRIBUTE
MANUAL FOR THE GRE

FULL SECTION MATH DRILL 1


Time: 35 minutes

Question 1

Quantity A Quantity B
5 5−1

6 6 −1

Quantity A is greater.
8 Quantity B is greater.
The two quantities are equal.
The relationship cannot be determined from the
information given.

Question 2

x < 1 and x ≠ 0

Quantity A Quantity B
x3 + 2 x4 + 2

Quantity A is greater.
Quantity B is greater.
The two quantities are equal.
The relationship cannot be determined from the
information given.

Question 3

x>0

Quantity A Quantity B
1
0.2% of x x
5
Quantity A is greater.
Quantity B is greater.
The two quantities are equal.
The relationship cannot be determined from the
information given.

328
DO NOT DISTRIBUTE
| © TPR Education IP Holdings, LLC..
LESSON 8 MATH

SCRATCH PAPER

DO NOT DISTRIBUTE © TPR Education IP Holdings, LLC. | 329


MANUAL FOR THE GRE

Question 4

The median income of a group of engineers who


work for XYZ Corporation is $4,500 greater than the
median income of a group of engineers who work
for ABC Corporation.

Quantity A Quantity B
The 80th percentile The 80th percentile
of the incomes of the of the incomes of the
group of engineers group of engineers
who work for XYZ who work for ABC
corporation corporation

8 Quantity A is greater.
Quantity B is greater.
The two quantities are equal.
The relationship cannot be determined from the
information given.

Question 5
20
16
15 15
15
Frequency

10 8
6
5

0
1 2 3 4 5

The graph above shows the frequency distribution of


integer values varying from 1 to 5.

Quantity A Quantity B
The average The median of the
(arithmetic mean) 60 values
of the 60 values

Quantity A is greater.
Quantity B is greater.
The two quantities are equal.
The relationship cannot be determined from the
information given.

330
DO NOT DISTRIBUTE
| © TPR Education IP Holdings, LLC..
LESSON 8 MATH

SCRATCH PAPER

DO NOT DISTRIBUTE © TPR Education IP Holdings, LLC. | 331


MANUAL FOR THE GRE

Question 6
N

2 13

3 13 O

M P
12

8 Quantity A Quantity B
The area of triangle The area of triangle
MNO MOP

Quantity A is greater.
Quantity B is greater.
The two quantities are equal.
The relationship cannot be determined from the
information given.

Question 7

Quantity A Quantity B
The sum of the even The sum of the odd
integers from 4 to integers from 3 to
200 201
Quantity A is greater.
Quantity B is greater.
The two quantities are equal.
The relationship cannot be determined from the
information given.

332
DO NOT DISTRIBUTE
| © TPR Education IP Holdings, LLC..
LESSON 8 MATH

SCRATCH PAPER

DO NOT DISTRIBUTE © TPR Education IP Holdings, LLC. | 333


MANUAL FOR THE GRE

Question 8

In 2011, the number of cars in India was approximately 10


percent of the number of cars in the United States, and
the population of India was approximately 400 percent
of the population of the United States. The number of
cars per capita (total number of cars divided by the total
population) in India in 2011 was approximately how many
times the number of cars per capita in the United States?

1

400
1
8
40
1

20
1

10
1

4

Question 9

(4y)°
k

25°
m

In the figure above, line k is parallel to line m. What is


the value of y ?
y=

Question 10

The numbers in data set T have a standard deviation


of 8. If a new data set is formed by subtracting 2 from
each number in T, what is the standard deviation of the
numbers in the new data set?

2
6
7
8

DO NOT DISTRIBUTE
10
334 | © TPR Education IP Holdings, LLC..
LESSON 8 MATH

SCRATCH PAPER

DO NOT DISTRIBUTE © TPR Education IP Holdings, LLC. | 335


MANUAL FOR THE GRE

Question 11

7− y 3y − 5
If = , which of the following could be the
3 y
value of y ?

7
5
2
−3
−5
8
Question 12

The total number of calories that Fred consumed on


Monday was 25% less than the number of calories
that he consumed on Tuesday. What is the ratio of the
number of calories Fred consumed on Tuesday to the
number of calories that he consumed on Monday?

1 to 4
3 to 4
4 to 3
4 to 1
25 to 1

Question 13

9x
A bakery makes x donuts daily. Each day of the
10
donuts are sold and the remainder are discarded. At

these rates, how many days will it take for the bakery

to discard 360 donuts, in terms of x ?

360

x
3,600

x
x

400
x

3,600
336
DO NOT DISTRIBUTE
| © TPR Education IP Holdings, LLC.. 400x
LESSON 8 MATH

SCRATCH PAPER

DO NOT DISTRIBUTE © TPR Education IP Holdings, LLC. | 337


MANUAL FOR THE GRE

Questions 14 to 16 are based on the following data.

LENGTH OF JOB TRAINING FOR WORKERS IN COUNTRY Y FOR TWO INDUSTRIES, 2013
Retail Industry IT Industry
Total = 6 million Total = 12 million
9 to 10 weeks
8% 10%
7 to 8 weeks
10% 12% 1 to 2
1 to 2 weeks
weeks 12% 5 to 6 weeks 35%
50% 18%
8
20% 25%
3 to 4 weeks

Note: The circle graphs show the distribution of workers who received on-the-job training
for at least 1 week in 2013, by length of job training, rounded to the nearest week.

Question 14

In the circle graphs, the degree measure of the


central angle of the sector representing the number
of workers who received job training for 3 to 4
weeks is how much greater for the IT industry graph
than for the retail industry graph?


10°
15°
18°
25°

Question 15

Which of the following could be the median number


of weeks of job training for IT industry workers who
received job training for at least 1 week?

1
3
5
7
9

338
DO NOT DISTRIBUTE
| © TPR Education IP Holdings, LLC..
LESSON 8 MATH

SCRATCH PAPER

DO NOT DISTRIBUTE © TPR Education IP Holdings, LLC. | 339


MANUAL FOR THE GRE

Question 16

If one of the workers in the retail and IT industries


who received job training for at least 1 week will be
randomly selected, what is the probability that the
person selected will be an IT industry worker who
received job training for 7 to 8 weeks?

0.06
0.08
0.12
0.22

8 0.88

Question 17

If |x − 2| < 4, which of the following could be the


value of x ?

Indicate all such values.

 −6
 −4
 −2
 −1
 5
 6

Question 18

The operation ◊ is defined for all integers a and b


as a ◊ b = a2 + a2b. If a and b are negative integers,
which of the following CANNOT be zero?

a ◊ b
(a − 2) ◊ b
(a + 1) ◊ b
a ◊ (b + 1)
a ◊ (b − 2)

340
DO NOT DISTRIBUTE
| © TPR Education IP Holdings, LLC..
LESSON 8 MATH

SCRATCH PAPER

DO NOT DISTRIBUTE © TPR Education IP Holdings, LLC. | 341


MANUAL FOR THE GRE

Question 19

A, B, and C are three points in a plane, and B is the


midpoint of line segment AC. Which of the following
is true about the set of all points in the plane that
are the same distance from point B as point B is
from points A and C ?

It contains only points A and C.


It contains four points.
It contains every point in the plane.
It is a line passing through point B.

8 It is a circle.

Question 20
What is the length of a diagonal of a rectangle
that has length 8 and perimeter 28 ?

342
DO NOT DISTRIBUTE
| © TPR Education IP Holdings, LLC..
LESSON 8 MATH

SCRATCH PAPER

DO NOT DISTRIBUTE © TPR Education IP Holdings, LLC. | 343


MANUAL FOR THE GRE

FULL SECTION MATH DRILL 2


Time: 35 minutes

Question 1

x<0

Quantity A Quantity B
x
10
x11

8 Quantity A is greater.
Quantity B is greater.
The two quantities are equal.
The relationship cannot be determined from the
information given.

Question 2

Quantity A Quantity B
(x − 3)(y − 5) (x − 5)(y − 3)

Quantity A is greater.
Quantity B is greater.
The two quantities are equal.
The relationship cannot be determined from the
information given.

Question 3

0.y represents the decimal in which the digit y is


repeated without end.

Quantity A Quantity B
1.0 − 0.4 0.6

Quantity A is greater.
Quantity B is greater.
The two quantities are equal.
The relationship cannot be determined from the
information given.

344
DO NOT DISTRIBUTE
| © TPR Education IP Holdings, LLC..
LESSON 8 MATH

SCRATCH PAPER

DO NOT DISTRIBUTE © TPR Education IP Holdings, LLC. | 345


MANUAL FOR THE GRE

Question 4

A bakery makes only two types of pies: apple and


cherry. The cost to bake each cherry pie is $0.25
less than three times the cost of each apple pie.

Quantity A Quantity B
The cost to bake 200 The cost to bake 250
cherry pies and 200 cherry pies
apple pies
Quantity A is greater.
Quantity B is greater.
8 The two quantities are equal.
The relationship cannot be determined from the
information given.

Question 5

Quantity A Quantity B
1 −4
1 −3
1 −2
− −
3 3 3
15
3

Quantity A is greater.
Quantity B is greater.
The two quantities are equal.
The relationship cannot be determined from the
information given.

Question 6

List A: 3, 7, 10, x, y
List B: 3, 7, 10, y

The average (arithmetic mean) of the numbers in list


A is one more than the average of the numbers in
list B.

Quantity A Quantity B
0 y

Quantity A is greater.
Quantity B is greater.
The two quantities are equal.
The relationship cannot be determined from the
information given.

346
DO NOT DISTRIBUTE
| © TPR Education IP Holdings, LLC..
LESSON 8 MATH

SCRATCH PAPER

DO NOT DISTRIBUTE © TPR Education IP Holdings, LLC. | 347


MANUAL FOR THE GRE

Question 7

The radius of circle O is 6 less than the radius of


circle N.

Quantity A Quantity B
36 The circumference
of circle N minus
the circumference of
circle O
Quantity A is greater.
Quantity B is greater.
8 The two quantities are equal.
The relationship cannot be determined from the
information given.

Question 8

T = {10, 11, 12, 13, 14, 16, 17, 18, 19, 20}

Quantity A Quantity B
The number of The number of
6-element subsets 7-element subsets
of T of T
Quantity A is greater.
Quantity B is greater.
The two quantities are equal.
The relationship cannot be determined from the
information given.

Question 9

In right triangle PQR, the ratio of the lengths of the


two legs is 1 to 4. If the area of triangle PQR is 8,
what is the length of the hypotenuse?

2 2
4
17
5
2 17

348
DO NOT DISTRIBUTE
| © TPR Education IP Holdings, LLC..
LESSON 8 MATH

SCRATCH PAPER

DO NOT DISTRIBUTE © TPR Education IP Holdings, LLC. | 349


MANUAL FOR THE GRE

Question 10

Three workers, Alan, Bob, and Chad, working


simultaneously at their respective constant rates,
can complete a job in 7 hours. Alan and Bob,
working simultaneously at their respective constant
rates, can complete the same job in 11 hours. How
many hours will it take Chad, working alone at his
constant rate, to complete the same job?

1.6
4.0
18.0
8 19.25
30.5

Question 11

If m and n are the two solutions of the equation

2+m 2+n
x2 − 7x + 10 = 0, what is the value of ?
m n
Give your answer as a fraction.

350
DO NOT DISTRIBUTE
| © TPR Education IP Holdings, LLC..
LESSON 8 MATH

SCRATCH PAPER

DO NOT DISTRIBUTE © TPR Education IP Holdings, LLC. | 351


MANUAL FOR THE GRE

Question 12

Two different positive integers, r and s, are selected


from the even integers that are less than 13.
If t = r − s and t is greater than 0, which of the
following could be the sum of r, s, and t ?

Select all such integers.

 0
 4
 6
 8
 10
8  12
 24
 26

Question 13

If x is an integer and 243x = 9y, which of the following


expresses y in terms of x ?

33x
x−2
3
33(x−2)
35x−2
35(x−2)

352
DO NOT DISTRIBUTE
| © TPR Education IP Holdings, LLC..
LESSON 8 MATH

SCRATCH PAPER

DO NOT DISTRIBUTE © TPR Education IP Holdings, LLC. | 353


MANUAL FOR THE GRE

Questions 14 to 16 are based on the following data.

SALES FOR COMPANY T IN YEAR X


Distribution of Sales Sales by U.S. State or Region
Worldwide, by Region Total = $59.1 million

Rest of $14
World $12
12%

Sales (in millions)


$10
China United
17% States $8
45% $6

8 Europe
$4
$2
26%
$0
California Texas Oregon Utah Wyoming East Coast Rest of US
State or Region of United States

Question 14

Which of the following is closest to the percent of


sales for Company T in the United States that were
in states other than California, Texas, Oregon, Utah,
and Wyoming?

36%
40%
44%
48%
52%

Question 15

If Company T sales in Oregon were approximately 4


times Company T sales in France, then Company T
sales in France were approximately what percent of
Company T sales in Europe?

2%
6%
10%
18%
25%

354
DO NOT DISTRIBUTE
| © TPR Education IP Holdings, LLC..
LESSON 8 MATH

SCRATCH PAPER

DO NOT DISTRIBUTE © TPR Education IP Holdings, LLC. | 355


MANUAL FOR THE GRE

Question 16

Based on the information given, which of the


following statements about Company T sales in year
X must be true?

Indicate all such statements.

 Company T sales in the United States were greater


than Company T sales in Europe and China combined.
 Company T sales in Utah were less than Company T
sales in any East Coast state.
8  Company T sales in California and Texas combined
were more than 30 percent of Company T sales in the
United States.

Question 17

The greatest of the 25 positive integers in a certain


list is 32. The median of the 25 integers is 13. What
is the least possible average (arithmetic mean) of
the 25 integers?

6
7
8
9
10

356
DO NOT DISTRIBUTE
| © TPR Education IP Holdings, LLC..
LESSON 8 MATH

SCRATCH PAPER

DO NOT DISTRIBUTE © TPR Education IP Holdings, LLC. | 357


MANUAL FOR THE GRE

Question 18

Of the 15 workers available for a project, 4 workers


have an engineering degree. If 6 of the available
workers are selected at random for a project, which
of the following represents the probability that only
1 of the workers selected will have an engineering
degree?

6
1

8 15
4

6
1

15
6

15
4

15
6

4 11
1 5

15
4

4 11
1 5

15
6

358
DO NOT DISTRIBUTE
| © TPR Education IP Holdings, LLC..
LESSON 8 MATH

SCRATCH PAPER

DO NOT DISTRIBUTE © TPR Education IP Holdings, LLC. | 359


MANUAL FOR THE GRE

Question 19
From 2013 to 2014, the value of Jill’s house
increased by 15 percent and the value of Roxanne’s
house decreased by 10 percent. If the value of their
houses were equal in 2014, then the value of Jill’s
house in 2013 was what percent less than the value
of Roxanne’s house in 2013 ?

Give your answer to the nearest 0.1 percent.

8 Question 20

For a certain probability experiment, the probability

1
that event F will occur is and the probability that
4
3
event G will occur is . Which of the following
5

values could be the probability that the event F G

(that is, the event F and G) both occur?

Indicate all such values.

1

5
1

4
3

5
17

20

360
DO NOT DISTRIBUTE
| © TPR Education IP Holdings, LLC..
LESSON 8 MATH

SCRATCH PAPER

DO NOT DISTRIBUTE © TPR Education IP Holdings, LLC. | 361


MANUAL FOR THE GRE

FULL SECTION MATH DRILL 3


Time: 35 minutes

Question 1

The length of the base of a parallelogram is an


integer and the area of the parallelogram is 40.

Quantity A Quantity B
5 The number of
possible values for
8 the perimeter of the
parallelogram
Quantity A is greater.
Quantity B is greater.
The two quantities are equal.
The relationship cannot be determined from the
information given.

Question 2

a = (d + 2)2
b = (d – 2)2

Quantity A Quantity B
d +42
The average (arithmetic
mean) of a and b

Quantity A is greater.
Quantity B is greater.
The two quantities are equal.
The relationship cannot be determined from the
information given.

Question 3

x+3=y
x2
=4
y

Quantity A Quantity B
1 x

Quantity A is greater.
Quantity B is greater.
The two quantities are equal.
The relationship cannot be determined from the

DO NOT DISTRIBUTE
information given.
362 | © TPR Education IP Holdings, LLC..
LESSON 8 MATH

SCRATCH PAPER

DO NOT DISTRIBUTE © TPR Education IP Holdings, LLC. | 363


MANUAL FOR THE GRE

Question 4

The lengths of the sides of an isosceles triangle are


x, x, and y.

Quantity A Quantity B
2x + y 4x

Quantity A is greater.
Quantity B is greater.
The two quantities are equal.
The relationship cannot be determined from the
information given.
8
Question 5

At a PTA meeting, there are 5 fewer women than


men. The number of women is w.

Quantity A Quantity B
The total number of 2w – 5
people at the PTA
meeting
Quantity A is greater.
Quantity B is greater.
The two quantities are equal.
The relationship cannot be determined from the
information given.

Question 6

Points A, B, C, and D lie, in that order, on a number


line such that B is the midpoint of AC and the
distance between B and D is 8.

Quantity A Quantity B
The distance between The distance between
A and C C and D

Quantity A is greater.
Quantity B is greater.
The two quantities are equal.
The relationship cannot be determined from the
information given.

364
DO NOT DISTRIBUTE
| © TPR Education IP Holdings, LLC..
LESSON 8 MATH

SCRATCH PAPER

DO NOT DISTRIBUTE © TPR Education IP Holdings, LLC. | 365


MANUAL FOR THE GRE

Question 7

N is a set of 7 numbers, of which 3 are positive, 3


are negative and 1 is neither positive nor negative.

Quantity A Quantity B
The median of the The average (arithmetic
numbers in N mean) of the numbers in N
Quantity A is greater.
Quantity B is greater.
The two quantities are equal.
The relationship cannot be determined from the
8 information given.

Question 8

If the circumference of circle A is 6 and the


circumference of circle B is 12 , what is the ratio of
the area of circle A to the area of circle B ?

1

6
1

4
1

2

Question 9

In the xy-plane, if a quadrilateral has vertices (–3, 3),


(4, 3), (–2, –5), and (–3, –5), what is the perimeter of
the quadrilateral?

16
25
26
28
30

366
DO NOT DISTRIBUTE
| © TPR Education IP Holdings, LLC..
LESSON 8 MATH

SCRATCH PAPER

DO NOT DISTRIBUTE © TPR Education IP Holdings, LLC. | 367


MANUAL FOR THE GRE

Question 10

For which of the following functions f defined for all


numbers x does f(x) = f(−x) for all numbers x ?

Indicate all such functions.

 f(x) = x3 + x
 f(x) = x ( x + 1)
 f(x) = x ( x3+ x)
 f(x) = x2 ( x2 – 1)

8
Question 11

If m and n are integers and 0.01 percent of 10m is


equal to 10n, which of the following must be true?

m=n−4
n=m−4
m=n−2
n=m−2
n=m+2

368
DO NOT DISTRIBUTE
| © TPR Education IP Holdings, LLC..
LESSON 8 MATH

SCRATCH PAPER

DO NOT DISTRIBUTE © TPR Education IP Holdings, LLC. | 369


MANUAL FOR THE GRE

Question 12

34% 34%

14% 14%
2% 2%
−2 −1 0 1 2
8
The figure above shows the standard normal distribution, with
mean 0 and standard deviation 1, including approximate percents
of the distribution corresponding to the six regions shown.
If the random variable X is normally distributed with a mean of 390
and the value X = 510 is at the 85th percentile of the distribution,
which of the following is the best estimate of the standard devia-
tion of the distribution?

115
120
125
130
135

Question 13

A textbook manufacturer made a batch of textbooks, 40% of

which were math texts, with the remainder of the textbooks

consisting of an equal number of biology, physics, chemistry, and

history texts. During the next semester, if the manufacturer sold


4 1 2
of the math textbooks, of the biology textbooks,
of the
5 3 3
1 1
physics textbooks, of the chemistry textbooks, and of the
6 2
history textbooks, then what percent of the batch of textbooks

made were sold during the semester?

32%
52%
57%
62%
370
DO NOT DISTRIBUTE
| © TPR Education IP Holdings, LLC.. 67%
LESSON 8 MATH

SCRATCH PAPER

DO NOT DISTRIBUTE © TPR Education IP Holdings, LLC. | 371


MANUAL FOR THE GRE

Questions 14 to 16 are based on the following data.

SELECTED DATA FOR CAR SALES


Number of Cars Sold, by Color, in 2002 Total Revenue from All Car Sales, 2002-2005
Color Number of Cars 1170

Revenue (billions of dollars)


Blue 4.5 million 1160
1150
Brown 2.8 million
1140
Green 3.6 million 1130
Grey 4.2 million 1120
Purple 700 thousand 1110
8 Red 5.8 million
1100
1090
Silver 6.7 million 1080
Tan 1.9 million 1070
Yellow 800 thousand 1060
2002 2003 2004 2005
Total 31 million
Year

Question 14

In 2002, a car company that sold 15% of all the blue


cars that year priced the different models of its blue
cars for that year between $12,000 and $48,000. If
the revenue from sales of the company’s blue cars
that year was x billion dollars, then which of the
following represents all possible values of x ?

8.1 < x < 30.4


8.1 < x < 32.4
32.4 < x < 54
54 < x < 216
54 < x < 202.5

Question 15

What was the approximate percent increase in total


revenue from all car sales from 2002 to 2005 ?

0.08%
1%
8%
11%

80%

372
DO NOT DISTRIBUTE
| © TPR Education IP Holdings, LLC..
LESSON 8 MATH

SCRATCH PAPER

DO NOT DISTRIBUTE © TPR Education IP Holdings, LLC. | 373


MANUAL FOR THE GRE

Question 16

In 2002, if the average (arithmetic mean) price per


car for all cars sold was $30,000, then for which
of the following colors in 2002 was the number of
cars sold in that color greater than the total number
of cars sold that year for colors other than the nine
colors listed?

Indicate all such colors.

 Blue
 Green
 Grey
8  Purple
 Red
 Silver
 Yellow

Question 17

If x dollars is invested in a CD and the value of the


CD triples every 6 years, what will be the value of
the CD, in dollars, 18 years from today?

3x3
x3
6x2
9x
27x

Question 18

The distribution of the number of customers who


enter a hobby store per hour has a mean of 15
customers and a standard deviation of 3 customers.
Which of the following numbers of customers
who enter the hobby store in one hour is within 2
standard deviations of the mean of the distribution?

Indicate all such numbers.

 6
 10
 12
 15
 19
 24

374
DO NOT DISTRIBUTE
| © TPR Education IP Holdings, LLC..
LESSON 8 MATH

SCRATCH PAPER

DO NOT DISTRIBUTE © TPR Education IP Holdings, LLC. | 375


MANUAL FOR THE GRE

Question 19

In a certain sequence of numbers, each term after


the first is found by subtracting 4 from the previous
term and then multiplying the result by 3. If the
5th term in the sequence is –75, then which of the
following numbers is in the sequence?


−229

−221
−213
−29
8 −21

Question 20

When the reciprocal of a certain positive integer is


multiplied by 4, the resulting number is equal to the
result of moving the decimal point of the original
number two places to the left. What is the original
number?

376
DO NOT DISTRIBUTE
| © TPR Education IP Holdings, LLC..
LESSON 8 MATH

SCRATCH PAPER

DO NOT DISTRIBUTE © TPR Education IP Holdings, LLC. | 377


DO NOT DISTRIBUTE
Lesson 8
Verbal

DO NOT DISTRIBUTE
MANUAL FOR THE GRE

FULL SECTION VERBAL DRILL 1


Time: 30 minutes
Question 1

Accustomed to his wife’s _________ nature, Stanton


was unfazed when she quickly changed her opinion
and just as quickly changed it back.

florid
painstaking
8 laconic
effusive
erratic

Question 2

The teen’s explanations, each more _________


than the last, only exacerbated his parents’ ire:
They considered every increasingly elaborate and
tortuous excuse a new offense.

opaque
conventional
lucid
byzantine
boorish

Question 3

One example that (i)________ the common


impression of the “Dark Ages” as a period of
intellectual stagnation is the development of
Carolingian minuscule, the first recognizably
modern handwriting. So (ii)________ was this new
form of writing—the most ancient surviving text of
nearly every classical literary work is in Carolingian
miniscule—that Italian Renaissance scholars
assumed it was the original and called it “Roman.”

Blank (i) Blank (ii)


caricatures intermittent
belies equivocal
substantiates ubiquitous

380 |
DO NOT DISTRIBUTE
© TPR Education IP Holdings, LLC.
LESSON 8 VERBAL

SCRATCH PAPER

DO NOT DISTRIBUTE © TPR Education IP Holdings, LLC. | 381


MANUAL FOR THE GRE

Question 4

Despite the recent (i)________ of ads that attack


political adversaries by name, many analysts have
begun to regard the strategy with skepticism.
Especially in crowded primaries, in which name
recognition is vital, they warn that using scarce
campaign resources to publicize one’s opponent—
even to (ii) ___________ could easily backfire.

Blank (i) Blank (ii)


paucity condemn
novelty laud
8 proliferation recount

Question 5

Economic policy is primarily reactionary, designed


solely to avoid downturns rather than foster upturns.
As a result, economic policies tend to be
(i)________ the previous crisis, rather than focusing
on (ii)________ management of the current boom.
Consequently, unrestrained financial growth leads
to rampant speculation, and soon even seemingly
(iii)________ investments collapse as another
recession begins.

Blank (i) Blank (ii) Blank (iii)


ignorant of prudent solid
responses to sparing tenuous
peripheral to irrational unstable

Question 6

An increase in the cost of food staples would have


(i)________ consequences for the general population.
Although such an increase would (ii)________ the
need for the steep agricultural subsidies that the
government currently pays farmers, it is unlikely that
the savings would be passed along to the taxpayers
to help (iii)________ increased food expenditures.

Blank (i) Blank (ii) Blank (iii)


unexpected squander augment
adverse abate offset
advantageous drive flag

382 |
DO NOT DISTRIBUTE
© TPR Education IP Holdings, LLC.
LESSON 8 VERBAL

SCRATCH PAPER

DO NOT DISTRIBUTE © TPR Education IP Holdings, LLC. | 383


MANUAL FOR THE GRE

Human-to-human blood transfusions in the Question 8


nineteenth century were plagued by two major
problems: first, the necessity of transfusing blood The passage suggests that the author believes
Line as soon as it was drawn, due to coagulation, which of the following about transfusions?
5 and second, the occasionally fatal hemolytic
reactions, often accompanied by fever or nausea, ABO and Rh matching, when used together,
that afflicted many patients. Rudimentary are sufficient to ensure a successful
anticoagulants were developed as early as 1860, transfusion.
solving the first obstacle, and Landsteiner’s White blood cells are classified according
10 classification of ABO blood groups in 1900 was to different classifications than are the
presumed capable of concluding the second. other components of blood.
However, even after eliminating disbursement Before classifications were understood, the
of mismatched blood due to clerical error, a majority of blood transfusion recipients
small percentage of recipients still experienced died.
8 15 symptoms that paralleled those of a hemolytic Accurate blood group matching during
reaction. Later advances in blood analysis revealed transfusions will eliminate the chance of
not only that there are numerous additional blood having a hemolytic reaction, or symptoms
groups with which donors may be incompatible, similar to one.
notably the Rh system, but also that white blood
Donor-to-recipient blood compatibility
20 cells may attack new blood from even seemingly
depends more on some blood groups than
suitable donors.
others.

Question 7
Question 9
The author mentions the difficulties associated with
The primary purpose of the passage is to
blood transfusions in order to
highlight two major problems with human
criticize the simplicity of early attempts at
blood transfusions
transfusions
describe the origins of human blood
present supplemental information that transfusions
could be used to help reduce these examine the difficulties associated with
difficulties transfusing different blood types
suggest that it will never be possible to recommend improvements to human
understand all the causes of a hemolytic blood transfusion methods
reaction illustrate the problems presented by blood
argue for increased measures of type in human blood transfusions
compatibility across various blood groups
describe best practices for blood
transfusions in order to prevent the
difficulties

384 |
DO NOT DISTRIBUTE
© TPR Education IP Holdings, LLC.
LESSON 8 VERBAL

SCRATCH PAPER

DO NOT DISTRIBUTE © TPR Education IP Holdings, LLC. | 385


MANUAL FOR THE GRE

Despite his unquestioned preeminence in Rome Question 10


from 31 bce to 14 ce, Augustus took pains to
present himself not as king, dictator, or The passage suggests which of the following about
Line emperor, but as princeps—first among equals. traditional republican values in the era of Augustus?
5 The exact nature of Augustus’s authority has
long been a subject of debate among historians. Roman nobles could only achieve status
In particular, much attention has been paid to through office-holding.
the role played by Augustus’s provincial reform An autonomous Senate was considered a
in both consolidating his power and furthering significant element.
10 his public image as a champion of traditional Traditional republican values allowed the
republican values. Emperor to have proconsular imperium
The most visible element of the provincial over the entire Empire.
reform, carried out between 27 and 23 bce, was The army should be evenly distributed
the division of the Empire into two types of throughout the Empire.
8 15 provinces: imperial and Senatorial. In imperial Traditional republican values were rooted in
provinces, which included such troublesome the dual mythos of the farmer-soldier.
frontier areas as Spain, Gaul, and Syria, Augustus
retained the highest official form of power,
proconsular imperium. Senatorial provinces, Question 11
20 conversely, were governed by proconsuls chosen
by the Senate, thus reflecting the time-honored Which of the following questions can be answered
republican belief in the balance of power. In by information provided in the passage?
addition to restoring prestige to the Senate as a
body, this development also opened up one of Why did Augustus resign the Roman
25 the primary offices by which Roman aristocrats consulship in 23 bce?
had traditionally achieved status, as did What accounts for the preservation of the
Augustus’s resignation of the Roman consulship Cyrene Edicts?
in 23 bce, ending his decade-long monopolization What were some of the offices that
of one of the Republic’s highest offices. affected how Roman society judged its
30 No historian has suggested an equal division elites?
of power between Augustus and the Senate—the Was Augustus the first to present himself
frontier nature of the imperial provinces meant as princeps?
that the provinces held most of the army—but Were there more imperial or Senatorial
this division was widely held to be real by provinces?
35 modern observers. In the words of Mommsen,
the greatest classicist of the nineteenth century,
Augustus had created a “dyarchy,” or joint Question 12
rule between princeps and Senate. This vision
dominated accounts of Augustus’s rule until the According to the passage, the Cyrene Edicts are
40 discovery of a series of inscriptions in North important because they
Africa in the early 1920s.
prevented Senators from being empanelled
These inscriptions, which came to be known as jurors in imperial provinces
as the Cyrene Edicts, date to around 6 bce, more
than a decade after Creta et Cyrenaica was suggested that Syme’s views on imperial
45 created as a Senatorial province. Nonetheless, authority were too extreme
they show Augustus intervening in such local conclusively refuted Mommsen’s
issues as the empanelling of jurors. The degree assessment of provincial reorganization
to which the discovery of the Cyrene Edicts authorized Augustus to reorganize the
shattered the historical orthodoxy is best seen Roman provinces
50 in Syme’s epoch-making 1974 work, The Roman cast doubt on the clear distinction between
Revolution. Syme contends that Augustus was, in Senatorial and imperial provinces
fact, granted proconsular imperium over the
entire Empire, and that Senatorial proconsuls
were effectively his legates.

386 |
DO NOT DISTRIBUTE
© TPR Education IP Holdings, LLC.
LESSON 8 VERBAL

SCRATCH PAPER

DO NOT DISTRIBUTE © TPR Education IP Holdings, LLC. | 387


MANUAL FOR THE GRE

Question 13 Question 16

Every new scientific theory that challenges the Audiences were rapt during the first half of the
reigning orthodoxy is viewed as _________ until it movie; it was only after the jejune plot twist that
is supported by mounting evidence and eventually their interest began to _________.
adopted as truth.
 flag
 dichotomous  wax
 suspect  swell
 heretical  prevaricate
 middling  converge
 critical  ebb
 inconsequential
8 Question 17
Question 14
As evidence of the severe damage inflicted on the
The concepts of “please” and “thank you” are usually American automobile industry by competition from
taught together in the same lesson when we are foreign automakers, an expert pointed to a 27%
young, which makes it ironic that the former drop in sales of American-made cars in the state of
is _________ used, while the latter is quite commonly California over the last year.
heard and read in daily communication.
Which of the following, if true, most seriously
 oft weakens the argument that the American
 rarely automobile industry has been weakened by foreign
competition?
 frequently
 seldom Only 68% of Californians over the age of
 redundantly 16 own cars.
 significantly Fewer than one-third of California drivers
report having negative feelings about
American-made cars.
Question 15 Major improvements in public
Although our current permissive society allows more transportation were made in California
explicit expressions, the tradition of _________ lives eighteen months ago.
on. Over the last year, foreign auto companies
have significantly increased their
 rhetoric advertising budgets for American television
 euphemism commercials.
 excoriation Total car sales in the state of California did
not significantly decrease over the last year.
 garrulousness
 genteelism
 pragmatism

388 |
DO NOT DISTRIBUTE
© TPR Education IP Holdings, LLC.
LESSON 8 VERBAL

SCRATCH PAPER

DO NOT DISTRIBUTE © TPR Education IP Holdings, LLC. | 389


MANUAL FOR THE GRE

Until about 1870, the field of linguistics was Question 19


marked by three successive approaches.
Language was studied first through grammar, In which sentence does the author present a set of
Line which provided a set of rules for a particular methodologies that Saussure finds inadequate?
5 language, then through philology, which sought to
discover truths about the customs and traditions
of ancient cultures through language, and finally Question 20
through comparative linguistics, which posited
a series of affinities between seemingly diverse Responding to complaints from administrative
10 languages. assistants about back pain after long periods of
For Ferdinand de Saussure, such linguistic sedentary assignments, an office commissioned a
paradigms were unsatisfactory. Saussure theorized study to determine whether sitting for long periods
that the constructs of language are inherently of time induces back pain. Researchers asked the
arbitrary, and that to truly understand linguistics, assistants how often they completed assignments
8 15 one needed to retreat from the notion that the requiring long periods of sitting and how often
evolution of words themselves was significant. As they had experienced back pain during the past six
an example, he suggested that there is no real months. The study found that the assistants who
relationship between the word “tree” and the spent the most time engaged in sedentary activities
concept that people associate with that word. The were most likely to report frequent back pain. The
20 word derives its meaning purely from the fact that study thus concluded that sitting for long periods of
speakers use it as a sign for a mutually agreed time does indeed cause back pain.
upon concept. One objection Saussure anticipated
Which of the following, if true, would most
to his theory was the existence of
undermine the argument?
onomatopoeia, commonly defined as language
25 that sounds like its meaning (“bam,” “squish,”
A third question in the survey asked
etc.). Saussure dismissed this objection by
whether respondents engaged in a regular
suggesting that true onomatopoeia, as opposed
exercise regimen, and they all said that
to that produced by conscious attempts at
they did not.
onomatopoeic pronunciation, is both rare and to
30 some extent socially constructed. The researchers did not ask the
administrative assistants whether the back
pain they experienced was severe or mild.
Question 18 Another recent study has shown that
sitting for long periods of time can cause
Saussure’s dismissal of onomatopoeia as a leg pain.
challenge to his theory would be most strengthened
Administrative assistants who said that
if he did which of the following?
they experienced frequent back pain were
especially likely to over-report the amount
Showed that conscious attempts at
of time they spent sitting down.
onomatopoeic pronunciation were socially
constructed Administrative assistants who reported
frequent sedentary activity were no more
Offered an example of an onomatopoetic
dissatisfied with their jobs than were those
word that emerged independently in
who did not report frequent sedentary
several languages
activity.
Challenged the validity of grammar,
philology, and comparative linguistics as
linguistic disciplines
Demonstrated that not all linguists agree
on the frequency with which onomatopoeic
words occur in language
Provided an example of a particular word
generally regarded as onomatopoeic and
gave a detailed explanation of why it is not
true onomatopoeia

390 |
DO NOT DISTRIBUTE
© TPR Education IP Holdings, LLC.
LESSON 8 VERBAL

SCRATCH PAPER

DO NOT DISTRIBUTE © TPR Education IP Holdings, LLC. | 391


MANUAL FOR THE GRE

FULL SECTION VERBAL DRILL 2


Time: 30 minutes
Question 1

By always seeking new ideas from all board


members and by fostering unique approaches for
organizing his company, the CEO followed through
on his pledge to bring about a fashion of corporate
governance that prioritizes _______________.

innovation
8 results
merit
cooperation
stagnation

Question 2

Indulging in all things insalubrious, Frank Watson


partook openly in binge drinking and smoking—
actions so _______________ that his family and
friends worried that he would die soon if he were
not stopped.

vivacious
enervating
lugubrious
tantalizing
firm

Question 3

That the economist opted not to speak on the news


program does not indicate that she is (i) __________;
in fact, she (ii) ___________ many prominent
conferences in her field.

Blank (i) Blank (ii)


hermitic made mention of
avaricious lacked faith in
uncooperative took part in

392 |
DO NOT DISTRIBUTE
© TPR Education IP Holdings, LLC.
LESSON 8 VERBAL

SCRATCH PAPER

DO NOT DISTRIBUTE © TPR Education IP Holdings, LLC. | 393


MANUAL FOR THE GRE

Question 4

The sparrows gathered nesting materials that are


so (i)_____________ that other nesting birds ignored
them. Among these materials were twigs that the
sparrows at first (ii)_______________ because the
twigs were sharp and thorny. Even then, these did
not apparently affect the birds’ opinion of the nest,
since the sparrows, after they finished their nest,
inhabited it (iii)_______________ enough.

Blank (i) Blank (ii) Blank (iii)


substantial desired equanimously

8 inviting placed belatedly


spiny refused haphazardly

Question 5

When the usually (i)_____________ teenager was


confronted, it was only the threat of being grounded
that (ii)___________ her attitude. Her (iii)_________
regarding that matter implied that it was very much
an undesirable outcome.

Blank (i) Blank (ii) Blank (iii)


recalcitrant blanched obsequiousness
magnanimous paused celerity
insipid ameliorated amity

Question 6

Movie studios are wont to (i)______________. So it is


astonishing that Ascendant Pictures is advertising a
slew of movies with brand new intellectual property.
Such a willingness to (ii)_______________ novelty
is very (iii)_______________. Maybe the studio,
frequently derided for changing, exploiting, and
even ruining franchises, is turning over a new leaf
after the previous board of directors was let go last
quarter.

Blank (i) Blank (ii) Blank (iii)


earn profits embrace reassuring
zealously
make movies abuse rote
conservatively
treat their actors stifle frustrating
differently

394 |
DO NOT DISTRIBUTE
© TPR Education IP Holdings, LLC.
LESSON 8 VERBAL

SCRATCH PAPER

DO NOT DISTRIBUTE © TPR Education IP Holdings, LLC. | 395


MANUAL FOR THE GRE

What makes a cat purr? From the 1990s to the For the following question, consider each of the
mid-2000s, animal psychologists advocated choices separately and select all that apply.
external stimuli for purring, such as contentment,
Line either with respect to feeding or being petted,
5 with the intensity of the purring reflecting the Question 8
level of contentment. Other reasons considered
It can be inferred from the passage that Buffington
then included comfort—cats tend to purr when
and earlier researchers would agree with which of
they approach other cats to signal friendliness—
the following statements about cats’ purring?
and the cat’s perceived lack of security. However,
10 subsequent cat researchers have focused on
 Purring can help to restore weakened bone
internal prompts for purring. In following this
tissue.
approach, Tony Buffington cites sonographic
experiments in which bone growth was  Cats’ lack of security is chief among their
stimulated at certain frequencies. By comparing motivations to purr.
8 15 relative frequencies of purring and tissue  Cats’ purring can be used for more than
one purpose.
stimulation, Buffington found that the frequencies
for tissue stimulation and purring were very
similar and are probably linked.
For the following question, consider each of the
choices separately and select all that apply.
For the following question, consider each of the
choices separately and select all that apply.
Question 9

Question 7 The last sentence serves which of the following


functions in the passage?
According to the passage, which of the following
factors were considered from the 1990s to the  It describes how the experiments
mid-2000s to influence what situations in which cats conducted by Buffington were dissimilar to
purred? those of earlier researchers.
 It justifies the principles upon which the
 Cats’ feelings of danger experiments cited by Buffington were
 The comfort of cats based.
 Cats’ desire to rejuvenate their tissue  It provides evidence that demonstrates
how the experiments cited by Buffington
support his stance.

396 |
DO NOT DISTRIBUTE
© TPR Education IP Holdings, LLC.
LESSON 8 VERBAL

SCRATCH PAPER

DO NOT DISTRIBUTE © TPR Education IP Holdings, LLC. | 397


MANUAL FOR THE GRE

This passage is adapted from material published in Question 11


2015.
According to the passage, one significant implication
In 2015, scientists at Harvard inspired by
of the size of the metamaterial is that such a
miniaturization devised a new metamaterial—an
material would
artificial material that is designed to interact with
Line and manipulate light. This new material could have cast doubt on the practicality of quantum
5 potentially earth-shattering ramifications. This new computing
metamaterial can manipulate light at incredibly
small scales, breaking any boundaries that size assist in shuttling electrons around a circuit
or scale present. With size no longer an issue, allow faster than light speeds to be
this metamaterial can be applied to any sort of achieved
10 miniaturized technologies, such as microchips. In make the size of items less of a concern
addition, the physics of the metamaterial allow with respect to applying computational
it to squeeze and bend light very efficiently with power to them
8 little loss in speed; instead of electrons shuttling demonstrate that heat remains a limiting
around a circuit, the metamaterial allows light to factor in miniaturization
15 travel unencumbered because the metamaterial
is “zero-index” (a term physicists use to describe
materials that do not warp light, but do not enable
faster-than-light speeds). Without energy being
wasted in the system due to heat, many more
20 channels can be opened in even smaller spaces:
The metamaterial suffers less resistance and
heat. Finally, while quantum computing is still
not a reality, the metamaterial may help realize
it by enabling light to travel at vast distances
25 without losing information it might carry. Further
applications are necessary to determine if
quantum computing will indeed work.

Question 10

The primary purpose of this passage is to

compare several methods of energy travel


discuss the implications of a scientific
development
underscore limitations in present
technology
evaluate the strengths of widely-
used technology in the face of a new
development
discuss the process of developing a new
technology

398 |
DO NOT DISTRIBUTE
© TPR Education IP Holdings, LLC.
LESSON 8 VERBAL

SCRATCH PAPER

DO NOT DISTRIBUTE © TPR Education IP Holdings, LLC. | 399


MANUAL FOR THE GRE

Question 12 Question 15

In classics—a textually oriented discipline Despite having a reputation of defending the


that is ________________ ancient manuscripts, indefensible, the criminal lawyer was no __________:
mythological stories, and grammatical knowledge— He relied on accurate accounts of events to explain
many scholars have often been ignorant of useful his clients’ actions.
contributions made by archaeologists.
 dissimulator
 focused on  feigner
 occupied by  champion
 dismissive of  shirker
 pleased by  boaster
 opposed to  blusterer
 apathetic to
8
Question 16
Question 13
Politician: Five years ago, when we streamlined
People may think that computers are new inventions, the operations of the Transportation Security
but designs for computers actually date back several Administration to save money, critics claimed that
hundred years; the ________________ is not that the streamlining would make the TSA less able
computers exist but that they are used in almost to detect any dangerous passengers on airplanes
every daily appliance. and would thus lead to less safe air travel. The TSA
has compiled statistics from the years following
 benefit the streamlining that show that the critics were
 impasse incorrect. There was an overall decrease in all
 divergence reports of dangerous passengers and dangerous
cargoes that have made it onto airplanes.
 dilemma
 distinction
Which of the following, if true, most seriously
 trend
undermines the politician’s argument?

Question 14 When the TSA’s resources are considered


to be stretched, civilians in airports are less
Luther attempts what seems impossible; a book likely to report dangerous passengers.
filled with literary insights, in an exacting field, that The critics of the politician agree that
however is open to _________________. the TSA’s statistics regarding dangerous
passengers provide the most accurate data
 dissenters on safety in air travel.
 professionals In other nations where air traffic security
 novices departments have been similarly
 fanatics streamlined, the numbers of reported
 specialists dangerous travelers have generally risen
following streamlining.
 dilettantes
The streamlining of the TSA failed to
produce the budgetary gains promised by
the politician.
During the 5 years prior to streamlining the
TSA, the number of reports of dangerous
passengers had been steadily increasing
compared to the reports of dangerous
cargoes.

400 |
DO NOT DISTRIBUTE
© TPR Education IP Holdings, LLC.
LESSON 8 VERBAL

SCRATCH PAPER

DO NOT DISTRIBUTE © TPR Education IP Holdings, LLC. | 401


MANUAL FOR THE GRE

During the 1870s, most advocates of the land- Question 18


value tax, Henry George’s preferred method for
taxing landowners for non-economic revenues It can be inferred that the author of the passage
Line they derive from owning land, opposed lowering mentions “steel” (line 20) primarily to imply that
5 tariffs on imports. Although many advocates
of the land-value tax conceded that tariffs Carnegie’s business was run more
caused economic imbalances between labor efficiently than those of other business
and landowners, in practice they more often leaders who desired high tariffs
spoke in favor of tariffs in order to protect labor Carnegie, unlike most other business
10 constituents from international competition. Those leaders, saw the benefits of lower tariffs
advocates who wished to advance the land-value for his company clearly
tax and abolish tariffs had to make an unpopular Carnegie could have been an advocate for
case, claiming that the latter punished capital just lower tariffs because of the specific nature
as much as labor. of his products
8 15 In contrast to the land-value tax advocates, laborers were more likely to support lower
Andrew Carnegie claimed that a complete lack of tariffs in certain sectors of the economy
tariffs provided the greatest impetus for improved than others
output and profits. However, few employers political parties were successful in bringing
matched Carnegie’s vision in marketing a specific about lower tariffs
20 product—steel—that required low barriers to trade,
and few advocates succeeded in adjusting the
tariff issue politically. George’s treatise Progress Question 19
and Poverty, published in 1879, boasted millions
of copies sold around the world. Although many It can be inferred that the author of the passage
25 people claimed to be influenced by Progress and would probably agree with which of the following
Poverty between 1879 and 1897, no major political claims about the boast referred to in lines 23–24?
party in the United States summoned the political
will to actualize its message. It is based on inflated numbers of
interested readers globally in 1897.
It could create an erroneous perception
Question 17 of how successful the treatise was in
achieving the goals of its authors.
The passage is primarily interested in discussing
which of the following?
It overstates the increase of readers
between 1879 and 1897 who would
The relative strengths of two points of view advocate lower tariffs.
regarding an issue It exaggerates the formative power of
The potential benefits to labor in the 1870s political parties during the 1890s.
of a change in economic policy It is based on an exaggeration of the
The causes of a labor-capital dispute during number of readers in political parties in the
the 1870s United States in 1879.
The state of a disputed labor issue in the
1870s
The role of labor in bringing about
economic policy reform.

402 |
DO NOT DISTRIBUTE
© TPR Education IP Holdings, LLC.
LESSON 8 VERBAL

SCRATCH PAPER

DO NOT DISTRIBUTE © TPR Education IP Holdings, LLC. | 403


MANUAL FOR THE GRE

Question 20

According to the passage, the advocates of the land-


value tax who favored abolishing tariffs claimed that

neither the land-value tax nor lowering


tariffs would cause an improvement in the
equity of wealth between labor and capital
the impact that high tariffs had on labor
was felt similarly by capital
capital innately abused labor by means of
tariffs
advocates of the land-value tax were less
likely than business leaders to advocate
8 lower tariffs
reducing tariffs would be more helpful than
imposing a land-value tax in raising the
standard of living of laborers

404 |
DO NOT DISTRIBUTE
© TPR Education IP Holdings, LLC.
LESSON 8 VERBAL

SCRATCH PAPER

DO NOT DISTRIBUTE © TPR Education IP Holdings, LLC. | 405


DO NOT DISTRIBUTE
Lesson 8
Essays

DO NOT DISTRIBUTE
MANUAL FOR THE GRE

ANALYSIS OF AN ISSUE

Directions
You will be given a brief quotation that states or implies an issue of general interest, and you will also be
given specific instructions on how to respond to that issue. Trained GRE readers will evaluate your response
according to how well you

• respond to the specific task instructions


• consider the complexities of the issue
• organize, develop, and express your ideas
• support your position with relevant reasons and/or examples
8 • control the elements of standard written English

408 |
DO NOT DISTRIBUTE
© TPR Education IP Holdings, LLC..
LESSON 8 ESSAYS

SAMPLE ISSUE ESSAY PROMPTS


1. Issue: One should not expect respect for disregarding the opinions of others. Only when every
point of view is taken into consideration should people take action.
• Task: Write a response in which you discuss the extent to which you agree or disagree with
the statement and explain your reasoning for the position you take. In developing and
supporting your position, you should consider ways in which the statement might or might
not hold true and explain how these considerations shape your position.

2. Issue: An idea alone, no matter how great, is meaningless unless it is put into practice.
8
• Task: Write a response in which you discuss the extent to which you agree or disagree with
the claim. In developing and supporting your position, be sure to address the most com-
pelling reasons or examples that could be used to challenge your position.

3. Issue: A student who wishes to succeed in business should study anything but business while
in school. Once in the workplace, he or she will learn the skills of business, but the opportu-
nity to gain additional perspectives by studying other fields of knowledge while in school is too
valuable to pass up.
• Task: Write a response in which you discuss the extent to which you agree or disagree with
the recommendation and explain your reasoning for the position you take. In developing
and supporting your position, describe specific circumstances in which adopting the rec-
ommendation would or would not be advantageous and explain how these examples shape
your position.

4. Issue: Unexamined conservatism is far more dangerous than reckless change.


• Task: Discuss the extent to which you agree or disagree with the opinion stated above.
Support your views with reasons and/or examples from your own experience, observations,
or reading.

DO NOT DISTRIBUTE © TPR Education IP Holdings, LLC. | 409


MANUAL FOR THE GRE

Step 1: Brainstorm Examples


Here’s a sample prompt.

Issue: To achieve great success, one must reject conventional thinking.

On your scratch paper draw a T that looks like this.

Success must be Success can be conventional


unconventional
8

On one side write “I agree” and on the other write “I disagree.” Now take a moment to ask yourself, “What
does it mean to agree or disagree with this prompt?” Your essay prompt could come in one of three forms.
Here are some examples.

Extreme
“No one can possibly achieve success in the world by conforming to conventional
practices and conventional ways of thinking.”

Wishy-washy
“People should treat experts with suspicion and mistrust because experts who have
specialized in a particular field often have an overly narrow frame of reference.”

Open-ended
“The purpose of education is...”

When your thinking runs dry, use this checklist:


Me, Friends, Family, School, City, Country, Company,
Species, The Very Old/Young, Science, History, Literature

410 |
DO NOT DISTRIBUTE
© TPR Education IP Holdings, LLC..
LESSON 8 ESSAYS

Step 2: Organize
Thesis: I believe ______________________________________________________________________

______________________________________because________________________________________

____________________________________ as seen through the examples of _____________________,

______________________, and _____________________.

Example 1: _________________________________ Notes: _________________________________

Example 2: _________________________________ Notes: _________________________________

Example 3: _________________________________ Notes: _________________________________

DO NOT DISTRIBUTE © TPR Education IP Holdings, LLC. | 411


MANUAL FOR THE GRE

Step 3: Write
When it comes to writing, there are two things that all essays must have and a laundry list of things they
could have to augment your score.

Must have:
Topic Sentences

Transitions

Could have:
Specifics

Quotations

Commands

Analogies

Rhetorical Questions

Length

Big Words

412 |
DO NOT DISTRIBUTE
© TPR Education IP Holdings, LLC..
LESSON 8 ESSAYS

PRACTICE
Issue: Unexamined conservatism is far more dangerous than reckless change.

Task: Discuss the extent to which you agree or disagree with the opinion stated above. Support your views
with reasons and/or examples from your own experience, observations, or reading.

Step 1: Brainstorm

Step 2: Organize
Thesis: I believe ______________________________________________________________________

___________________________________________ because __________________________________

__________________________________________ as seen through the examples of ______________,

______________, and ______________.

Example 1: _________________________________ Notes: _________________________________

Example 2: _________________________________ Notes: _________________________________

Example 3: _________________________________ Notes: _________________________________

Step 3: Write
First Paragraph: ______________________________________________________________________

__________________________________________________________________________________

__________________________________________________________________________________

__________________________________________________________________________________

__________________________________________________________________________________

__________________________________________________________________________________

DO NOT DISTRIBUTE © TPR Education IP Holdings, LLC. | 413


MANUAL FOR THE GRE

ANALYSIS OF AN ARGUMENT
Question 1

The following appeared in an email from the president of


CheapSteak Foods.

Six months ago, we opened our first restaurant,


CheapSteak, which sells quickly-prepared and low-
priced steak sandwiches. Waitstaff have reported that
the price and convenience drew many customers away
from nearby restaurants, which are generally more
expensive. Although many of these restaurants have
8 since tried to compete with CheapSteak by lowering
their prices and adding drive-thru windows, CheapSteak
has made a profit every month since it opened. Given
our success with our first restaurant, we believe it is
time to open another CheapSteak restaurant and launch
a frozen food line.

Question 2

The following appeared in a memo to the head of marketing


for Dulcified Treacle Co., an established candy company.

In the last four years, gross sales in the candy


market have remained static, but ice cream, another
confectionary product, has experienced huge increases.
Specifically, the growth of boutique ice cream brands
specializing in unusual, savory ice cream flavors such
as pink peppercorn, basil, and ginger, has exploded. In
response, we gave out free samples of our new savory-
flavored candy chews at a number of national gourmet
food fairs, and in every case, our supplies ran out within
minutes. Therefore, we should jump to the forefront of
this trend and launch our savory candy chews nationally
at all of our retail outlets.

Question 3

The following appeared in a letter from a car owner to a


business associate.

Of the two leading used car lots in our town—Lyme


Cars and Limmon Automobiles—Lyme Cars is clearly
better. Lyme Cars has more than a square mile of cars
to choose from, whereas Limmon Automobiles has a
much smaller indoor showroom, with fewer cars to
choose from. Lyme also has twelve sales associates; in
contrast, Limmon only has four, two of whom work only
part-time. Cars from Lyme are better as well: Seven
years ago, I purchased a car from Limmon for $7,000,
and it broke down within a year. A car I purchased from
Lyme Cars last year cost only $4,000 and is still running
smoothly. Thus, if you want to buy an inexpensive,
quality used car, you should use Lyme Cars.
414 |
DO NOT DISTRIBUTE
© TPR Education IP Holdings, LLC..
LESSON 8 ESSAYS

Conclusion: _____________________________________________________

Premise: ________________________________________________________

_______________________________________________________________

Assumptions: ____________________________________________________

_______________________________________________________________

_______________________________________________________________

_______________________________________________________________

_______________________________________________________________
8

Conclusion: _____________________________________________________

Premise: ________________________________________________________

_______________________________________________________________

Assumptions: ____________________________________________________

_______________________________________________________________

_______________________________________________________________

_______________________________________________________________

_______________________________________________________________

Conclusion: _____________________________________________________

Premise: ________________________________________________________

_______________________________________________________________

Assumptions: ____________________________________________________

_______________________________________________________________

_______________________________________________________________

_______________________________________________________________

_______________________________________________________________

DO NOT DISTRIBUTE © TPR Education IP Holdings, LLC. | 415


MANUAL FOR THE GRE

An ancient, traditional treatment to help reduce


obesity—coconut oil—has recently been proven
effective. In a study completed last year, 50
volunteers who had issues maintaining a healthy
weight were fed a special diet rich in coconut oil for
two weeks. No volunteer was allowed to eat outside
foods during the trial, and each volunteer had his
or her weight measured several times each day.
After 2 weeks, the volunteers had lost an average
of 8 pounds each. Therefore, the study proves that
coconut oil reduces obesity within a short period of
time.

Write a response in which you discuss one or more alternative explanations that could rival the proposed
8 explanation and explain how your explanation(s) could plausibly account for the facts presented in the
argument.

Conclusion: _____________________________________________________

Premise: ________________________________________________________

_______________________________________________________________

Assumptions: ____________________________________________________

_______________________________________________________________

_______________________________________________________________

_______________________________________________________________

_______________________________________________________________

_______________________________________________________________

_______________________________________________________________

_______________________________________________________________

Essay Question Types


• What are the stated or unstated assumptions of this argument?
• What questions would need to be answered to determine whether the recommendation is
valid?
• What evidence is needed to evaluate the argument?
• What other explanations could rival the proposed explanation?

416 |
DO NOT DISTRIBUTE
© TPR Education IP Holdings, LLC..
LESSON 8 ESSAYS

SCRATCH PAPER
Introduction: _____________________________________________________

_______________________________________________________________

_______________________________________________________________

_______________________________________________________________

Flaw #1

What is the assumption: ____________________________________________

_______________________________________________________________

Why that is a problem: _____________________________________________


8
_______________________________________________________________

How to fix it: ____________________________________________________

_______________________________________________________________

Flaw #2

What is the assumption: ____________________________________________

_______________________________________________________________

Why that is a problem: _____________________________________________

_______________________________________________________________

How to fix it: ____________________________________________________

_______________________________________________________________

Flaw #3 (optional)

What is the assumption: ____________________________________________

_______________________________________________________________

Why that is a problem: _____________________________________________

_______________________________________________________________

How to fix it: ____________________________________________________

_______________________________________________________________

Conclusion: ______________________________________________________

_______________________________________________________________

_______________________________________________________________

DO NOT DISTRIBUTE © TPR Education IP Holdings, LLC. | 417


DO NOT DISTRIBUTE
Math Practice

DO NOT DISTRIBUTE
MANUAL FOR THE GRE

EXTRA PRACTICE DRILL

Distribution of Employees by Education and


Gender in 2010
Highest Level Completed Males Females
High School 15 26
Undergraduate Degree 29 34
Graduate Degree 16 12
Doctorate 5 8
Total 65 80

Question 1

If the total number of employees in 2010 was 16


percent greater than in 2009, how many total
employees were there in 2009 ?

employees

Question 2
A B C

F E D

In the figure above, equilateral triangle BEG is


inscribed in square ACDF. The area of square ACDF
is 16.

Quantity A Quantity B
The perimeter of the 10
shaded region
Quantity A is greater.
Quantity B is greater.
The two quantities are equal.
The relationship cannot be determined from the
information given.

420 | © The Princeton Review, Inc.

DO NOT DISTRIBUTE
MATH PRACTICE

SCRATCH PAPER

DO NOT DISTRIBUTE © TPR Education IP Holdings, LLC. | 421


MANUAL FOR THE GRE

Question 3

2x + 5y = 5
x – 2y = 7

Quantity A Quantity B
x y
Quantity A is greater.
Quantity B is greater.
The two quantities are equal.
The relationship cannot be determined from the
information given.

WEIGHT PLATES AT GYM X


Weight Pairs
10 lbs 1
15 lbs 1
25 lbs 1
55 lbs 2

Question 4

The table above shows how many pairs of different


weight plates are available at Gym X. Dan, Travis and
Kyle can bench press an average (arithmetic mean)
of 280 pounds with Dan bench pressing 315 pounds
and Travis bench pressing 20 pounds less than Dan.
If three pairs of weight plates are chosen at random
and loaded onto a bar that weighs 45 pounds, what
is the probability that Dan and Travis can bench press
the selected weight but Kyle cannot?
2

5

3

7
1
2

4
7

3

5

422 | © The Princeton Review, Inc.

DO NOT DISTRIBUTE
MATH PRACTICE

SCRATCH PAPER

DO NOT DISTRIBUTE © TPR Education IP Holdings, LLC. | 423


MANUAL FOR THE GRE

Question 5

The area of the circular region with center O is 25π.

Quantity A Quantity B
z 6

Quantity A is greater.
Quantity B is greater.
The two quantities are equal.
The relationship cannot be determined from the
information given.

Question 6

A sequence of numbers satisfies the equation


An = 2An − 1 + 1. If A4 = 10, what is the value of A1 ?

A1 =

Question 7

At the salad bar, Nicole can choose from 4 different


lettuce mixes, 8 different vegetables, 3 different
cheeses, 5 different dressings, and must choose
exactly one item from each category. She must
also decide whether to have croutons. How many
different salads could Nicole make at the salad bar?

424 | © The Princeton Review, Inc.

DO NOT DISTRIBUTE
MATH PRACTICE

SCRATCH PAPER

DO NOT DISTRIBUTE © TPR Education IP Holdings, LLC. | 425


MANUAL FOR THE GRE

Question 8

2 2
x+ y
What is the value of 3 9 ?
2 2
x+ y
5 15

Give your answer as a fraction.

Question 9

In the sequence a, b, c, d, the value of each term is


three times the value of the previous term. If a > 0
and d = a 18 , what is the value of a ?

3 2

2

3
2

3 2

9

2

Question 10
B C

(2x)º


A D

In rectangle ABCD above, if AC = 20, what is the


area of the shaded region?

50 2
50 3
100
100 3

DO NOT DISTRIBUTE
200
426 | © The Princeton Review, Inc.
MATH PRACTICE

SCRATCH PAPER

DO NOT DISTRIBUTE © TPR Education IP Holdings, LLC. | 427


MANUAL FOR THE GRE

Question 11

4≤j≤7

6≤k≤9

Which of the following could be the value of jk ?

Indicate all such values.

 10
 13
 16
 24
 48
 90

Advertising Budget for Company X by


Expenditure
Expenditure Percentage of Budget
Television 52%
Internet 11%
Print 32%
Other 5%
Total in 2006: $210,000

Question12

Of the money Company X spent on print

1
advertising in 2006, was spent on newspaper
2
3
advertisements and of the remainder was spent
4
on flyers. Approximately how many more thousands

of dollars was spent on newspaper advertisements

than on flyers?

8.4
13.6
25.2
33.6
39.9

428 | © The Princeton Review, Inc.

DO NOT DISTRIBUTE
MATH PRACTICE

SCRATCH PAPER

DO NOT DISTRIBUTE © TPR Education IP Holdings, LLC. | 429


MANUAL FOR THE GRE

Question 13

Which of the following is NOT a factor of 1515 ?

3
9
1 35
1 80
2 25

Question 14

40 percent of a certain number is 1,992. What is


95 percent of that number?

Question 15

n is an even integer, such that 5 < n < 15.

Quantity A Quantity B
(–3) n
–(3n)

Quantity A is greater.
Quantity B is greater.
The two quantities are equal.
The relationship cannot be determined from the
information given.

Question 16

Miki learns that b books cost d dollars. At this rate,


how much will Miki pay for seven books?

7b

d

7db

b

7d
7d

b
d

7b

430 | © The Princeton Review, Inc.

DO NOT DISTRIBUTE
MATH PRACTICE

SCRATCH PAPER

DO NOT DISTRIBUTE © TPR Education IP Holdings, LLC. | 431


MANUAL FOR THE GRE

Question 17

k is an integer.

Quantity A Quantity B
5 5
5+3 k
5 + 7k

Quantity A is greater.
Quantity B is greater.
The two quantities are equal.
The relationship cannot be determined from the
information given.

Question 18

A group of 5 coworkers plans to equally divide the proceeds,


after taxes, of a winning lottery ticket. If 40% of the total amount
of the proceeds is paid in taxes and each coworker receives
$1,800,000, what is the total amount of the prize?

$15,000,000
$17,500,000
$20,000,000
$22,500,000
$25,000,000

Question 19

If 0 < x < 1, which of the following expressions must be greater


than x ?

Indicate all such expressions.

 x2
 2x
 1 – x2
 1 – 2x
 x2 – 2
 2 – x2

432 | © The Princeton Review, Inc.

DO NOT DISTRIBUTE
MATH PRACTICE

SCRATCH PAPER

DO NOT DISTRIBUTE © TPR Education IP Holdings, LLC. | 433


MANUAL FOR THE GRE

Question 20

Of the 20 water heaters for sale at AAA Appliances, one-half are


both Energy Star certified and eligible for a federal tax rebate. If 14
water heaters are Energy Star certified, and the number of water
heaters eligible for the federal tax rebate is 3 times the number
that are neither Energy Star certified nor eligible for the federal tax
rebate, then how many of the water heaters are eligible for the
federal tax rebate?

4
12
14
15
18

Question 21

3
In a certain museum exhibit, of the artifacts are Paleolithic,
5
3
and the remaining 16 are Neolithic. Of the Paleolithic artifacts,
4
are Mediterranean. If 26 of the artifacts are Mediterranean, how

many Neolithic artifacts are NOT Mediterranean?

Question 22
B C

135º
A D

Which of the following could be the degree measurement of an


angle within parallelogram ABCD shown above?

Indicate all such degree measurements.


 45°
 55°
 80°
 100°
 115°
 135°
 225°

434 | © The Princeton Review, Inc.

DO NOT DISTRIBUTE
MATH PRACTICE

SCRATCH PAPER

DO NOT DISTRIBUTE © TPR Education IP Holdings, LLC. | 435


MANUAL FOR THE GRE

Question 23

B C

A D

In the figure above, if BC = 8 and CD = 4 2 and the degree


measurement of BCD is three times the degree measurement
of CDA, what is the area of parallelogram ABCD ?

16
32
32 2
32 3
64

Question 24

36
If Circle O has an area of , what is the circumference of Circle O ?

Question 25

If the area of the circular region shown above is 4π, what is the area
of the inscribed square with sides of length s ?
4
π2
2π2
8
16

436 | © The Princeton Review, Inc.

DO NOT DISTRIBUTE
MATH PRACTICE

SCRATCH PAPER

DO NOT DISTRIBUTE © TPR Education IP Holdings, LLC. | 437


MANUAL FOR THE GRE

Question 26
y

R (20, 5)
Q

x
S

The equation of the line in the rectangular coordinate system


5
above is y = − x + 10. What is the distance from point S to
4
point R ?

5
5 5
12
13
5 17

Question 27

12! + 11!
What is the value of ?
10!
23
23!
13!
143
264

438 | © The Princeton Review, Inc.

DO NOT DISTRIBUTE
MATH PRACTICE

SCRATCH PAPER

DO NOT DISTRIBUTE © TPR Education IP Holdings, LLC. | 439


MANUAL FOR THE GRE

Question 28

In a study of 130,000 teenagers, the number of hours


of television watched per teenager in the month
of October was normally distributed. If the average
(arithmetic mean) number of hours watched was 90
and the standard deviation was 5 hours, which of the
following statements must be true?

Indicate all such statements.

 18,200 teenagers watched between 95


and 100 hours of television in October.
 More than 25% of the teenagers watched
between 80 and 85 hours of television in
October.
 2,600 teenagers watched fewer than 80
hours of television in October.

Question 29

Colleen is hosting a party attended by 125 people,


75 of whom are male. At the end of the party, one
of the partygoers will be chosen at random to win a
1 1
prize. If of the male partygoers and of the female
5 2
partygoers arrive late, what is the probability that the
prize will be won by a partygoer who arrives late?

Give your answer as a fraction.

440 | © The Princeton Review, Inc.

DO NOT DISTRIBUTE
MATH PRACTICE

SCRATCH PAPER

DO NOT DISTRIBUTE © TPR Education IP Holdings, LLC. | 441


MANUAL FOR THE GRE

Question 30

Three students, Mark, Peter, and Wanda, are each attempting

to answer the same math problem. If their individual


1 2
probabilities of answering the problem correctly are ,,
4 5
3
and , respectively, then what is the probability that at least
8
one of the students will answer the problem correctly?

3

80
9

32
23

32
77

80
39

40

Question 31

A grandmother wants to display photos of her


grandchildren on her mantel. She has one photo of each
of her 11 grandchildren, but she can fit only four photos on
the mantel. How many different arrangements of photos
are possible?

330
484
7,920
1,663,200
9,979,200

Question 32

There are nine high school basketball teams in Johnston


County. If each team plays each of the other teams in the
county exactly once and each team also plays two games
against schools from a neighboring county, how many total
games are played?

36
54
90
108
126

442 | © The Princeton Review, Inc.

DO NOT DISTRIBUTE
Between Now and
Test Day

DO NOT DISTRIBUTE
MANUAL FOR THE GRE

PICK A SUBJECT AND MASTER IT


There are only about 20 concepts tested on the whole GRE. Some of these you are comfortable with and
some you are not. Between now and test day, pick one or two distinct topics you haven’t yet mastered and
master them.

444 |
DO NOT DISTRIBUTE
© TPR Education IP Holdings, LLC.
BETWEEN NOW AND TEST DAY

Verbal

IO Y
TY
PR I T
RI
W OR
GH E D
L O PRI
HI E R
T
AS
M

¨ ¨ ¨ Vocab
¨ ¨ ¨ Text Completion Strategy
¨ ¨ ¨ Sentence Equivalence Strategy
¨ ¨ ¨ Reading Comprehension Basic Approach
¨ ¨ ¨ Argument Question Basic Approach

Math
IO Y
TY
PR I T
RI
W OR
GH R E D
L O PRI
TE
AS
HI
M

¨ ¨ ¨ Math POE
¨ ¨ ¨ Plugging In
¨ ¨ ¨ Plugging In the Answers (PITA)
¨ ¨ ¨ Quantitative Comparison/Must Be
¨ ¨ ¨ Percents and Percent Change
¨ ¨ ¨ Ratios and Proportions
¨ ¨ ¨ Exponents and Square Roots
¨ ¨ ¨ Mean, Median, Mode, and Statistics
¨ ¨ ¨ Lines and Angles
¨ ¨ ¨ Triangles
¨ ¨ ¨ Quadrilaterals
¨ ¨ ¨ Circles
¨ ¨ ¨ Solid Geometry
¨ ¨ ¨ Coordinate Geometry
¨ ¨ ¨ Charts and Graphs
¨ ¨ ¨ Linear Equations and Inequalities
¨ ¨ ¨ Quadratic Equations
¨ ¨ ¨ Probability
¨ ¨ ¨ Rates
¨ ¨ ¨ Groups, Sequences, and Functions
¨ ¨ ¨ Number Theory
¨ ¨ ¨ Arrangements and Combinations

Essays
IO Y
TY
PR I T
RI
W OR
GH R E D
L O PRI
TE
AS
HI
M

¨ ¨ ¨ Issue Essay
¨ ¨ ¨ Argument Essay

DO NOT DISTRIBUTE © TPR Education IP Holdings, LLC. | 445


MANUAL FOR THE GRE

A FEW LAST THINGS TO NOTE

Practice Tests and Drills


Once you have taken them all, delete them all and do them again. The second time around, the score is
moot because you will have seen many of the questions. That’s okay. Practice your technique and use your
scratch paper.

The First Run Is a Practice Run


If possible, plan to take the GRE more than once. This allows you to relax the first time. If you get a great
score, you are done. If you need more points, you’ve got a second option.

Stick to the Plan


It’s easy to get stressed out on test day, so focus on the positive: You have a plan. You’ve applied it on the
practice tests. Stick to it. Go slowly. Focus on accuracy. Do the easy questions first. If you get stuck on a
problem, mark it and come back.

Standardized = Predictable
You have now seen every subject and question type you could see on test day. There should be no surprises.
Most of what you see will look very familiar.

446 |
DO NOT DISTRIBUTE
© TPR Education IP Holdings, LLC.
BETWEEN NOW AND TEST DAY

MATH REVIEW

Arithmetic Exponents
0: Even integer. Not prime. Not positive. Not Multiply
negative. Add
1: Odd positive integer. Not prime. Divide
2: Even prime integer. First prime. Subtract
Primes less than 20: 2, 3, 5, 7, 11, 13, 17, 19 Power
Multiply
Order of Operations (PEMDAS):
x1 = x
Parentheses x 0 = 1, for all x ≠ 0
E xponents 1x = 1
Multiplication & Division 0x = 0, for all x ≠ 0
Addition & Subtraction (–x)even = positive
(–x)odd = negative

Percents
Quadratics
% ÷ 100 x 2 + bx + c = 0 → (x + d)(x + e) = 0

of × (times) d+e=b de = c

is, are, was =


(x + y)2 = x 2 + 2xy + y2
what x, y, z (variable) (x – y)2 = x 2 – 2xy + y2
difference (x + y)(x – y) = x 2 – y2
Percent Change = × 100
original

Rate
Groups
Work/Distance
Total = Group 1 + Group 2 + Neither – Both
÷ ÷
Time × Rate

Probability
want
0≤ ≤1 Average
total

Probability of events A and B = A × B Total

Probability of mutually exclusive events A or B = A + B # of Average


things

DO NOT DISTRIBUTE
Probability of “at least once” = 1 – probability of never
© TPR Education IP Holdings, LLC. | 447
MANUAL FOR THE GRE

Triangles Quadrilaterals
Angles add up to 180°. Angles add up to 360°.

Perimeter: add up the sides


1
A = b×h
2 Area of parallelogram (rectangle or square) = b × h

Isosceles triangle: Two equal sides are opposite two


equal angles.
Circles
Similar triangles: If all angles of two triangles are the
same, then corresponding sides are proportional. d = 2r

C = pd = 2pr

A = pr 2
Third Side Rule
The length of any side of a triangle is greater than the
difference and less than the sum of the lengths of the Length of Arc = central angle × Circumference
other two sides. 360°

Area of Sector = central angle × Area of Circle


Right Triangles 360°
a2 + b2 = c 2

3 : 4 : 5; 6 : 8 : 10; 5 : 12 : 13 Coordinate Geometry


rise y 2 − y1
a : a : a 2 (45°:45°:90°) Slope: =
run x 2 − x1

45° Equation of a Line: y = mx + b


a a √2 (m is slope; b is y-intercept)

45°
a
Solids
Volume of Cube: V = s3
a : a 3 : 2a (30°:60°:90°)
Volume of Box: V = l × w × h

Volume of Cylinder: V = pr 2h
30°
2a Diagonal of Box: a2 + b2 + c 2 = d2
a √3

60°
a

448 |
DO NOT DISTRIBUTE
© TPR Education IP Holdings, LLC.
NOTES

DO NOT DISTRIBUTE
NOTES

DO NOT DISTRIBUTE
NOTES

DO NOT DISTRIBUTE
NOTES

DO NOT DISTRIBUTE
NOTES

DO NOT DISTRIBUTE
NOTES

DO NOT DISTRIBUTE
NOTES

DO NOT DISTRIBUTE
NOTES

DO NOT DISTRIBUTE
NOTES

DO NOT DISTRIBUTE
NOTES

DO NOT DISTRIBUTE

You might also like